Critical Thinking, Implementing Nursing Care, Nursing Diagnosis, Planning Nursing Care

Pataasin ang iyong marka sa homework at exams ngayon gamit ang Quizwiz!

Complex critical thinking is...

*Being able to recognize that alternative & conflicting solutions exist

What do intellectual standards ensure?

*Nurse obtains a complete database of information

Diagnostic reasoning means

*collecting pt data *analyzing that data *determining pt problems

What do professional standards refer to ?

*ethical criteria *evidence-based criteria *criteria for professional responsibility

Effective communication is built on __________

- courtesy -comfort -connection -confirmation

When is a branching set of questions used?

- during assessment -collects data & cluster cues -identifies emerging patterns & problems

What does a nurse do after diagnoses & collaborative problems are identified?

- establish plan of care -prioritize diagnoses & establish intervention -create goals & expected outcomes

What are clinical guidelines?

- evidence-based documents -protocols -guide decisions & interventions -establishes evidence-based interventions

How can you anticipate & prevent complications?

- identify risks -adapt interventions to situation -evaluate -initiate risk-prevention measures

What is important to know about interventions?

- know the : -purpose of each intervention -associated preassessment -postassessment risks -steps in performing intervention -current medical condition of the pt -expected response of pt ( so you can anticipate)

What are health care provider-initiated interventions?

- require specific nursing responsibilities & technical nursing knowledge

What is direct care intervention?

- treatment performed through interaction with pt -nurse-initiated -health-care initiated -collaborative approaches

data clustering

-A set of signs or symptoms gathered during assessment that you group together in a logical way. -are patterns of data that contain defining characteristics—clinical criteria that are observable and verifiable.

What is Hashimoto's thyroiditis?

-Chronic autoimmune thyroiditis -----Caused by the destruction of thyroid tissue by antibodies -Goiter present -Risk factor for hypothyroidism

Concept mapping is one way to:

-Connect concepts to a central subject -Relate ideas to patient health problems -Challenge a nurse's thinking about patient needs and problems -Graphically display ideas by organizing data

Sources of diagnostic error

-Data collection -Interpretation and analysis of data -Clustering -Diagnostic statement

4 steps to help you anticipate and prevent complications

-Identify risks to the patient. -Adapt interventions to the situation. -Evaluate the relative benefit of a treatment vs. the risk. -Initiate risk prevention measures.

If D.S. wants to have bariatric surgery, what risks does the surgery pose?

-Postanesthesia problems due to her weight and pain following surgery. -There will need to be psychologic support as well as medical follow up.

NANDA-I nursing diagnoses include:

-Problem focused -Risk -Health promotion

modifications of an existing written care plan

-Revise data assessment. -Revise the nursing diagnoses. -Revise specific interventions. -Determine how to evaluate whether you have achieved outcomes.

write SMART outcomes & goals:

-Specific -Measurable -Attainable -Realistic -Timed

preventative measures

-actions promote health and prevent illness to avoid the need for acute or rehabilitative health care. -includes assessment and promotion of the patient's health potential, application of prescribed measures, health teaching, and identification of risk factors for illness and/or trauma.

Patient-Centered Interview

-allows pt to set focus & initiate discussion

What are outcomes?

-desired responses pertaining to: -physiological -psychological -social -developmental -spiritual -indicate resolution of pt's health process

What is implementation?

-fourth step in nursing process -begins after development of plan of care -initiating interventions designed to achieve pt goals

controlling for adverse reactions

-is a harmful or unintended effect of a medication, diagnostic test, or therapeutic intervention -Before performing any skill or task, you need to know the possible effects or reactions that can occur.

What are independent nursing interventions?

-nurse can initiate without supervision/direction from others -autonomous -require order from health care provider

implementation process

-reassessing the patient -reviewing and revising the existing nursing care plan -organizing resources and care delivery -anticipating and preventing complications

Comprehensive Assessment

-structured database format -use of a problem-focused approach

66. The client with a below-the-knee amputation (BKA) has a large amount of bright red blood on the residual limb dressing and the nurse suspects an arterial bleed. Which intervention should the nurse implement first? 1. Increase the client's intravenous rate. 2. Assess the client's vital signs. 3. Apply a tourniquet above the amputation. 4. Notify the client's healthcare provider.

3. Apply a tourniquet above the amputation.

118. The nurse on the psychiatric unit observes one client shove another client. Which intervention should the nurse implement first? 1. Discuss the aggressive behavior with the client. 2. Document the occurrence in the client's chart. 3. Approach the client with another staff member. 4. Instruct the client to go to the unit's quiet room.

3. Approach the client with another staff member.

17. The new graduate nurse is assigned to work with an unlicensed assistive personnel (UAP) to provide care for a group of clients. Which action by the nurse is the best method to evaluate whether delegated care is being provided? 1. Check with the clients to see whether they are satisfied. 2. Ask the charge nurse whether the UAP is qualitied. 3. Make rounds to see that the clients are being turned. 4. Watch the UAP perform all the delegated tasks.

3. Make rounds to see that the clients are being turned.

37. The home health (HH) nurse notes the 88-year-old female client is unable to cook for herself and mainly eats frozen foods and sandwiches. Which intervention should the nurse implement? 1. Discuss the situation with the client's family. 2. Refer the client to the HH occupational therapist. 3. Request the HH aide to cook all the client's meals. 4. Contact the community's Meals on Wheels.

4. Contact the community's Meals on Wheels.

58. At 0830, the day shift nurse is preparing to administer medications to the client. Which action should the nurse take first? 1. Check the client's armband against the medication administration record (MAR). 2. Assess the client's IV site for redness and patency. 3. Ask for the client's date of birth. 4. Determine the client's last K+ level.

4. Determine the client's last K+ level.

119. The client in the operating room states, "I don't think I will have this surgery after all." Which intervention should the nurse implement first? 1. Have the surgeon speak with the client. 2. Ask the client to discuss the concerns. 3. Continue to prep the client for surgery. 4. Immediately stop the surgical procedure.

4. Immediately stop the surgical procedure.

67. The client who had surgery on the right elbow has no right radial pulse and the fingers are cold, the client complains of tingling, and she cannot move the fingers of the right hand. Which intervention should the nurse implement first? 1. Document the findings in the client's chart. 2. Elevate the client's right hand. 3. Assess the radial pulse with the Doppler. 4. Notify the client's healthcare provider.

4. Notify the client's healthcare provider.

70. The licensed practical nurse (LPN) is working in a surgical rehabilitation unit. Which nursing task would be most appropriate for the LPN to implement? 1. Bathe the client who is incontinent of urine. 2. Document the amount of food the client eats. 3. Conduct the afternoon bingo game in the lobby. 4. Perform routine dressing changes on assigned clients.

4. Perform routine dressing changes on assigned clients.

93. The male client in a long-term care facility complains that the staff does not listen to his complaints unless a family member also complains. Which action should the director of nurses implement? 1. Call a staff meeting and tell the staff to listen to the resident when he talks to them. 2. Determine who neglected to listen to the resident and place the staff member on leave. 3. Ignore the situation because a resident in long-term care cannot determine his needs. 4. Talk with the resident about his concerns and then initiate a plan of action.

4. Talk with the resident about his concerns and then initiate a plan of action.

106. The clinic nurse is reviewing the laboratory data of clients seen in the clinic the previous day. Which client requires immediate intervention by the nurse? 1. The client whose white blood cell (WBC) count is 9.5 mm3. 2. The client whose cholesterol level is 230 mg/dL. 3. The client whose calcium level is 10.4 mg/dL. 4. The client whose International Normalized Ratio (INR) is 3.8.

4. The client whose International Normalized Ratio (INR) is 3.8.

42. Which client warrants immediate intervention from the nurse on the medical unit? 1. The client diagnosed with an abdominal aortic aneurysm who has an audible bruit. 2. The client with adult respiratory distress syndrome (ARDS) who has bilateral crackles. 3. The client diagnosed with bacterial meningitis who has nucal rigidity and neck pain. 4. The client with Crohn's disease who has right lower abdominal pain and has diarrhea.

4. The client with Crohn's disease who has right lower abdominal pain and has diarrhea.

Implementation

4th step, Nurse initiates interventions to achieve goals

Nursing Process

5- step clinical decision- making approach

20. A nurse is developing nursing diagnoses for a patient. Beginning with the first step, place in order the steps the nurse will use. 1. Observes the patient having dyspnea (shortness of breath) and a diagnosis of asthma 2. Writes a diagnostic label of impaired gas exchange 3. Organizes data into meaningful clusters 4. Interprets information from patient 5. Writes an etiology a.1, 3, 4, 2, 5 b.1, 3, 4, 5, 2 c.1, 4, 3, 5, 2 d.1, 4, 3, 2, 5

ANS: A The diagnostic process flows from the assessment process (observing and gathering data) and includes decision-making steps. These steps include data clustering, identifying patient health problems, and formulating the diagnosis (diagnosis is written as problem or NANDA-I approved diagnosis then etiology or cause).

4. The nurse is reviewing a patient's plan of care, which includes the nursing diagnostic statement, Impaired physical mobility related to tibial fracture as evidenced by patient's inability to ambulate. Which part of the diagnostic statement does the nurse need to revise? a.Etiology b.Nursing diagnosis c.Collaborative problem d.Defining characteristic

ANS: A The etiology, or related to factor, of tibial fracture is a medical diagnosis and needs to be revised. The nursing diagnosis is appropriate because the patient is unable to ambulate. A collaborative problem is an actual or potential physiological complication that nurses monitor to detect the onset of changes in a patient's health status; there is no collaborative problem listed. The defining characteristic (subjective and objective data that support the diagnosis) is appropriate for Impaired physical mobility.

3. A nurse is preparing to carry out interventions. Which resources will the nurse make sure are available? (Select all that apply.) a.Equipment b.Safe environment c.Confidence d.Assistive personnel e.Creativity

ANS: A, B, D A nurse will organize time and resources in preparation for implementing nursing care. Most nursing procedures require some equipment or supplies. Before performing an intervention, decide which supplies you need and determine their availability. Patient care staff (assistive personnel) work together as patients' needs demand it. A patient's care environment needs to be safe and conducive to implementing therapies. Confidence and creativity are needed to provide safe and effective patient care; however, these are critical thinking attitudes, not resources.

2. The nurse is teaching a new nurse about protocols. Which information from the new nurse indicates a correct understanding of the teaching? a.Protocols are guidelines to follow that replace the nursing care plan. b.Protocols assist the clinician in making decisions and choosing interventions for specific health care problems or conditions. c.Protocols are policies designating each nurse's duty according to standards of care and a code of ethics. d.Protocols are prescriptive order forms that help individualize the plan of care.

ANS: B A clinical practice guideline or protocol is a systematically developed set of statements that helps nurses, physicians, and other health care providers make decisions about appropriate health care for specific clinical situations. This guideline establishes interventions for specific health care problems or conditions. The protocol does not replace the nursing care plan. Evidence-based guidelines from protocols can be incorporated into an individualized plan of care. A clinical guideline is not the same as a hospital policy. Standing orders contain orders for the care of a specific group of patients. A protocol is not a prescriptive order form like a standing order.

2. Which diagnosis will the nurse document in a patient's care plan that is NANDA-I approved? a.Sore throat b.Acute pain c.Sleep apnea d.Heart failure

ANS: B Acute pain is the only NANDA-I approved diagnosis listed. Sleep apnea and heart failure are medical diagnoses, and sore throat is subjective data.

28. The paramedics transport an adult involved in a motor vehicle accident to the emergency department. On physical examination, the patient's level of consciousness is reported as opening eyes to pain and responding with inappropriate words and flexion withdrawal to painful stimuli. Which value will the nurse report for the patient's Glasgow Coma Scale score? a.5 b.7 c.9 d.11

ANS: C According to the guidelines of the Glasgow Coma Scale, the patient has a score of 9. Opening eyes to pain is 2 points; inappropriate word use is 3 points; and flexion withdrawal is 4 points. The total for this patient is 2 + 3 + 4 = 9.

11. The nurse performs an intervention for a collaborative problem. Which type of intervention did the nurse perform? a.Dependent b.Independent c.Interdependent d.Physician-initiated

ANS: C Collaborative interventions, or interdependent interventions, are therapies that require the combined knowledge, skill, and expertise of multiple health care professionals. Health care provider-initiated (HCP) interventions are dependent nursing interventions, or actions that require an order from the HCP. Nurse-initiated interventions are the independent nursing interventions, or actions that a nurse initiates without supervision or direction from others.

12. A nurse performs an assessment on a patient. Which assessment data will the nurse use as an etiology for Acute pain? a.Discomfort while changing position b.Reports pain as a 7 on a 0 to 10 scale c.Disruption of tissue integrity d.Dull headache

ANS: C Disruption of tissue integrity is a possible cause or etiology of pain. A report of pain, headache, and discomfort are examples of things a patient might say (subjective data or defining characteristics) that may lead a nurse to select Acute pain as a nursing diagnosis.

19. During a school physical examination, the nurse reviews the patient's current medical history. The nurse discovers the patient has allergies. Which assessment finding is consistent with allergies? a.Clubbing b.Yellow discharge c.Pale nasal mucosa d.Puffiness of nasal mucosa

ANS: C Pale nasal mucosa with clear discharge indicates allergy. Clubbing is due to insufficient oxygenation at the periphery resulting from conditions such as chronic emphysema and congenital heart disease; it is noted in the nails. A sinus infection results in yellowish or greenish discharge. Habitual use of intranasal cocaine and opioids causes puffiness and increased vascularity of the nasal mucosa.

7. A school nurse recognizes a belt buckle-shaped ecchymosis on a 7-year-old student. When privately asked about how the injury occurred, the student described falling on the playground. Which action will the nurse take next? a.Talk to the principal about how to proceed. b.Disregard the finding based upon child's response. c.Interview the patient in the presence of the teacher. d.Contact social services and report suspected abuse.

ANS: D Most states mandate a report to a social service center if nurses suspect abuse or neglect. When abuse is suspected, the nurse interviews the patient in private, not with a teacher. Observe the behavior of the individual for any signs of frustration, explanations that do not fit his or her physical presentation, or signs of injury. The nurse knows how to proceed and does not need to talk to the principal about what to do. Disregarding the finding is not advised because victims often will not complain or report that they are in an abusive situation.

3. A nurse develops a nursing diagnostic statement for a patient with a medical diagnosis of pneumonia with chest x-ray results of lower lobe infiltrates. Which nursing diagnosis did the nurse write? a.Ineffective breathing pattern related to pneumonia b.Risk for infection related to chest x-ray procedure c.Risk for deficient fluid volume related to dehydration d.Impaired gas exchange related to alveolar-capillary membrane changes

ANS: D The related to factor of alveolar-capillary membrane changes is accurately written because it is a patient response to the disease process of pneumonia that the nurse can treat. The related to factor should be the cause of the problem (nursing diagnosis) that a nurse can address. The related to factors of dehydration and pneumonia are all medical diagnoses that the nurse cannot change. A diagnostic test or a chronic dysfunction is not an etiology or a condition that a nursing intervention is able to treat.

5. A head and neck physical examination is completed on a 50-year-old female patient. All physical findings are normal except for fine brittle hair. Which laboratory test will the nurse expect to be ordered, based upon the physical findings? a.Oxygen saturation b.Liver function test c.Carbon monoxide d.Thyroid-stimulating hormone test

ANS: D Thyroid disease can make hair thin and brittle. Liver function testing is indicated for a patient who has jaundice. Oxygen saturation will be used for cyanosis. Cherry-colored lips indicate carbon monoxide poisoning.

1. Nurse-initiated interventions are A. Determined by state Nurse Practice Acts. B. Supervised by the entire health care team. C. Made in concert with the plan of care initiated by the physician. D. Developed after interventions for the recent medical diagnoses are evaluated.

Answer: A

2. For a student to avoid a data collection error, the student should A. Assess the patient and, if unsure of the finding, ask a faculty member to assess the patient. B. Review his or her own comfort level and competency with assessment skills. C. Ask another student to perform the assessment. D. Consider whether the diagnosis should be actual, potential, or risk.

Answer: A

1. Concept mapping is one way to A. Connect concepts to a central subject. B. Relate ideas to patient health problems. C. Challenge a nurse's thinking about patient needs and problems. D. Graphically display ideas by organizing data. E. All of the above

Answer: E

Miranda searches the hospital's database for additional information on tuberculosis. True or False: A clinical practice guideline is a collection of institutional policies that assist nurses, physicians, and other health care providers in making decisions about appropriate health care for specific clinical situations, such as the management of tuberculosis.

Answer: False! Rationale: A clinical practice guideline or protocol is a systematically developed set of statements that assist nurses, physicians, and other health care providers in making decisions about appropriate health care for specific clinical situations.

John learns the four types of nursing diagnoses. Which of the following are the four types of nursing diagnoses? (Select all that apply.) Actual diagnoses Risk diagnoses Wellness diagnoses Health promotion diagnoses Disease prevention diagnoses

Answer: The four types of nursing diagnoses are actual diagnoses, risk diagnoses, wellness diagnoses, and health promotion diagnoses.

Cognitive Skills

Application of critical thinking in the nursing process

A client with contact dermatitis is about to undergo skin testing to determine the cause of the skin reaction. The client asks the nurse, "How will the doctor know which allergens to test me for?" The best response by the nurse is that the tests are: A) A routine selection that most individuals react to B) Based on the client's history C) A prepackaged test kit D) The client' decision

B) Based on the client's history The nurse and the doctor each take a detailed history to determine the possible allergens that provoke the reaction and then test for those possibilities. Some common substances may be included in the test, but the history determines most of the allergens used. There is no prepackaged test kit. The client gives information that helps in the decision, but the doctor ultimately decides what to include in the test.

COPD patients should practice A) Breathing in a bag B) Pursed-lip breathing C) Slow Breathing D) Fast breaths

B) Pursed-lip breathing Teach client to take slow, deep breaths and exhale through pursed lips; strengthens muscles of respiration

A nurse is assigned to a new patient admitted to the nursing unit following admission through the emergency department. The nurse collects a nursing history and interviews the patient. Place the following steps for making a nursing diagnosis in the correct order, beginning with the first step: A. Considers context of patient's health problem and selects a related factor B. Reviews assessment data, noting objective and subjective clinical information C. Clusters clinical cues that form a pattern D. Chooses diagnostic label

B) Reviews assessment data, noting objective and subjective clinical information C) Cluster clinical cues that form a pattern D) Chooses diagnostic label A) Considers context of patients health problem and selects a related factor

A nursing student is working with a faculty member to identify a nursing diagnosis for an assigned patient. The student has assessed that the patient is undergoing radiation treatment and has had liquid stool and the skin is clean and intact; therefore she selects the nursing diagnosis Impaired Skin Integrity. The faculty member explains that the student has made a diagnostic error for which of the following reasons? A) Incorrect clustering B) Wrong diagnostic label C) condition is collaborative problem D) premature closure of clusters

B) Wrong diagnostic label!!

An important factor associated with both short-term and long-term weight-loss success is A) Higher initial body mass index. B) Simultaneous smoking cessation. C) A strong desire to improve appearance. D) Fewer dieting attempts in the past year

C) A strong desire to improve appearance. Motivation to lose weight is essential for a favorable and successful outcome.

What is Trousseau's sign

Carpal spasms induced by inflating a BP buff on the arm

Emphysema

Characterized by destruction of the walls of alveoli

Clinical Decision Making

Choosing the options for the best patient outcomes based on patients condition and priority

Nursing Diagnosis

Clinical judgment about individual responses to actual/ potential health problems

Health Promotion Nursing Diagnosis

Clinical judgment of persons motivation to increase well being, nutrition, and exercise

Assessment

Collection of information from primary and secondary sources, interpretation, and validation of data

What precautions need to taken for RSV patients?

Contact Precautions (gown and gloves help protect the spread of RSV) The virus (RSV) that usually causes bronchiolitis can spread to other babies if extra precautions are not taken.

What is Chvostek's sign?

Contraction of facial muscles in response to a tap over the facial nerve of in front of the ear

A nurse interviewed and conducted a physical examination of a patient. Among the assessment data the nurse gathered were an increased respiratory rate, the patient reporting difficulty breathing while lying flat, and pursed-lip breathing. This data set is an example of: A) Collaborative Data Set B) Diagnostic Label C) Related Factors D) Data cluster

D) Data Cluster A data cluster is a set of cues (i.e., the signs or symptoms gathered during assessment).

Concomitant Symptoms

Does patient experience other symptoms w/ primary symptoms. Ex. Nausea with pain

in 1953

Fry proposed the formulation of a nursing diagnosis.

Establishing Priorities

High-emergent Intermediate- non-life-threatening Low-affect patient's future well being Ethical care is a part of priority setting

Psychomotor Skills

Integration of cognitive and motor activities

5 components of critical thinking

Knowledge Base Experience Competency Attitudes (11)- truth seeking, open-minded, disciplined Standards

Critical Thinking Intellectual Standard Steps

Location- Where? Onset & Duration- when? How long? Precipitating Factors- What makes it worse? Quality- Describe symptoms. Severity- 0-10?

What do you do if the goal is not met?

Repeat the entire nursing process for that nursing diagnosis to identify changes.

IADLs

Shopping, prep meals, cleaning, writing checks, taking medications

Review of Systems (ROS)

Systematic approach for collecting patients self reported data on all body systems

When does the order of priorities change?

The order of priorities change when the patient's condition changes.

Wait-to-hip ratio for those with obesity....

Upper body obesity >1 in men > 0.8 in women Lower body obesity <0.8 in women

A

You have finished with several nursing interventions. To evaluate interventions, you need to examine the: A. appropriateness of the interventions and the correct application of the implementation process. B. nursing diagnoses to ensure that they are not medical diagnoses. C. care planning process for errors in other health care team members' judgments. D. interventions of each nurse to enable the nurse manager to correctly evaluate performance.

B

Your patient has met the goals set for improvement of ambulatory status. You would now: A. modify the care plan. B. discontinue the care plan. C. create a new nursing diagnosis that states goals have been met. D. reassess the patient's response to care and evaluate the implementation step of the nursing process.

In evaluating an asthmatic patient's knowledge of self-care, the nurse recognizes that additional instruction is needed when the patient says, a. "I use my corticosteroid inhaler when I feel short of breath." b. "I get a flu shot every year and see my HCP if I have an upper respiratory tract infection." c. "I use my inhaler before I visit my aunt who has a cat, but I only visit for a few minutes because of my allergies." d. "I walk 30 minutes every day but sometimes I have to use my bronchodilator inhaler before walking to prevent me from getting short of breath."

a. "I use my corticosteroid inhaler when I feel short of breath."

What is the role of unlicensed assistive personnel (UAP) in caring for a client with a cast or in traction? Select all that apply. a. Applying ice to the cast b. Positioning the casted extremity above heart level c. Marking the circumference of any drainage on the cast d. Looking for clinical manifestations of compartment syndrome e. Teaching range-of-motion exercises to the client and caregiver

a. Applying ice to the cast b. Positioning the casted extremity above heart level The role of unlicensed assistive personnel (UAP) in caring for the client with a cast or in traction involves applying ice to the cast and positioning the casted extremity above heart level. The licensed practical/vocational nurse (LPN/LVN) marks the circumference of any drainage on the cast. The registered nurse (RN) assesses the client for clinical manifestations of compartment syndrome and teaches the client and caregiver range-of-motion exercises.

Which radiographic test is used to view the entire skeleton? a. Bone scan b. Gallium and thallium scan c. Computed tomography (CT) d. Magnetic resonance imaging (MRI) scan

a. Bone scan

What pain scale is used to measure the intensity of pain in preschoolers? a. FACES scale b. Visual analogue scale c. Numerical rating scale d. Verbal descriptor scale

a. FACES scale

A client visits the clinic because of concerns about insomnia and recent weight loss. A tentative diagnosis of hyperthyroidism is made. What symptom might the nurse identify when assessing this client? a. Fatigue b. Dry skin c. Anorexia d. Bradycardia

a. Fatigue Excessive metabolic activity associated with hyperthyroidism causes fatigue. Warm, moist skin is expected because of increased peripheral perfusion associated with increased metabolism. Increased appetite is expected because of the increased metabolism associated with hyperthyroidism. Tachycardia is expected because of the increased metabolism associated with hyperthyroidism.

Teach the patient with fibromyalgia the importance of limiting intake of which foods (select all that apply)? a. Sugar b. Alcohol c. Caffeine d. Red meat e. Root vegetables

a. Sugar b. Alcohol c. Caffeine

When caring for a patient with acute bronchitis, the nurse will prioritize a. auscultating lung sounds. b. encouraging fluid restriction. c. administering antibiotic therapy. d. teaching the patient to avoid cough suppressants.

a. auscultating lung sounds.

When assessing a patient with a partial-thickness burn, the nurse would expect to find (select all that apply) a. blisters. b. exposed fascia. c. exposed muscles. d. intact nerve endings. e. red, shiny, wet appearance.

a. blisters. d. intact nerve endings. e. red, shiny, wet appearance.

The most common early clinical manifestations of ARDS that the nurse may observe are a. dyspnea and tachypnea. b. cyanosis and apprehension. c. hypotension and tachycardia. d. respiratory distress and frothy sputum.

a. dyspnea and tachypnea.

The lungs act as an acid-base buffer by a. increasing respiratory rate and depth when CO2 levels in the blood are high, reducing acid load. b. increasing respiratory rate and depth when CO2 levels in the blood are low, reducing base load. c. decreasing respiratory rate and depth when CO2 levels in the blood are high, reducing acid load. d. decreasing respiratory rate and depth when CO2 levels in the blood are low, increasing acid load.

a. increasing respiratory rate and depth when CO2 levels in the blood are high, reducing acid load.

Clinical criterion

an objective or subjective sign, symptom, or risk factor that when analyzed with other criteria leads to a diagnostic conclusion

cognitive skills

application of critical thinking in the nursing process

A client is admitted to the mental health unit with the diagnosis of major depressive disorder. Which statement alerts the nurse to the possibility of a suicide attempt? a. "I don't feel too good today." b. "I feel much better; today is a lovely day." c. "I feel a little better, but it probably won't last." d. "I'm really tired today, so I'll take things a little slower."

b. "I feel much better; today is a lovely day." A rapid mood upswing and psychomotor change may signal that the client has made a decision and has developed a plan for suicide.

The home health nurse visits a 40-year-old patient with metastatic breast cancer who is receiving palliative care. The patient is experiencing pain at a level of 7 (0-10 point scale). In prioritizing activities for the visit, what should the nurse do first? a. Auscultate for breath sounds. b. Administer PRN pain medication. c. Check pressure points for skin breakdown. d. Ask family about patient's food and fluid intake.

b. Administer PRN pain medication.

A 40-year-old client with a terminal illness wishes to die at home in the presence of his or her spouse and child. Who will be the primary concern of the nurse viewing this family as context? a. Child b. Client c. Spouse d. Whole family

b. Client When the family is viewed as context, the primary focus is the client and fulfilling the basic needs of the client. The nurse may focus on the child and spouse when the family is viewed as client. The whole family will be considered when the nurse sees the family as a system in which the client and each family member are considered. Needs will be fulfilled with the use of all available environmental, social, psychologic, and community resources.

A client with type 1 diabetes complains of hunger, thirst, tiredness, and frequent urination. Based on these findings, the nurse should take what action? a. Notify the physician immediately about the client's symptoms. b. Determine the client's blood glucose level. c. Administer the client's prescribed insulin. d. Give the client a peanut butter and graham cracker snack.

b. Determine the client's blood glucose level.

The nurse is providing home care to an older adult client with decreased bone density. Which nursing intervention will be most beneficial for the client? a. Teaching isometric exercises b. Encouraging the client to do weight-bearing exercises c. Instructing the client to sit in supportive chairs with arms d. Providing moist heat such as shower or moist compresses

b. Encouraging the client to do weight-bearing exercises Older adults are at risk of developing decreased bone density. Elderly clients with decreased bone density should be encouraged to do weight-bearing exercises. Teaching isometric exercises would be beneficial for a client with muscular atrophy. A client with kyphotic posture should be instructed to sit in supportive chairs with arms. Providing moist heat would be beneficial for a client with cartilage degeneration.

A client with chronic obstructive pulmonary disease (COPD) states, "I have had steady weight loss, and I am often too tired to eat." Which nursing diagnosis would be most appropriate for this client? a. Fatigue related to weight loss secondary to COPD b. Imbalanced nutrition: less than body requirements, related to fatigue c. Imbalanced nutrition: less than body requirements, related to COPD d. Ineffective breathing pattern, related to alveolar hypoventilation

b. Imbalanced nutrition: less than body requirements, related to fatigue The response portion of the nursing diagnosis is Imbalanced nutrition: less than body requirements, and the etiology is fatigue associated with the disease process of COPD. Interventions should be planned to deal with the breathing problem and the fatigue associated with it while implementing actions to combat the weight loss. Weight loss related to COPD is not a NANDA-approved nursing diagnosis. Fatigue associated with the COPD disease process is the cause of the weight loss, not COPD in itself. Altered breathing pattern is also a problem, but does not specifically relate to the weight loss problem.

After an above-the-knee amputation of a right leg, a client reports pain in the right foot. The nurse should inform the client that phantom limb pain is the result of what? a. Tactile illusions associated with severed blood vessels b. Nerve endings in the limb that are still intact and react to stimuli c. An unconscious phenomenon to aid with grieving over the lost body part d. Hallucinations secondary to emotional symptoms associated with the distress of amputation

b. Nerve endings in the limb that are still intact and react to stimuli

When obtaining a health history from a client recently diagnosed with type 1 diabetes, the nurse expects the client to report what clinical manifestations? a. Irritability, polydipsia, and polyuria b. Polyuria, polydipsia, and polyphagia c. Nocturia, weight loss, and polydipsia d. Polyphagia, polyuria, and diaphoresis

b. Polyuria, polydipsia, and polyphagia

A nurse is caring for a patient who has a pressure ulcer that is treated with debridement, irrigations, and moist gauze dressings. How should the nurse anticipate healing to occur? a. Tertiary intention b. Secondary intention c. Regeneration of cells d. Remodeling of tissues

b. Secondary intention

In caring for a patient after a spinal fusion, the nurse would immediately report which of the following to the surgeon? a. The patient experiences a single episode of emesis. b. The patient is unable to move the lower extremities. c. The patient is nauseated and has not voided in 4 hours. d. The patient complains of pain at the bone graft donor site.

b. The patient is unable to move the lower extremities.

Pain is best described as a. a creation of a person's imagination. b. an unpleasant, subjective experience. c. a maladaptive response to a stimulus. d. a neurologic event resulting from activation of nociceptors.

b. an unpleasant, subjective experience.

An example of distraction to provide pain relief is a. TENS. b. music. c. exercise. d. biofeedback.

b. music.

A client is admitted to the intensive care unit with a diagnosis of acute respiratory distress syndrome. When assessing the client, what does the nurse expect to identify? a. Hypertension b. Tenacious sputum c. Altered mental status d. Slow rate of breathing

c. Altered mental status Altered mental status is secondary to cerebral hypoxia, which accompanies acute respiratory distress syndrome (ARDS); cognition and level of consciousness are reduced. Hypotension occurs because of hypoxia. The sputum is not tenacious, but it may be frothy if pulmonary edema is present. Breathing will be fast and shallow.

A weak, dyspneic, terminally ill client is visited frequently by the spouse and teenage children. What should the client's plan of care include? a. Foster self-activity whenever possible. b. Plan care to be completed at one time followed by a long rest. c. Teach family members how to assist with the client's basic care. d. Limit visiting to evening hours before the client goes to sleep.

c. Teach family members how to assist with the client's basic care.

An important nursing responsibility related to pain is to a. leave the patient alone to rest. b. help the patient appear to not be in pain. c. believe what the patient says about the pain. d. assume responsibility for eliminating the patient's pain.

c. believe what the patient says about the pain.

A client with a fractured head of the right femur and osteoporosis is placed in Buck extension before surgical repair. What should the nurse do when caring for this client until surgery is performed? a. Remove the weights from the traction every 2 hours to promote comfort. b. Turn the client from side to side every 2 hours to prevent pressure on the coccyx. c. Raise the knee gatch on the bed every 2 hours to limit the shearing force of traction. d. Assess the circulation of the affected leg every 2 hours to ensure adequate tissue perfusion.

d. Assess the circulation of the affected leg every 2 hours to ensure adequate tissue perfusion.

A nurse identifies the establishment of trust as a major nursing goal for a depressed client. How can this goal best be accomplished? a. By spending a day with the client b. By asking the client at least one question daily c. By waiting for the client to initiate the conversation d. By visiting frequently for short periods with the client each day

d. By visiting frequently for short periods with the client each day Frequent short visits with the client each day demonstrate to the client that the nurse feels that the client is worth spending time with and helps restore and build trust.

While assessing an immobilized client, the nurse notes that the client has shortened muscles over a joint, preventing full extension. What is this condition known as? a. Osteoarthritis b. Osteoporosis c. Muscle atrophy d. Contracture

d. Contracture Immobilized clients are at high risk for the development of contractures. Contractures are characterized by permanent shortening of the muscle covering a joint. Osteoarthritis is a disease process of the weight-bearing joints caused by wear and tear. Osteoporosis is a metabolic disease process in which the bones lose calcium. Muscle atrophy is a wasting and/or decrease in the strength and size of muscles because of a lack of physical activity or a neurologic or musculoskeletal disorder.

A client's blood gases reflect diabetic ketoacidosis. Which clinical indicator should the nurse identify when monitoring this client's laboratory values? a. Increased pH b. Decreased PO 2 c. Increased PCO 2 d. Decreased HCO 3

d. Decreased HCO 3 The bicarbonate-carbonic acid buffer system helps maintain the pH of body fluids; in metabolic acidosis, there is a decrease in bicarbonate because of an increase of metabolic acids. The pH is decreased. The PO 2 is not decreased in diabetic acidosis. The PCO 2 may be decreased by the body's attempt to eliminate CO 2 to compensate for a decreased pH.

A nurse is reviewing the laboratory report of a 13-year-old adolescent with type 1 diabetes. What test is considered the most accurate in the evaluation of the effectiveness of diet and insulin therapy over time? a. Blood pH b. Serum protein level c. Serum glucose level d. Glycosylated hemoglobin

d. Glycosylated hemoglobin

A mental health crisis occurs as a result of what stress-related factor? a. The stress is chronic and maturational in nature. b. The stress is perceived rather than real in nature. c. The stress is extremely severe and situational in its origin. d. The stress is not managed by the individual's usual methods.

d. The stress is not managed by the individual's usual methods. An individual experiences a crisis when stress, either real or imagined, cannot be controlled by the person's usual coping mechanisms. It would not be considered a crisis if it was chronic and maturational, severe and situational, or perceived rather than real.

After thyroid surgery, the nurse suspects damage or removal of the parathyroid glands when the patient develops a. muscle weakness and weight loss. b. hyperthermia and severe tachycardia. c. hypertension and difficulty swallowing. d. laryngospasms and tingling in the hands and feet.

d. laryngospasms and tingling in the hands and feet.

A patient has the following arterial blood gas results: pH 7.52, PaCO2 30 mm Hg, HCO3− 24 mEq/L. The nurse determines that these results indicate a. metabolic acidosis. b. metabolic alkalosis. c. respiratory acidosis. d. respiratory alkalosis.

d. respiratory alkalosis.

Priority setting begins at a ___________ level when you identify and prioritize a patient's main diagnoses or problems.

holistic

instrumental activities of daily living (IADLs)

include the day-to-day activities a person performs such as shopping, preparing meals, writing checks to pay the bills, and taking medications

counseling

involves providing emotional, intellectual, spiritual, and psychological support to your patients

Graves Disease

is a form of hyperthyroidism o Mimics hyperthyroidism Abnormal antibodies that mimic TSH Work overtime and exceed their normal quota

Clinical Manifestations of Emphysema

o Air trapping o Possible wheezing o Dyspnea o Barrel chest o Pursed lip breathing o Posturing

Best position for ARDS patient?

prone (face-down)

physical care techniques

the activities that nurses perform while rendering care. These including turning, positioning, and administering care, as well as performing tasks such as Foley catheter insertion, NG tube insertion, IV insertion, and administering medications.

lifesaving measures

those activities you perform when a patient's physiological or psychological state is threatened. They include CPR, administering emergency medications, and falls prevention.

55. The visitor on a medical unit is shouting and making threats about harming the staff because of perceived poor care his loved one has received. Which statement is the nurse's best initial response? 1. "If you don't stop shouting, I will have to call security." 2. "I hear that you are frustrated. Can we discuss the issues calmly?" 3. "Sir, you are disrupting the unit. Calm down or leave the hospital." 4. "This type of behavior is uncalled for and will not resolve anything."

2. "I hear that you are frustrated. Can we discuss the issues calmly?"

74. A terrible storm causes the electricity to go out in the hospital and the emergency generator lights come on. Which action should the charge nurse implement? 1. Request all family members to leave the hospital as soon as possible. 2. Instruct the staff to plug critical electrical equipment into the red outlets. 3. Have the unlicensed assistive personnel (UAP) place a portable flashlight on each bedside table. 4. Contact the maintenance department to determine how long the electricity will be out.

2. Instruct the staff to plug critical electrical equipment into the red outlets.

60. The nurse has been named in a lawsuit concerning the care provided. Which action should the nurse take first? 1. Consult with the hospital's attorney. 2. Review the client's chart. 3. Purchase personal liability insurance. 4. Discuss the case with the supervisor.

2. Review the client's chart.

41. The hospital will be implementing a new medication administration record (MAR) for documenting medication administration. Which action should the clinical manager take first when implementing the new MAR? 1. Discuss the new MAR with each nurse individually. 2. Schedule meetings on all shifts to discuss the new MAR. 3. Require the nurse to read a handout explaining the new MAR. 4. Ask the nurses to watch a video explaining the new MAR.

2. Schedule meetings on all shifts to discuss the new MAR.

4. The primary nurse informs the shift manager one of the unlicensed assistive personnel (UAPs) is falsifying vital signs. Which action should the shift manager implement first? 1. Notify the unit manager of the potential situation of falsifying vital signs. 2. Take the assigned client's vital signs and compare with the UAP's results. 3. Talk to the UAP about the primary nurse's allegation. 4. Complete a counseling record and place in the UAP's file.

2. Take the assigned client's vital signs and compare with the UAP's results.

16. The client on a medical unit died of a communicable disease. Which information should the nurse provide to the mortuary workers? 1. No information can be released to the mortuary service. 2. The nurse should tell the funeral home the client's diagnosis. 3. Ask the family for permission to talk with the mortician. 4. Refer the funeral home to the HCP for information.

2. The nurse should tell the funeral home the client's diagnosis.

Best education for new parents to avoid SIDS?

"Back to sleep, tummy to play"

What factors does implementation require?

- additional knowledge -nursing skills -personnel resources

Three parts to nursing diagnostic label

-Problem -Etiology -Symptoms

99. The resident in a long-term care facility tells the nurse, "I think my family just put me here to die because they think I am too much trouble." Which statement is the nurse's best response? 1. "Can you tell me more about how you feel since your family placed you here?" 2. "Your family did what they felt was best for your safety." 3. "Why would you think that about your family? They care for you." 4. "Tell me, how much trouble were you when you were at home?"

1. "Can you tell me more about how you feel since your family placed you here?"

51. The nurse is preparing to perform a sterile dressing change on a client with full-thickness burns on the right leg. Which intervention should the nurse implement first? 1. Pre-medicate the client with a narcotic analgesic. 2. Prepare the equipment and bandages at the bedside. 3. Remove the old dressing with non-sterile gloves. 4. Place a sterile glove on the dominant hand.

1. Pre-medicate the client with a narcotic analgesic.

Decision Making

A product of critical thinking that focuses on problem resolution

Independent- Nursing Interventions

Actions a nurse initiates

Role of patient in Goal/Outcome Setting

Always partner with patients when setting their individualized goals. Mutual goal setting includes the patient and family (when appropriate) in prioritizing the goals of care and developing a plan of action. Act as a patient advocate

For a student to avoid a data collection error, the student should:

Assess the patient and, if unsure of the finding, ask a faculty member to assess the patient

Evaluative Measures

Assessment skills and techniques. ex. observation, interviewing, etc.

You know that _____ diagnosis is applied to vulnerable populations.

At risk

What are some examples of independent nursing interventions that the nurse can implement with DJ? (Select all that apply) a. Medication administration b. Medication teaching c. Patient positioning d. Family teaching

B, C, D

Defining Characteristics

Clinical criteria that are observable and verifiable

You know that _____ interventions require an order from a physician or another health care professionals.

Dependent

Health Care provider initiated

Dependent-require an order from a physician or other health care professional

Actual Nursing Diagnoses

Describes human responses to health conditions or life process that exist

Interpersonal Skills

Develop trust relationship, express caring attitude, communicate clearly

What is the purpose of the nursing process?

Diagnose and treat human responses to actual and potential problems

Etiology

Factor of nursing diagnosis is always w/in domain of nursing practice and a condition that responses to nursing intervention

Hypoxemia & Hypercapnia in COPD

Gas exchange abnormalities result in hypoxemia and hypercapnia (increased CO2) as the disease worsens. As the air trapping increases, walls of alveoli are destroyed, and bullae (large air spaces in the parenchyma) and blebs (air spaces adjacent to pleurae) can form. Bullae and blebs are not effective in gas exchange, since they do not contain the capillary bed that normally surrounds each alveolus. Therefore a significant ventilation-perfusion (V/Q) mismatch and hypoxemia result.

Medical Diagnosis

Identification of a disease condition based on evaluation of physical signs, symptoms, and history, and diagnostic test results

Nursing Care Plan

Includes nursing diagnosis, goals, and/or outcomes, interventions, and evaluation

Nurse initiated Intervention

Independent—Actions that a nurse initiates

Cue

Information obtained through use of the senses

Steps of a physical exam?

Inspect Palpate Percussion Auscultation Smell

Steps of developing a nursing diagnosis? (5)

Interpret/ analyze data Data clustering Look for related factors Identify patient needs Formulate nursing diagnoses

Evidence- based Knowledge

Knowledge based on research or clinical expertise

3 levels of critical thinking (Kataoka- Yahika & Saylor Model)

Level 1: Basic CT * Learners trust experts have right answers * Answers are either right or wrong Level 2: Complex CT * Begin to separate from expert *Multiple solutions exist Level 3: Commitment * Makes decisions w/o help * Assumes accountability of the decision

Explain expected outcome.

Measurable change that must be achieved to reach a goal. Many times, several must be met to meet a single goal.

Expected Outcome

Measurable criterion to evaluate goal achievement

Standard of Care

Minimum level of care accepted to ensure high quality of care to patients

What are nursing interventions?

Nursing interventions are treatments or actions based on clinical judgment and knowledge that nurses perform to enhance patient outcomes.

Clinical Criterion

Objective/ subjective signs & symptoms that when analyzed lead to diagnostic conclusions

Objective Data

Observations of a patients health status

Problem Solving

Obtaining information to find a solution

Why are children at risk for respiratory alkalosis?

Older adults and young children at risk with large-dose salicylate ingestion

You evaluate DJ's response to the medication therapy to update his care plan. The nurse assesses Mr. Mashoud's pain before NSAID administration and then approximately one hour after administration. Miyoko knows that evaluation is an _____ process that occurs whenever contact with a patient occurs.

Ongoing

Critical Pathway

Patient care management plans that provide multiple healthcare teams with activities to be put into practice overtime.

When a nurse assists a postoperative client to the chair, which type of nursing intervention does this represent? Maintenance Surveillance Psychomotor Psychosocial

Psychomotor Explanation: Psychomotor interventions include activities such as positioning, inserting, and applying. A psychosocial intervention focuses on supporting, exploring, and encouraging. Maintenance and surveillance are monitoring interventions.

78. The nurse educator is discussing fire safety with new employees. List in order of performance the following actions the nurse should teach to ensure the safety of clients and employees in the case of fire on the unit. 1. Extinguish. 2. Rescue. 3. Confine. 4. Alert.

RACE

Dependent- Nursing interventions

Require order from physician

A nurse identified that a patient has difficulty turning in bed, moves slowly when assisted into a chair, and expresses having breathlessness after walking to the bathroom and back. The patient has been in the hospital for over 4 days. Write a three-part nursing diagnostic statement using the PES format.

See Evolve.

Data Cluster

Set of signs and symptoms gathered during assessment grouped in logical way.

Clinical Practice Guidelines

Set of statements that help nurses and healthcare providers make decisions about appropriate healthcare for specific situations

Planning

Setting priorities, goals, and outcomes

In order to make an inference you must have?

Supporting cues.

Scientific Rationale

The reason that you chose a specific nursing action, based on supporting evidence

Direct Care

Treatments preformed through interaction with patients

Scientific Method

Way to solve problems using reasoning

collaborative problem

actual or potential physiological complication that nurses monitor to detect a change in patient status

Ascites and encephalopathy are signs of?

cirrhosis

What is Asterixis?

flapping hand tremor (sign of encephalopathy)

psychomotor skills

integration of cognitive and motor activities

in 1950

nursing diagnosis was first introduced

When writing goals and expected outcomes, they must be...

patient-centered

What is a patient-centered goal?

reflects a pt's specific behavior

Data cluster

set of cues (the signs or symptoms gathered during assessment)

in 1973

the first national conference was held.

The expected outcome for a client with a new diagnosis of rheumatoid arthritis (RA) is: "Client will describe appropriate actions when implementing the prescribed medication routine." Which statement by the client indicates the outcome expectation has been met? "I will take the medications until the inflammation goes away." "I will take my medications on an empty stomach for maximum effect." "I should increase water intake if I have dark bowel movements." "I should call my health care provider if I have a sore that won't heal."

"I should call my health care provider if I have a sore that won't heal." Explanation: The primary purpose of a client outcome in a plan of care is to evaluate the successful prevention, reduction, or resolution of client health problems and the attainment of the client's health expectations. A client learning about a new medication routine must learn appropriate actions of administration, storage, and conditions that require contact with the health care provider.

The expected outcome for a client with a new diagnosis of diabetes mellitus is: "Client will describe appropriate actions when implementing the prescribed medication routine." Which statement by the client indicates the outcome expectation has been met? "I will take insulin until my blood sugar levels are normal." "I will take my medications between meals for maximum effect." "I will mix insulin glargine with insulin lispro at bedtime." "I will test my glucose level before meals and use sliding scale insulin."

"I will test my glucose level before meals and use sliding scale insulin." Explanation: The primary purpose of a client outcome in a plan of care is to evaluate the successful prevention, reduction, or resolution of client health problems and the attainment of the client's health expectations. A client learning about a new medication routine must learn appropriate actions of administration and storage and conditions that require contact with the health care provider. Diabetes mellitus is a chronic disease, so the client who takes insulin should not expect to ever achieve a normal blood glucose level without taking insulin. The client should test blood glucose level before, not between, meals. Mixing different types of insulin is not necessary.

The nurse asks if the client with a new diagnosis of lung cancer would like medication to help treat nicotine withdrawal symptoms. The client refuses by saying, "I have smoked since I was 12 years old. I am not going to stop now." What is the appropriate response by the nurse? "You need to stop smoking for us to effectively combat this disease." "Please tell me your thoughts about treating this diagnosis." "Do you want to be discharged without treatment?" "What are your plans after discharge?"

"Please tell me your thoughts about treating this diagnosis." Explanation: In the planning stage of the nursing process, the nurse must focus on the client's interests and preferences, keep an open mind, and include interventions that are supported by research. While the nurse knows that research shows smoking cessation is valuable in successful treatment of lung cancer, the client's choices must be included in the plan for it to be successful. Asking about plans after discharge is too broad and may not elicit the information the nurse needs to design the best plan of care.

Assessment

*collecting information from pt & secondary sources *forming a complete database

Clinical decision making involves:

*judgement *critical & reflective thinking & action *application of scientific & practical logic

Critical thinking model is a combination of:

*knowledge base *experience *competence *attitudes *standards

Define critical thinking

*solving problems *making decisions *rather than carelessly forming quick solutions

What do nursing interventions consist of?

- actions -frequency -quantity -method -person who will perform the above

competencies

- being systematic - using criterion based evaluation - collaborating - using ongoing assessment data to revise health care plan - communicating skills

Diagnostic Label:

- name of nursing diagnosis as approved by NANDA-I - describes essence of patient's response to health conditions in as few words as possible - definition describes the characteristics of the human response identified and helps to select correct diagnosis

What are you responsible for when delegating aspects of care?

- responsible for ensuring each task is assigned appropriately & completed

What is priority setting?

- setting in order of urgency & importance

What is involved in a patient-centered interview?

- setting the stage -gathering information about pt problems -setting an agenda -collecting assessment/history -terminating the interview

Why is it so critical to review plan of care before implementing?

- to ensure that no nursing action is automatic

Questions to consider using when making a culturally competent nursing diagnosis

-How has this problem affected you and your family? -What do you believe will help or fix the problem? -What worries you the most about this problem? -What do you expect from us (nurses) to maintain your values/practices to stay healthy? -Which cultural practices do you observe?

Guidelines to reduce errors

-Identify the patient's response -Identify a NANDA-I diagnostic statement, not the symptom -Identify a treatable cause or risk factor -Identify the problem caused by the treatment or diagnostic study -Identify the patient response to the equipment -Identify the patient's problems -Identify the patient problem, not nursing intervention -Identify patient problem, not the goal -Make professional judgments -Avoid legally inadvisable statements -Identify the problem and its cause -Identify only one patient problem in the diagnostic statement

what does the PES format stand for

-Problem -Etiology -symptoms (or defining characteristics)

What does NIC taxonomy provide?

-Standardiztion -Help select suitable interventions for pt's problems

nursing intervention

-any treatment based on clinical judgment and knowledge that a nurse performs to enhance patient outcomes. -include direct and indirect care measures aimed at individuals, families, and/or the community

What is the purpose of goals & outcomes?

-designed to focus efforts of team members on common purpose

teaching

-is a constant part of nursing -occurs formally and informally and involves patients and their family members

40. The nurse and licensed practical nurse (LPN) have been assigned to care for clients on a pediatric unit. Which nursing task should be assigned to the LPN? 1. Administer PO medications to a client diagnosed with gastroenteritis. 2. Take the routine vital signs for all the clients on the pediatric unit. 3. Transcribe the HCP's orders into the computer. 4. Assess the urinary output of a client diagnosed with nephrotic syndrome.

1. Administer PO medications to a client diagnosed with gastroenteritis.

27. The wound care nurse in a long-term care facility asks the unlicensed assistive personnel (UAP) for assistance. Which task should not be delegated to the UAP? 1. Apply the wound debriding paste to the wound. 2. Keep the resident's heels off the surface of the bed. 3. Turn the resident at least every 2 hours. 4. Encourage the resident to drink a high-protein shake.

1. Apply the wound debriding paste to the wound.

117. The nurse manager of the maternal-child department is developing the budget for the next fiscal year. Which statement best explains the first step of the budgetary process? 1. Ask the staff for input about needed equipment. 2. Assess any new department project for costs. 3. Review the department's current year budget. 4. Explain the new budget requirements to the staff.

1. Ask the staff for input about needed equipment.

5. The nurse hung the wrong intravenous antibiotic for the postoperative client. Which intervention should the nurse implement first? 1. Assess the client for any adverse reactions. 2. Complete the incident or adverse occurrence report. 3. Administer the correct intravenous antibiotic medication. 4. Notify the client's healthcare provider.

1. Assess the client for any adverse reactions.

Steps in nursing process

1. Assessment 2. Diagnosis 3. Planning 4. Implementation 5. Evaluation

Steps of the Nursing Process: (5)

1. Assessment 2. Nursing Diagnosis 3. Planning 4. Implementation 5. Evaluation

57. The nurse is preparing to administer the client's first intravenous antibiotic. Prioritize the nurse's actions from first (1) to last (5). 1. Check the healthcare provider's order in the chart. 2. Determine if the client has any known allergies. 3. Hang the secondary IV piggyback higher than the primary IV. 4. Set the intravenous pump at the correct rate. 5. Determine if the antibiotic is compatible with the primary IV.

1. Check the healthcare provider's order in the chart. 5. Determine if the antibiotic is compatible with the primary IV. 2. Determine if the client has any known allergies. 3. Hang the secondary IV piggyback higher than the primary IV. 4. Set the intravenous pump at the correct rate.

A nurse interviewed and conducted a physical examination of a patient. Among the assessment data the nurse gathered were an increased respiratory rate, the patient reporting difficulty breathing while lying flat, and pursed-lip breathing. This data set is an example of:

1. Collaborative data set. 2. Diagnostic label. 3. Related factors. 4. Data cluster. 4

A nurse is assigned to a new patient admitted to the nursing unit following admission through the emergency department. The nurse collects a nursing history and interviews the patient. Place the following steps for making a nursing diagnosis in the correct order, beginning with the first step.

1. Considers context of patient's health problem and selects a related factor 2. Reviews assessment data, noting objective and subjective clinical information 3. Clusters clinical cues that form a pattern 4. Chooses diagnostic label 2, 3, 4, and 1

A nurse reviews data gathered regarding a patient's ability to cope with loss. The nurse compares the defining characteristics for Ineffective Coping with those for Readiness for Enhanced Coping and selects Ineffective Coping as the correct diagnosis. This is an example of the nurse avoiding an error in: (Select all that apply.)

1. Data collection. 2. Data clustering. 3. Data interpretation. 4. Making a diagnostic statement. 5. Goal setting. 1 and 3

The use of standard formal nursing diagnostic statements serves several purposes in nursing practice, including which of the following? (Select all that apply.)

1. Defines a patient's problem, giving members of the health care team a common language for understanding the patient's needs 2. Allows physicians and allied health staff to communicate with nurses how they provide care among themselves 239 3. Helps nurses focus on the scope of nursing practice 4. Creates practice guidelines for collaborative health care activities 5. Builds and expands nursing knowledge 1, 3, and 5

29. The clinic nurse is caring for a client diagnosed with osteoarthritis. The client tells the nurse, "I am having problems getting in and out of my bathtub." Which intervention should the clinic nurse implement first? 1. Determine whether the client has grab bars in the bathroom. 2. Encourage the client to take a shower instead of a bath. 3. Initiate a referral to a physical therapist for the client. 4. Discuss whether the client takes nonsteroidal anti-inflammatory drugs (NSAIDs).

1. Determine whether the client has grab bars in the bathroom.

62. The charge nurse notices that one of the staff takes frequent breaks, has unpredictable mood swings, and often volunteers to care for clients who require narcotics. Which priority action should the charge nurse implement regarding this employee? 1. Discuss the nurse's actions with the unit manager. 2. Confront the nurse about the behavior. 3. Do not allow the nurse to take breaks alone. 4. Prepare an occurrence report on the employee.

1. Discuss the nurse's actions with the unit manager.

A nurse assesses a young woman who works part time but also cares for her mother at home. The nurse reviews clusters of data that include the patient's report of frequent awakenings at night, reduced ability to think clearly at work, and a sense of not feeling well rested. Which of the following diagnoses is in the correct PES format?

1. Disturbed Sleep Pattern evidenced by frequent awakening 2. Disturbed Sleep Pattern related to family caregiving responsibilities 3. Disturbed Sleep Pattern related to need to improve sleep habits 4. Disturbed Sleep Pattern related to caregiving responsibilities as evidenced by frequent awakening and not feeling rested 4

115. The 36-year-old client in the women's health clinic is being prescribed birth control pills. Which information is important for the nurse to teach the client? Select all that apply. 1. Do not smoke while taking birth control pills. 2. Take one pill at the same time every day. 3. If a birth control pill is missed, do not double up. 4. Stop taking the pill if breakthrough bleeding occurs. 5. There can be interactions with other medications.

1. Do not smoke while taking birth control pills. 2. Take one pill at the same time every day. 5. There can be interactions with other medications.

102. The charge nurse in an extended care facility notes an elderly male resident holding hands with an elderly female resident. Which intervention should the charge nurse implement? 1. Do nothing, because this is a natural human need. 2. Notify the family of the residents about the situation. 3. Separate the residents for all activities. 4. Call a care plan meeting with other staff members.

1. Do nothing, because this is a natural human need.

61. The nurse has accepted the position of clinical manager for a medical-surgical unit. Which role is an important aspect of this management position? 1. Evaluate the job performance of the staff. 2. Be the sole decision maker for the unit. 3. Take responsibility for the staff nurse's actions. 4. Attend the medical staff meetings.

1. Evaluate the job performance of the staff.

Which of the following nursing diagnoses is stated correctly? (Select all that apply.)

1. Fluid Volume Excess related to heart failure 2. Sleep Deprivation related to sustained noisy environment 3. Impaired Bed Mobility related to postcardiac catheterization 4. Ineffective Protection related to inadequate nutrition 5. Diarrhea related to frequent, small, watery stools. 2 and 4

Steps to Discontinuing Care Plan

1. Has the goal been met? 2. Does the patient agree? 3. Document and discontinue plan.

A nurse is getting ready to assess a patient in a neighborhood community clinic. He was newly diagnosed with diabetes just a month ago. He has other health problems and a history of not being able to manage his health. Which of the following questions reflects the nurse's cultural competence in making an accurate diagnosis? (Select all that apply.)

1. How is your diabetic diet affecting you and your family? 2. You seem to not want to follow health guidelines. Can you explain why? 3. What worries you the most about having diabetes? 4. What do you expect from us when you do not take your insulin as instructed? 5. What do you believe will help you control your blood sugar? 1, 3, and 5

Scientific Method Steps: (5)

1. Identify problem 2. Collecting Data 3. Formulation a question or hypothesis 4. Testing the question or hypothesis 5. Evaluating results of the test or study

A nurse is reviewing a patient's list of nursing diagnoses in the medical record. The most recent nursing diagnosis is Diarrhea related to intestinal colitis. For which of the following reasons is this an incorrectly stated diagnostic statement?

1. Identifying the clinical sign instead of an etiology 2. Identifying a diagnosis on the basis of prejudicial judgment 3. Identifying the diagnostic study rather than a problem caused by the diagnostic study 4. Identifying the medical diagnosis instead of the patient's response to the diagnosis. 4

Review the following problem-focused nursing diagnoses and identify the diagnoses that are stated correctly. (Select all that apply.)

1. Impaired Skin Integrity related to physical immobility 2. Fatigue related to heart disease 3. Nausea related to gastric distention 4. Need for improved Oral Mucosa Integrity related to inflamed mucosa 5. Risk for Infection related to surgery 1 and 3

A nursing student is working with a faculty member to identify a nursing diagnosis for an assigned patient. The student has assessed that the patient is undergoing radiation treatment and has had liquid stool and the skin is clean and intact; therefore she selects the nursing diagnosis Impaired Skin Integrity. The faculty member explains that the student has made a diagnostic error for which of the following reasons?

1. Incorrect clustering 2. Wrong diagnostic label 3. Condition is a collaborative problem. 4. Premature closure of clusters 2

A nurse in a mother-baby clinic learns that a 16-year-old has given birth to her first child and has not been to a well-baby class yet. The nurse's assessment reveals that the infant cries when breastfeeding and has difficulty latching on to the nipple. The infant has not gained weight over the last 2 weeks. The nurse identifies the patient's nursing diagnosis as Ineffective Breastfeeding. Which of the following is the best "related to" factor?

1. Infant crying at breast 2. Infant unable to latch on to breast correctly 3. Mother's deficient knowledge 4. Lack of infant weight gain 3

14. The home health (HH) nurse is discussing the care of a client with the female HH aide. Which task should the HH nurse delegate to the HH aide? 1. Instruct her to assist the client with a shower. 2. Ask her to prepare the breakfast meal for the client. 3. Request her to take the client to an HCP's appointment. 4. Tell her to show the client how to use a glucometer.

1. Instruct her to assist the client with a shower.

A nursing student reports to a lead charge nurse that his assigned patient seems to be less alert and his blood pressure is lower, dropping from 140/80 to 110/60. The nursing student states, "I believe this is a nursing diagnosis of Deficient Fluid Volume." The lead charge nurse immediately goes to the patient's room with the student to assess the patient's orientation, heart rate, skin turgor, and urine output for last 8 hours. The lead charge nurse suspects that the student has made which type of diagnostic error?

1. Insufficient cluster of cues 2. Disorganization 3. Insufficient number of cues 4. Evidence that another diagnosis is more likely 3

46. The unconscious 4-year-old child with bruises covering the torso in varying stages of healing is brought to the emergency department by paramedics. The nurse notes small burn marks on the child's genitalia. Which actions should the nurse implement? Select all that apply. 1. Notify Child Protective Services. 2. Ask the parent how the child was injured. 3. Perform a thorough examination for more injuries. 4. Tell the parents that the police have been called. 5. Prepare the child for skull x-rays and a CT scan.

1. Notify Child Protective Services. 3. Perform a thorough examination for more injuries. 5. Prepare the child for skull x-rays and a CT scan.

11. The clinic manager is discussing osteoporosis with the clinic staff. Which activity is an example of a secondary nursing intervention when discussing osteoporosis? 1. Obtain a bone density evaluation test on a female client older than 50. 2. Perform a spinal screening examination on all female clients. 3. Encourage the client to walk 30 minutes daily on a hard surface. 4. Discuss risk factors for developing osteoporosis.

1. Obtain a bone density evaluation test on a female client older than 50.

95. The charge nurse overhears two unlicensed assistive personnel (UAPs) discussing a client in the hallway. Which action should the charge nurse implement first? 1. Remind the UAPs that clients should not be discussed in a public area. 2. Tell the unit manager that the UAPs might have been overheard. 3. Have the UAPs review policies on client confidentiality and HIPAA. 4. Find some nursing tasks the UAPs can be performing at this time.

1. Remind the UAPs that clients should not be discussed in a public area.

81. The client is confused and pulling at the IV and indwelling catheter. Which order from the HCP should the nurse clarify concerning restraining the client? 1. Restrain the client's wrists, as needed. 2. Offer the client fluids every 2 hours. 3. Apply a hand mitt to the arm opposite the IV site for 12 hours. 4. Check circulation of the restrained limb every 2 hours.

1. Restrain the client's wrists, as needed.

The nursing diagnosis Impaired Parenting related to mother's developmental delay is an example of a(n):

1. Risk nursing diagnosis. 2. Problem-focused nursing diagnosis. 3. Health promotion nursing diagnosis. 4. Wellness nursing diagnosis. 2

39. The unlicensed assistive personnel (UAP) accidentally pulled the client's chest tube out while assisting the client to the bedside commode (BSC). Which intervention should the nurse implement first? 1. Securely tape petroleum gauze over the insertion site. 2. Instruct the UAP how to move a client with a chest tube. 3. Assess the client's respirations and lung sounds. 4. Obtain a chest tube and a chest tube insertion tray.

1. Securely tape petroleum gauze over the insertion site.

10. The nurse is assigned to a quality improvement committee to decide on a quality improvement project for the unit. Which issue should the nurse discuss at the committee meetings? 1. Systems that make it difficult for the nurses to do their job. 2. How unhappy the nurses are with their current pay scale. 3. Collective bargaining activity at a nearby hospital. 4. The number of medication errors committed by an individual nurse.

1. Systems that make it difficult for the nurses to do their job.

114. The psychiatric nurse and mental health worker (MHW) on a psychiatric unit are caring for a group of clients. Which nursing task should the nurse delegate to the MHW? 1. Take the school-aged children to the on-campus classroom. 2. Lead a group therapy session on behavior control. 3. Explain the purpose of recreation therapy to the client. 4. Give a bipolar client a bed bath and shampoo the hair.

1. Take the school-aged children to the on-campus classroom.

88. Which situation should the charge nurse in the critical care unit address first after receiving the shift report? 1. Talk to the family member who is irate over his loved one's nursing care. 2. Complete the 90-day probationary evaluation for a new ICU graduate intern. 3. Call the laboratory concerning the type and crossmatch for a client who needs blood. 4. Arrange for a client to be transferred to the telemetry step-down unit.

1. Talk to the family member who is irate over his loved one's nursing care.

63. A male HCP frequently tells jokes with sexual overtones at the nursing station. Which action should the female charge nurse implement? 1. Tell the HCP that the jokes are inappropriate and offensive. 2. Report the behavior to the medical staff committee. 3. Discuss the problem with the chief nursing of officer 4. Call a Code Purple and have the nurses surround the HCP.

1. Tell the HCP that the jokes are inappropriate and offensive.

112. The newborn nursery nurse has received report. Which client should the nurse assess first? 1. The 2-hour-old infant who has nasal flaring and is grunting. 2. The 6-hour-old infant who has not passed meconium stool. 3. The 12-hour-old infant who refuses to latch onto the breast. 4. The 24-hour-old infant who has a positive startle reflex.

1. The 2-hour-old infant who has nasal flaring and is grunting.

50. The home health nurse is planning his rounds for the day. Which client should the nurse plan to see first? 1. The 56-year-old client diagnosed with multiple sclerosis who is complaining of a cough. 2. The 78-year-old client diagnosed with congestive heart failure (CHF) who reports losing 3 pounds. 3. The 42-year-old client diagnosed with an L-5 spinal cord injury who has developed a Stage 4 pressure ulcer. 4. The 80-year-old client diagnosed with a cerebrovascular accident (CVA) who has right-sided paralysis.

1. The 56-year-old client diagnosed with multiple sclerosis who is complaining of a cough.

86. The nurse is preparing to administer medications to clients on a surgical unit. Which medication should the nurse question administering? 1. The antiplatelet clopidogrel (Plavix) to a client scheduled for surgery. 2. The anticoagulant enoxaparin (Lovenox) to a client who had a TKR. 3. The sliding scale insulin Humalog to a client who had a Whipple procedure. 4. The aminoglycoside vancomycin to a client allergic to the antibiotic penicillin.

1. The antiplatelet clopidogrel (Plavix) to a client scheduled for surgery.

48. The nurse has received the shift report. Which client should the nurse assess first? 1. The client diagnosed with a deep vein thrombosis (DVT) who complains of a feeling of doom. 2. The client diagnosed with gallbladder ulcer disease who refuses to eat the food served. 3. The client diagnosed with pancreatitis who wants the nasogastric tube removed. 4. The client diagnosed with osteoarthritis who is complaining of stiff joints.

1. The client diagnosed with a deep vein thrombosis (DVT) who complains of a feeling of doom.

22. The nurse on a medical unit has just received the evening shift report. Which client should the nurse assess first? 1. The client diagnosed with a deep vein thrombosis (DVT) who has a heparin drip infusion and a PTT of 92. 2. The client diagnosed with pneumonia who has an oral temperature of 100.2°F. 3. The client diagnosed with cystitis who complains of burning on urination. 4. The client diagnosed with pancreatitis who complains of pain that is an 8.

1. The client diagnosed with a deep vein thrombosis (DVT) who has a heparin drip infusion and a PTT of 92.

21. The charge nurse of a critical care unit is making assignments for the night shift. Which client should be assigned to the graduate nurse who has just completed an internship? 1. The client diagnosed with a head injury resulting from a motor vehicle accident (MVA) whose Glasgow Coma Scale score is 13. 2. The client diagnosed with inflammatory bowel disease (IBD) who has severe diarrhea and has a serum K+ level of 3.2 mEq/L. 3. The client diagnosed with Addison's disease who is lethargic and has a BP of 80/45, P of 124, and R rate of 28. 4. The client diagnosed with hyperthyroidism who has undergone a thyroidectomy and has a positive Trousseau's sign.

1. The client diagnosed with a head injury resulting from a motor vehicle accident (MVA) whose Glasgow Coma Scale score is 13.

20. The charge nurse is making assignments in the day surgery center. Which client should be assigned to the most experienced nurse? 1. The client who had surgery for an inguinal hernia and who is being prepared for discharge. 2. The client who is in the preoperative area and who is scheduled for laparoscopic cholecystectomy. 3. The client who has completed scheduled chemotherapy treatment and who is receiving two units of blood. 4. The client who has end-stage renal disease and who has had an arteriovenous fistula created.

1. The client who had surgery for an inguinal hernia and who is being prepared for discharge.

8. Which situation would prompt the healthcare team to utilize the client's advance directive when needing to make decisions for the client? 1. The client with a head injury who is exhibiting decerebrate posturing. 2. The client with a C-6 spinal cord injury (SCI) who is on a ventilator. 3. The client in chronic renal disease who is being placed on dialysis. 4. The client diagnosed with terminal cancer who is mentally retarded.

1. The client with a head injury who is exhibiting decerebrate posturing.

In which of the following examples are nurses making diagnostic errors? (Select all that apply.)

1. The nurse who observes a patient wincing and holding his left side and gathers no additional assessment data 2. The nurse who measures joint range of motion after the patient reports pain in the left elbow 3. The nurse who considers conflicting cues in deciding which diagnostic label to choose 4. The nurse who identifies a diagnosis on the basis of a patient reporting difficulty sleeping 5. The nurse who makes a diagnosis of Ineffective Airway Clearance related to pneumonia. 1, 4, and 5

96. The family member of a client in a long-term care facility is unhappy with the care being provided for the loved one. Which person would be most appropriate to investigate the complaint and report the findings during a client care conference? 1. The ombudsperson for the facility. 2. The social worker for the facility. 3. The family member who is unhappy. 4. The director of nurses.

1. The ombudsperson for the facility.

108. Which statement best describes the role of the parish nurse? 1. The parish nurse practices holistic healthcare within a faith community. 2. The parish nurse cares for clients in a religious-based hospital. 3. The parish nurse practices nursing in a parish clinic. 4. The parish nurse is a licensed practical nurse (LPN) who cares for clients in the home.

1. The parish nurse practices holistic healthcare within a faith community.

diagnostic statement guidelines

1. identify the patient's response, not the medical diagnosis 2. identify a NANDA-I diagnostic statement rather than the symptom 3. identify a treatable cause or risk factor rather than a clinical sign or chronic problem that is not treatable through nursing intervention 4. identify the problem caused by the treatment or diagnostic study rather than the treatment or study itself 5. identify the patient response to the equipment rather than the equipment itself 6. identify the patient's problems rather than your problems with nursing care 7. identify the patient problem rather than the nursing intervention 8. identify the patient problem rather than the goal of care 9. make professional rather than prejudicial judgements 10. avoid legally inadvisable statement 11. identify the problem and its cause to avoid a circular statement 12. identify only one patient problem in the diagnostic statement

A nurse is assigned to a new patient admitted to the nursing unit following admission through the emergency department. The nurse collects a nursing history and interviews the patient. What are the steps for making a nursing diagnosis in the correct order, beginning with the first step? 1. Considers context of patient's health problem and selects a related factor 2. Reviews assessment data, noting objective and subjective clinical information 3. Clusters clinical cues that form a pattern 4. Chooses diagnostic label

2, 3, 4, 1 Review assessment data Cluster clinical cues that form a pattern Choose diagnostic Label Consider context of patients health problem

44. The nurse is taking a history on a client in a women's clinic when the client tells the nurse, "I have been trying to get pregnant for 3 years." Which question is the nurse's best response? 1. "How many attempts have you made to get pregnant?" 2. "What have you tried to help you get pregnant?" 3. "Does your insurance cover infertility treatments?" 4. "Have you considered adoption as an option?"

2. "What have you tried to help you get pregnant?"

97. The 65-year-old client is being discharged from the hospital following major abdominal surgery and is unable to drive. Which referral should the nurse make to ensure continuity of care? 1. A church that can provide transportation. 2. A home health agency. 3. An outpatient clinic. 4. The healthcare provider's office.

2. A home health agency.

75. The HCP is angry and yelling in the nurse's station because the client's laboratory data are not available. Which action should the charge nurse implement first? 1. Contact the laboratory for the client's results. 2. Ask the HCP to step into the nurse's office 3. Tell the HCP to discuss the issue with the laboratory. 4. Report the HCP's behavior to the chief nursing of officer.

2. Ask the HCP to step into the nurse's office

98. The nurse in an assisted living facility notes that the male client has several new bruises on both of his arms and hands. Which intervention should the nurse implement first? 1. File an elder abuse report with the Department of Human Services. 2. Ask the client whether he has fallen and hurt himself during the night. 3. Check the medication administration record (MAR) to determine which medications the client is receiving. 4. Notify the client's family of the bruises so they are not surprised on their visit.

2. Ask the client whether he has fallen and hurt himself during the night.

85. The nurse is caring for clients on a 12-bed intermediate care surgical unit. Which task should the nurse implement first? 1. Reinsert the nasogastric tube for the client who has pulled it out. 2. Complete the preoperative checklist for the client scheduled for surgery. 3. Instruct the client who is being discharged home about colostomy care. 4. Change the client's surgical dressing that has a 20 cm area of drainage.

2. Complete the preoperative checklist for the client scheduled for surgery.

76. The staff nurse is concerned about possible increasing infection rates among clients with peripherally inserted central catheters (PICCs). The nurse has noticed several clients with problems in the last few months. Which action would be appropriate for the staff nurse to implement first? 1. Discuss the infections with the chief nursing officer. 2. Contact the infection control nurse to discuss the problem. 3. Assume the employee health nurse is monitoring the situation. 4. Volunteer to be on an ad hoc committee to research the infection rate.

2. Contact the infection control nurse to discuss the problem.

56. The experienced nurse has recently taken a position on a medical unit in a community hospital, but after 1 week on the job, he finds that the staffing is not what was discussed during his employment interview. Which approach would be most appropriate for the nurse to take when attempting to resolve the issue? 1. Immediately give a 2-week notice and find a different job. 2. Discuss the situation with the manager who interviewed him. 3. Talk with the other employees about the staffing situation. 4. Tell the charge nurse the staffing is not what was explained to him.

2. Discuss the situation with the manager who interviewed him.

26. The director of nurses in a long-term care facility observes the licensed practical nurse (LPN) charge nurse explaining to an unlicensed assistive personnel (UAP) how to calculate the amount of food a resident has eaten from the food tray. Which action should the director of nurses implement? 1. Ask the charge nurse to teach all the other UAPs. 2. Encourage the nurse to continue to work with the UAP. 3. Tell the charge nurse to discuss this in a private area. 4. Give the UAP a better explanation of the procedure.

2. Encourage the nurse to continue to work with the UAP.

91. The nurse in the burn unit is preparing to perform a wound dressing change at the bedside. Which interventions should the nurse implement? Rank in order of priority. 1. Obtain the needed supplies for the procedure. 2. Explain the procedure to the client. 3. Remove the old dressing with non-sterile gloves. 4. Medicate the client with narcotic analgesics. 5. Assess the client's burned area.

2. Explain the procedure to the client. 4. Medicate the client with narcotic analgesics. 1. Obtain the needed supplies for the procedure. 3. Remove the old dressing with non-sterile gloves. 5. Assess the client's burned area.

116. The nurse is caring for a female client 3 days post-knee replacement surgery when the client complains of vaginal itching. The medication administration report (MAR) indicates the client has been receiving the antacid calcium carbonate (Maalox), the antibiotic ceftriaxone (Rocephin), and the anticoagulant enoxaparin (Lovenox). Which priority intervention should the nurse implement? 1. Request the dietary department to send yogurt on each tray. 2. Explain to the client this is the result of the antibiotic therapy. 3. Notify the HCP on rounds of the client's vaginal itching. 4. Ask the client whether she is having unprotected sexual activity.

2. Explain to the client this is the result of the antibiotic therapy.

45. The nurse working at the county hospital is admitting a client who is Rh-negative to the labor and delivery unit. The client is gravida 2, para 0. Which assessment data is most important for the nurse to assess? 1. Why the client did not have a viable baby with the first pregnancy. 2. If the mother received a Rhogam injection after the last pregnancy. 3. The period of time between the client's pregnancies. 4. When the mother terminated the previous pregnancy.

2. If the mother received a Rhogam injection after the last pregnancy.

79. The client tells the nurse, "I am having surgery on my right knee." However, the operative permit is for surgery on the left knee. Which action should the nurse implement first? 1. Notify the operating room team. 2. Initiate the time-out procedure. 3. Clarify the correct extremity with the client. 4. Call the surgeon to discuss the discrepancy.

2. Initiate the time-out procedure.

2. The charge nurse observes two unlicensed assistive personnel (UAPs) arguing in the hallway. Which action should the nurse implement first in this situation? 1. Tell the manager to check on the UAPs. 2. Instruct the UAPs to stop arguing in the hallway. 3. Have the UAPs go to a private room to talk. 4. Mediate the dispute between the UAPs.

2. Instruct the UAPs to stop arguing in the hallway.

103. The chief nursing officer (CNO) of an extended care facility is attending shift report with two charge nurses, and an argument about a resident's care ensues. Which ac- tion should the CNO implement first? 1. Ask the two charge nurses to stop arguing and go to a private area. 2. Listen to both sides of the argument and then implement a plan of care. 3. Ask the family to join the discussion before deciding how to implement care. 4. Tell the nurses to stop arguing and continue to give report.

2. Listen to both sides of the argument and then implement a plan of care.

28. The older adult client becomes confused and wanders in the hallways. Which fall precaution intervention should the nurse implement first? 1. Place a Posey vest restraint on the client. 2. Move the client to a room near the station. 3. Ask the HCP for an antipsychotic medication. 4. Raise all four side rails on the client's bed.

2. Move the client to a room near the station.

30. The employee health nurse has cared for six clients who have similar complaints. The clients have a fever, nausea, vomiting, and diarrhea. Which action should the nurse implement first after assessing the clients? 1. Have another employee drive the clients home. 2. Notify the public health department immediately. 3. Send the clients to the emergency department. 4. Obtain stool specimens from the clients.

2. Notify the public health department immediately.

69. Which task would be most appropriate for the nurse to delegate to the unlicensed assistive personnel (UAP) working on a surgical unit? 1. Escort the client to the smoking area outside. 2. Obtain vital signs on a newly admitted client. 3. Administer a feeding to the client with a gastrostomy tube. 4. Check the toes of a client who just had a cast application.

2. Obtain vital signs on a newly admitted client.

38. Which legal intervention should the nurse implement on the initial visit when admitting a client to the home healthcare agency? 1. Discuss the professional boundary-crossing policy with the client. 2. Provide the client with a copy of the NAHC Bill of Rights. 3. Tell the client how many visits the client will have while on service. 4. Explain that the client must be homebound to be eligible for home healthcare

2. Provide the client with a copy of the NAHC Bill of Rights.

110. The 32-year-old male client with a traumatic right above-the-elbow amputation tells the home health (HH) nurse he is worried about supporting his family and finding employment since he can't be a mechanic anymore. Which intervention should the nurse implement? 1. Contact the HH agency's occupational therapist. 2. Refer the client to the state rehabilitation commission. 3. Ask the HH agency's social worker about disability. 4. Suggest he talk to his wife about his concerns.

2. Refer the client to the state rehabilitation commission.

68. The HCP writes an order for the client with a fractured right hip to ambulate with a walker four times per day. Which action should the nurse implement? 1. Tell the unlicensed assistive personnel (UAP) to ambulate the client with the walker. 2. Request a referral to the physical therapy department. 3. Obtain a walker that is appropriate for the client's height. 4. Notify the social worker of the HCP's order for a walker.

2. Request a referral to the physical therapy department.

7. Which task should the critical care nurse delegate to the unlicensed assistive personnel (UAP)? 1. Check the pulse oximeter reading for the client on a ventilator. 2. Take the client's sterile urine specimen to the laboratory. 3. Obtain the vital signs for the client in an Addisonian crisis. 4. Assist the HCP with performing a paracentesis at the bedside.

2. Take the client's sterile urine specimen to the laboratory.

1. The new graduate working on a medical unit night shift is concerned that the charge nurse is drinking alcohol on duty. On more than one occasion, the new graduate has smelled alcohol when the charge nurse returns from a break. Which action should the new graduate nurse implement first? 1. Confront the charge nurse with the suspicions. 2. Talk with the night supervisor about the concerns. 3. Ignore the situation unless the nurse cannot do her job. 4. Ask to speak to the nurse educator about the problem.

2. Talk with the night supervisor about the concerns.

31. The clinic nurse is caring for clients in a pediatric clinic. Which client should the nurse assess first? 1. The 4-year-old child who fell and is complaining of left leg pain. 2. The 3-year-old child who is drooling and does not want to swallow. 3. The 8-year-old child who has complained of a headache for 2 days. 4. The 10-year-old child who is thirsty all the time and has lost weight.

2. The 3-year-old child who is drooling and does not want to swallow.

82. The charge nurse on a 20-bed surgical unit has one RN, two licensed practical nurses (LPNs), and two unlicensed assistive personnel (UAPs) for a 12-hour shift. Which task would be an inappropriate delegation of assignments? 1. The RN will perform the shift assessments. 2. The LPN should administer all IVP medications. 3. The UAP will complete all a.m. care. 4. The RN will monitor laboratory values.

2. The LPN should administer all IVP medications.

15. The unlicensed assistive personnel (UAP) is preparing to provide postmortem care to a client with a questionable diagnosis of anthrax. Which instruction is priority for the nurse to provide to the UAP? 1. The UAP is not at risk for contracting an illness. 2. The UAP should wear a mask, gown, and gloves. 3. The UAP may skip performing postmortem care. 4. Ask whether the UAP is pregnant before she enters the client's room.

2. The UAP should wear a mask, gown, and gloves.

120. Which data indicates therapy has been effective for the client diagnosed with bipolar disorder? 1. The client only has four episodes of mania in 6 months. 2. The client goes to work every day for 9 months. 3. The client wears a nightgown to the day room for therapy. 4. The client has had three motor vehicle accidents.

2. The client goes to work every day for 9 months.

65. Which client should the nurse in the post-anesthesia care unit (PACU) assess first? 1. The client who received general anesthesia who is complaining of a sore throat. 2. The client who had right knee surgery and has a pulse oximeter reading of 90%. 3. The client who received epidural surgery and has a palpable 2+ dorsalis pedal pulse. 4. The client who had abdominal surgery and has green bile draining from the N/G tube.

2. The client who had right knee surgery and has a pulse oximeter reading of 90%.

64. The night shift nurse is caring for clients on the surgical unit. Which client situation would warrant immediate notification of the surgeon? 1. The client who is 2 days postoperative for bowel resection and who refuses to turn, cough, and deep breathe. 2. The client who is 5 hours postoperative for abdominal hysterectomy who reported feeling a "pop" and then her pain went away. 3. The client who is 2 hours postoperative for TKR and who has 400 mL in the cell-saver collection device. 4. The client who is 1 day postoperative for bilateral thyroidectomy and who has a negative Chvostek sign.

2. The client who is 5 hours postoperative for abdominal hysterectomy who reported feeling a "pop" and then her pain went away.

105. The community health nurse is triaging victims at a bus accident. Which client would the nurse categorize as red, priority 1? 1. The client with head trauma whose pupils are fixed and dilated. 2. The client with compound fractures of the tibia and fibula. 3. The client with a sprained right wrist with a 1-inch laceration. 4. The client with a piece of metal embedded in the right eye.

2. The client with compound fractures of the tibia and fibula.

113. The psychiatric clinic nurse is returning telephone calls. Which telephone call should the nurse return first? 1. The female client who reports being slapped by her husband when he got drunk last night. 2. The male client who reports he is tired of living, since his wife just left him because he lost his job. 3. The female client diagnosed with anorexia who reports she does not think she can stand to eat today. 4. The male client diagnosed with Parkinson's disease who reports his hands are shaking more than yesterday.

2. The male client who reports he is tired of living, since his wife just left him because he lost his job.

32. Which statement is an example of community-oriented, population-focused nursing? 1. The nurse cares for an older adult client who had a kidney transplant and who lives in the community. 2. The nurse develops an educational program for the type 2 diabetics in the community. 3. The nurse refers a client with Cushing's syndrome to the registered dietician. 4. The nurse provides the client chronic renal disease with pamphlets.

2. The nurse develops an educational program for the type 2 diabetics in the community.

24. The surgical unit has a low census and is overstaffed. Which staff member should the house supervisor notify first and request to stay home? 1. The nurse who has the most vacation time. 2. The nurse who requested to be off. 3. The nurse who has the least experience on the unit. 4. The nurse who has called in sick the previous 2 days.

2. The nurse who requested to be off.

109. The HH aide calls the HH nurse to report that the client has a reddened area on the sacral area. Which intervention should the nurse implement first? 1. Notify the client's healthcare provider. 2. Visit the client to assess the reddened area. 3. Document the finding in the client's chart. 4. Refer the client to the wound care nurse.

2. Visit the client to assess the reddened area.

90. The critical care unit is having problems with staff members clocking in late and clocking out early from the shift. Which statement by the charge nurse indicates he has a democratic leadership style? 1. "You cannot clock out 1 minute before your shift is complete." 2. "As long as your work is done you can clock out any time you want." 3. "We are going to have a meeting to discuss the clocking in procedure." 4. "The clinical manager will take care of anyone who clocks out early."

3. "We are going to have a meeting to discuss the clocking in procedure."

94. The newly admitted client in a long-term care facility stays in the room and refuses to participate in client activities. Which statement is a priority for the nurse to discuss with the client? 1. "You have to get out of this room or you will never make friends here at the home." 2. "It is not so bad living here; you are lucky that we care about what happens to you." 3. "You seem sad; would you like to talk about how you are feeling about being here?" 4. "The activities director can arrange for someone to come and visit you in your room."

3. "You seem sad; would you like to talk about how you are feeling about being here?"

52. The physical therapist has notified the unit secretary that the client will be ambulated in 45 minutes. After receiving notification from the unit secretary, which task should the charge nurse delegate to the unlicensed assistive personnel (UAP)? 1. Administer a pain medication 30 minutes before therapy. 2. Give the client a washcloth to wash his or her face before walking. 3. Check to make sure the client has been offered the use of the bathroom. 4. Find a walker that is the correct height for the client to use.

3. Check to make sure the client has been offered the use of the bathroom.

72. The unlicensed assistive personnel (UAP) tells the nurse the client who is 5 hours postoperative for an L-3/L-4 laminectomy is complaining of feeling numbness in both feet. Which intervention should the nurse implement? 1. Ask the UAP to take the client's vital signs. 2. Request the UAP to log roll the client to the right side. 3. Complete the neurovascular assessment on the client's legs. 4. Contact the physical therapist to check the client.

3. Complete the neurovascular assessment on the client's legs.

25. The nurse and the unlicensed assistive personnel (UAP) are caring for residents in a long-term care facility. Which task should the nurse delegate to the UAP? 1. Apply a sterile dressing to a Stage IV pressure wound. 2. Check the blood glucose level of a resident who is weak and shaky. 3. Document the amount of food the residents ate after a meal. 4. Teach the residents how to play different types of bingo.

3. Document the amount of food the residents ate after a meal.

13. The female client with osteoarthritis is 6 weeks postoperative for open reduction and internal fixation of the right hip. The home health (HH) aide tells the HH nurse the client will not get in the shower in the morning because she "hurts all over." Which action would be most appropriate by the HH nurse? 1. Tell the HH aide to allow the client to stay in bed until the pain goes away. 2. Instruct the HH aide to get the client up to a chair and give her a bath. 3. Explain to the HH aide the client should get up and take a warm shower. 4. Arrange an appointment for the client to visit her healthcare provider.

3. Explain to the HH aide the client should get up and take a warm shower.

54. The medical unit is governed by a system of shared governance. Which statement best describes an advantage of this system? 1. It guarantees that unions will not be able to come into the hospital. 2. It makes the manager responsible for sharing information with the staff. 3. It involves staff nurses in the decision-making process of the unit. 4. It is a system used to represent the nurses in labor disputes.

3. It involves staff nurses in the decision-making process of the unit.

89. The nurse in the critical care unit of a medical center answers the phone and the person says, "There is a bomb in the hospital kitchen." Which action should the nurse take? 1. Notify the kitchen that there is a bomb. 2. Call the operator to trace the phone call. 3. Notify the hospital security department. 4. Call the local police department.

3. Notify the hospital security department.

107. The community health nurse is triaging victims at the scene of a building collapse. Which intervention should the nurse implement first? 1. Discuss the disaster situation with the media. 2. Write the client's name clearly in the disaster log. 3. Place disaster tags securely on the victims. 4. Identify an area for family members to wait.

3. Place disaster tags securely on the victims.

71. The unlicensed assistive personnel (UAP) is changing a full sharps container in the client's room. Which action should the nurse implement? 1. Tell the UAP she cannot change the sharps container. 2. Explain the housekeeping department changes the sharps containers. 3. Praise the UAP for taking the initiative to change the sharps container. 4. Report the behavior to the clinical manager on the unit.

3. Praise the UAP for taking the initiative to change the sharps container.

9. The nurse is caring for clients on a skilled nursing unit. Which task should not be delegated to the unlicensed assistive personnel (UAP)? 1. Instruct the UAP to apply sequential compression devices to the client on strict bed rest. 2. Ask the UAP to assist the radiology tech to perform a STAT portable chest x-ray. 3. Request the UAP to prepare the client for a wound debridement at the bedside. 4. Tell the UAP to obtain the intakes and outputs (I&Os) for all the clients on the unit.

3. Request the UAP to prepare the client for a wound debridement at the bedside.

77. The charge nurse on the 30-bed surgical unit has been told to send one staff member to the medical unit. The surgical unit is full, with multiple clients who require custodial care. Which staff member would be most appropriate to send to the medical unit? 1. Send the unlicensed assistive personnel (UAP) who has worked on the surgical unit for 5 years. 2. Send the RN who has worked in the hospital for 8 years in a variety of areas. 3. Send the licensed practical nurse (LPN) who has 3 years of experience, which includes 6 months on the medical unit. 4. Send the new graduate nurse who is orienting to the surgical unit.

3. Send the licensed practical nurse (LPN) who has 3 years of experience, which includes 6 months on the medical unit.

23. The 75-year-old client has undergone an open cholecystectomy for cholelithiasis 2 days ago and has a t-tube drain in place. Which intervention should the nurse delegate to the unlicensed assistive personnel (UAP)? Select all that apply. 1. Explain the procedure for using the patient-controlled analgesia (PCA) pump. 2. Check the client's abdominal dressing for drainage. 3. Take and record the client's vital signs. 4. Empty the client's indwelling catheter bag at the end of the shift. 5. Assist the client to ambulate in the hallway three to four times a day.

3. Take and record the client's vital signs. 4. Empty the client's indwelling catheter bag at the end of the shift. 5. Assist the client to ambulate in the hallway three to four times a day.

34. The home health (HH) nurse along with an HH aide is caring for a client who is 3 weeks postoperative for open reduction and internal fixation of a right hip fracture. Which task would be appropriate for the nurse to delegate to the aide? 1. Instruct the HH aide to palpate the right pedal pulse. 2. Ask the HH aide to change the right hip dressing. 3. Tell the HH aide to elevate the right leg on two pillows. 4. Request the HH aide to mop the client's bedroom floor.

3. Tell the HH aide to elevate the right leg on two pillows.

101. The charge nurse must notify a staff member to stay home because of low census. The unit currently has 35 clients who all have at least one IV and multiple IV medications. The unit is staffed with two RNs, three licensed practical nurses (LPNs), and three unlicensed assistive personnel (UAPs). Which nurse should be notified to stay home? 1. The least experienced RN. 2. The most experienced LPN. 3. The UAP who asked to be requested off. 4. The UAP who was hired 4 weeks ago.

3. The UAP who asked to be requested off.

73. The ED nurse is requesting a bed in the intensive care unit (ICU). The ICU charge nurse must request a transfer of one client from the ICU to the surgical unit to make room for the client coming into the ICU from the ED. Which client should the ICU charge nurse request to transfer to the surgical unit? 1. The client diagnosed with fail chest who has just come from the operating room with a right-sided chest tube. 2. The client diagnosed with acute diverticulitis who is 1 day postoperative for creation of a sigmoid colostomy. 3. The client who is 1 day postoperative for total hip replacement (THR) whose incisional dressing is dry and intact. 4. The client who is 2 days postoperative for repair of a fractured femur and who has had a fat embolism.

3. The client who is 1 day postoperative for total hip replacement (THR) whose incisional dressing is dry and intact.

92. Which client should the charge nurse of a long-term care facility see first after receiving shift report? 1. The client who is unhappy about being placed in a long-term care facility. 2. The client who wants to have the HCP to order a nightly glass of wine. 3. The client who is upset because the call light was not answered for 30 minutes. 4. The client whose son is being discharged from the hospital after heart surgery.

3. The client who is upset because the call light was not answered for 30 minutes.

59. A major disaster has been called, and the charge nurse on a medical unit must recom- mend to the medical discharge officer on rounds which clients to discharge. Which client should not be discharged? 1. The client diagnosed with chronic angina pectoris who has been on new medication for 2 days. 2. The client diagnosed with deep vein thrombosis (DVT) who has had heparin discontinued and has been on warfarin (Coumadin) for 4 days. 3. The client with an infected leg wound who is receiving vancomycin IVPB every 24 hours for methicillin-resistant Staphylococcus aureus (MRSA) infection. 4. The client diagnosed with COPD who has the following arterial blood gas (ABG) levels: pH, 7.34; PCO2, 55; HCO3, 28; PaO2, 89.

3. The client with an infected leg wound who is receiving vancomycin IVPB every 24 hours for methicillin-resistant Staphylococcus aureus (MRSA) infection.

19. The nurse is caring for the following clients on a medical unit. Which client should the nurse assess first? 1. The client with disseminated intravascular coagulation (DIC) who has blood oozing from the intravenous site. 2. The client with benign prostatic hypertrophy (BPH) who is complaining of terminal dribbling and inability to empty bladder. 3. The client with renal calculi who is complaining of severe flank pain and has hematuria. 4. The client with Addison's disease who has bronze skin pigmentation and hypoglycemia.

3. The client with renal calculi who is complaining of severe flank pain and has hematuria.

43. Which assessment data warrants immediate intervention by the nurse for the client diagnosed with chronic kidney disease (CKD) who is on peritoneal dialysis? 1. The client's serum creatinine level is 2.4 mg/dL. 2. The client's abdomen is soft to touch and nontender. 3. The dialysate being removed from the abdomen is cloudy. 4. The dialysate instilled was 1,500 mL and removed was 2,100 mL.

3. The dialysate being removed from the abdomen is cloudy.

104. Which action by the nurse is a violation of the Joint Commission's Patient Safety Goals? 1. The surgery nurse calls a time-out when a discrepancy is noted on the surgical permit. 2. The unit nurse asks the client for his or her date of birth before administering medications. 3. The nurse educator gives the orientee the answers to the quiz covering the IV pumps. 4. The admitting nurse initiates the facility's fall prevention program on an older adult

3. The nurse educator gives the orientee the answers to the quiz covering the IV pumps.

84. The nurse is discharging the 72-year-old client who is 5 days postoperative for repair of a fractured hip with comorbid medical conditions. At this time, which referral would be the most appropriate for the nurse to make for this client? 1. To a home healthcare agency. 2. To a senior citizen center. 3. To a rehabilitation facility. 4. To an outpatient physical therapist.

3. To a rehabilitation facility.

80. The older adult client fell and fractured her left femur. The nurse finds the client crying, and she tells the nurse, "I don't want to go to the nursing home but my son says I have to." Which response would be most appropriate by the nurse? 1. "Let me call a meeting of the healthcare team and your son." 2. "Has the social worker talked to you about this already?" 3. "Why are you so upset about going to the nursing home?" 4. "I can see you are upset. Would you like to talk about it?"

4. "I can see you are upset. Would you like to talk about it?"

3. The graduate nurse is working with an unlicensed assistive personnel (UAP) who has been an employee of the hospital for 12 years. However, tasks delegated to the UAP by the graduate nurse are frequently not completed. Which action should the graduate nurse take first? 1. Tell the charge nurse the UAP will not do tasks as delegated by the nurse. 2. Write up a counseling record with objective data and give it to the manager. 3. Complete the delegated tasks and do nothing about the insubordination. 4. Address the UAP to discuss why the tasks are not being done as requested.

4. Address the UAP to discuss why the tasks are not being done as requested.

53. The volunteer on a medical unit tells the nurse that one of the clients on the unit is her neighbor and asks about the client's condition. Which information should the nurse discuss with the volunteer? 1. Determine how well she knows the client before talking with the volunteer. 2. Tell the volunteer the client's condition in layperson's terms. 3. Ask the client if it is all right to talk with the volunteer. 4. Explain that client information is on a need-to-know basis only.

4. Explain that client information is on a need-to-know basis only.

111. The labor and delivery nurse has assisted in the delivery of a 37-week fetal demise. Which intervention should the nurse implement? 1. Remove the baby from the delivery area quickly. 2. Tell the father to arrange to take the infant home. 3. Wrap the infant in a towel and place it aside. 4. Obtain a lock of the infant's hair for the parents.

4. Obtain a lock of the infant's hair for the parents.

12. The female home health (HH) aide calls the office and reports pain after feeling a pulling in her back when she was transferring the client from the bed to the wheel- chair. Which priority action should the HH nurse tell the HH aide? 1. Explain how to perform isometric exercises. 2. Instruct her to go to the local emergency room. 3. Tell her to complete an occurrence report. 4. Recommend that she apply an ice pack to the back.

4. Recommend that she apply an ice pack to the back.

6. The nurse, a licensed practical nurse (LPN), and an unlicensed assistive personnel (UAP) are caring for clients in a critical care unit. Which task would be most appropriate for the nurse to assign/delegate? 1. Instruct the UAP to obtain the client's serum glucose level. 2. Request the LPN to change the central line dressing. 3. Ask the LPN to bathe the client and change the bed linens. 4. Tell the UAP to obtain urine output for the 12-hour shift.

4. Tell the UAP to obtain urine output for the 12-hour shift.

18. The charge nurse is making assignments on a pediatric unit. Which client should be assigned to the licensed practical nurse (LPN)? 1. The 6-year-old client diagnosed with sickle cell crisis. 2. The 8-year-old client diagnosed with biliary atresia. 3. The 10-year-old client diagnosed with anaphylaxis. 4. The 11-year-old client diagnosed with pneumonia.

4. The 11-year-old client diagnosed with pneumonia.

83. The head nurse is completing the yearly performance evaluation on a nurse. Which data regarding the nurse's performance should be included in the evaluation? 1. The number of times the nurse has been tardy. 2. The attitude of the nurse at the client's bedside. 3. The thank you notes the nurse received from clients. 4. The chart audits of the clients for whom the nurse cared.

4. The chart audits of the clients for whom the nurse cared.

33. The home health (HH) agency director of nursing is making assignments for the nurses. Which client should be assigned to the HH nurse new to HH nursing? 1. The client diagnosed with AIDS who is dyspneic and confused. 2. The client who does not have the money to get prescriptions filled. 3. The client with full-thickness burns on the arm who needs a dressing change. 4. The client complaining of pain who is diagnosed with diabetic neuropathy.

4. The client complaining of pain who is diagnosed with diabetic neuropathy.

36. Which client would most benefit from acupuncture, a traditional Chinese medicine considered complementary alternative medicine? 1. The client who is diagnosed with deep vein thrombosis. 2. The client who is diagnosed with Alzheimer's disease. 3. The client diagnosed with reactive airway disease. 4. The client diagnosed with osteoarthritis.

4. The client diagnosed with osteoarthritis.

35. The charge nurse has received laboratory data for clients in the medical department. Which client would require intervention by the charge nurse? 1. The client diagnosed with a myocardial infarction (MI) who has an elevated troponin level. 2. The client receiving the IV anticoagulant heparin who has a partial thromboplastin time (PTT) of 68 seconds. 3. The client diagnosed with end-stage liver failure who has an elevated ammonia level. 4. The client receiving the anticonvulsant phenytoin (Dilantin) who has levels of 24 mg/dL.

4. The client receiving the anticonvulsant phenytoin (Dilantin) who has levels of 24 mg/dL.

87. The nurse is caring for clients on a surgical intensive care unit. Which client should the nurse assess first? 1. The client who is 4 hours postoperative for abdominal surgery who is complaining of abdominal pain and has hypoactive bowel sounds. 2. The client who is 1 day postoperative for total hip replacement (THR) who has voided 550 mL of clear amber urine in the last 8 hours. 3. The client who is 8 hours postoperative for open cholecystectomy who has a T-tube draining green bile. 4. The client who is 12 hours postoperative for total knee replacement (TKR) who is complaining of numbness and tingling in the foot.

4. The client who is 12 hours postoperative for total knee replacement (TKR) who is complaining of numbness and tingling in the foot.

100. The admitting nurse is subpoenaed to give testimony in a case in which the client fell from the bed and fractured the left hip. The nurse initiated fall precautions on ad- mission but was not on duty when the client fell. Which issue should the nurse be prepared to testify about the incident? 1. What preceded the client's fall from the bed. 2. The extent of injuries the client sustained. 3. The client's mental status before the incident. 4. The facility's policy covering falls prevention.

4. The facility's policy covering falls prevention.

49. The nurse and the unlicensed assistive personnel (UAP) are caring for clients on a pediatric unit. Which task should the nurse delegate to the UAP? 1. Sit with the 6-year-old client while the parent goes outside to smoke. 2. Stay with the 4-year-old client during scheduled play therapy sessions. 3. Position the 2-year-old client for the postural drainage therapy. 4. Weigh the diaper of the 6-month-old client who is on intake and output (I&O).

4. Weigh the diaper of the 6-month-old client who is on intake and output (I&O).

47. The 24-month-old toddler is admitted to the pediatric unit with vomiting and diarrhea. Which interventions should the nurse implement? Rank in order of performance. 1. Teach the parent about weighing diapers to determine output status. 2. Show the parent the call light and explain safety regimens. 3. Assess the toddler's tissue turgor. 4. Place the appropriate size diapers in the room. 5. Take the toddler's vital signs.

5. Take the toddler's vital signs. 3. Assess the toddler's tissue turgor. 2. Show the parent the call light and explain safety regimens 4. Place the appropriate size diapers in the room. 1. Teach the parent about weighing diapers to determine output status.

For a student to avoid a data collection error they should? a. assess the patient and if unsure of the finding, ask a faculty member to assess the patient. b. Review his/her own comfort level and competency with assessment skills. c. Ask another student to preform assessment. d. Consider whether the diagnosis should be actual, potential, or risk.

A

Nurse- initiated interventions are: a. determined by state Nurse Practice Acts. b. Supervised by the entire health care team. c. Made in concert with the plan of care initiated by the physician. d. Developed after interventions for the recent medical diagnosis are evaluated.

A

You have finished with several nursing interventions. To evaluate interventions, you need to examine the: a. Appropriateness of the interventions and the correct application of the implementation process. b. Nursing diagnosis to ensure that they are not medical diagnoses. c. Care planning process for errors in other health care team members' judgments d. interventions of each nurse to enable the nurse manager to correctly evaluate performance.

A

Goal

A broad statement that describes the desired change in a patient's condition, perceptions, or behavior An aim, intent, or end

Define goal.

A broad statement that describes the desired change in a patient's condition, perceptions, or behaviors. An aim, intent, or end.

Hand off Reporting

A critical time, when nurses collaborate and share important information that ensures the continuity of care for a patient and prevents errors or delays in providing nursing interventions Transferring essential information from one nurse to the next during transitions in care Ask questions, clarify, and confirm important details about a patient's progress and continuing care needs

Nursing-sensitive patient outcome

A measurable patient, family, or community state, behavior, or perception largely influenced by and sensitive to nursing interventions

Explain what a nursing care plan is.

A nursing care plan is nursing diagnoses, goals and expected outcomes, and nursing interventions, and a section for evaluation findings so any nurse is able to quickly identify a patient's clinical needs and situation. They reduce the risk for incomplete, incorrect, or inaccurate care. Changes as the patient's problems and status changes.

Patient centered goal

A patient's highest possible level of wellness and independence in function, based on patient needs, abilities, and resources

standing orders

A preprinted document containing orders for the conduct of routine therapies, monitoring guidelines, and/or diagnostic procedures for specific patients with identified clinical problems

A nurse caring for a client admitted with a deep vein thrombosis is individualizing a prepared plan of care that identifies nursing diagnoses, outcomes, and related nursing interventions common to this condition. What type of tool is the nurse using? A standardized care plan An order set Guidelines An algorithm

A standardized care plan Explanation: Standardized care plans are prepared plans of care that identify nursing diagnoses, outcomes, and related nursing interventions common to a specific population or health problem. An algorithm in nursing is a set of developed evidence-based clinical practice guidelines that guides nursing interventions. A guideline is a statement by which to determine a course of action. An order set is a predetermined set of orders by a prescriber that dictates care of the client.

For a student to avoid a date collection error, the student should: A) Assess the patient and, if unsure of the finding, ask a faculty member to assess the patient. B) Review his/her own comfort level and competency with assessment skills C) Ask another student to preform the assessment D) Consider weather the diagnosis should be actual, potential, or risk

A) !!! Always as a faculty member for help if you are confused/have questions!

IV potassium chloride (KCL) is prescribed for treatment of a patient with severe hypokalemia. IV KCL infusion rates should NOT exceed __________ unless the patient is in a critical care setting with continuous ECG monitoring and central line access for administration. A) 10 mEq/hr B) 20 mEq/hr C) 40 mEq/hr D) 60 mEq/hr

A) 10 mEq/hr

SAFETY Alerts for administering IV KCL, include all of the following, EXCEPT: A) Administer the KCL as a rapid IV bolus B) Never give KCL via IV push C) IV KCL must always be diluted and never given in concentrated amounts D) Continue cardiac monitoring during the infusion

A) Administer the KCL as a rapid IV bolus

1. A nurse reviews data gathered regarding a patient's ability to cope with loss. The nurse compares the defining characteristics for Ineffective Coping with those for Readiness for Enhanced Coping and selects Ineffective Coping as the correct diagnosis. This is an example of the nurse avoiding an error in: (Select all that apply.) a. Data collection b. Data clustering c. Data interpretation d. Making a diagnostic statement e. Goal setting

A) Data Collection C) Data interpretation This is an example of an error in interpretation and data collection. When making a diagnosis, the nurse must interpret data that he or she has collected by identifying and organizing relevant assessment patterns to support the presence of patient problems. In the case of the two diagnoses in this question, there can be conflicting cues. The nurse must obtain more information and recognize the cues that point to the correct diagnosis.

9. Review the following problem-focused nursing diagnoses and identify the diagnoses that are stated correctly. (Select all that apply.) A) Impaired Skin Integrity related to physical immobility B) Fatigue related to heart disease C) Nausea related to gastric distention D) Need for improved Oral Mucosa Integrity related to inflamed mucosa E) Risk for Infection related to surgery

A) Impaired skin integrity related to physical immobility C) nausea related to gastric detention. Both of these can be corrected through nursing intervention (unlike B), There must be a diagnosis and not a goal of care (unlike D), and E is incorrect because it has not occured yet. The related factors in diagnoses "Fatigue related to heart disease" and "Need for improved oral mucosa integrity related to inflamed mucosa" are incorrect. The related factor of a medical diagnosis (in Fatigue related to heart disease) cannot be corrected through nursing intervention. In "Need for improved oral mucosa integrity related to inflamed mucosa" there is no diagnosis, but instead a goal of care. "Risk for infection related to surgery" is incorrect; risk nursing diagnoses do not have defining characteristics or related factors because they have not occurred yet.

Anxiety can arise from lack of knowledge and patients may become anxious when facing hospital admissions because of the unknown. When anxiety levels are high, the nurse knows that the following can be reduced in the patient, except:. A) Intelligence B) Cognition C) Decision Making D) Coping Abilities

A) Intelligence

You are admitting a patient with complaints of abdominal pain and constipation. The patient also states they have been vomiting. A proximal bowel obstruction is suspected. Which acid-base imbalance do you anticipate in this patient? A) Metabolic Alkalosis B) Respiratory Acidosis C) Respiratory Alkalosis D) Metabolic Acidosis

A) Metabolic Alkalosis

Spironolactone (Aldactone), a potassium sparing diuretic, is prescribed for a patient. The nurse teaches all of the following EXCEPT: A) You may combine this drug with other potassium-sparing diuretics B) This drug may cause hypotension C) This drug inhibits potassium excreting effects, causing hyperkalemia D) This drug inhibits the sodium retaining effects, causing hyponatremia

A) You may combine this drug with other potassium-sparing diuretics

The four hallmark clinical manifestations of an obstruction are all of the following, except: A) diarrhea B) abdominal pain C) vomiting D) distention and constipation

A) diarrhea

In which of the following examples are nurses making diagnostic errors? (Select all that apply.) a. The nurse who observes a patient wincing or holding his left side and gathers no additional assessment data b. The nurse who measures joint range of motion after the patient reports pain in the left elbow c. The nurse who considers conflicting cues in deciding which diagnostic label to choose d. The nurse who identifies a diagnosis on the basis of a patient reporting difficulty sleeping e. The nurse who makes a diagnosis of Ineffective Airway Clearance related to pneumonia

A) error: observing a patient wincing or holding left side and gathering no additional assessment data D) error: identifying a diagnosis on the basis of a patient reporting difficulty sleeping E) error : making a diagnosis of ineffective airway clearance related to pneumonia When the nurse observes the patient wincing and holding his left side but does not gather additional assessment data, he or she makes a data collection error by omitting important data (i.e., pain severity). A nursing diagnosis cannot be made on basis of a single defining characteristic, as seen when the nurse identifies a diagnosis on the basis of a patient reporting difficulty sleeping. A nursing diagnosis needs to be related to a patient's response, not a medical diagnosis such as pneumonia. The nurse who measures joint range of motion after the patient reports pain is correctly validating findings. Considering conflicting clues ensures that the nurse does not make an interpretation error.

The nurse follows which steps to objectively evaluate the degree of success in achieving outcomes of care for DJ. (Select all that apply) a. Identify the exact desired patient behavior b. Evaluate the patient's actual behavior c. Compare the outcome criteria with the actual behavior d. Assess the desired behavior and anticipate outcome. e. Judge the degree of agreement between the outcome criteria and the actual behavior

A, B, C, E

You are developing the plan of care for DJ, including writing the goals and expected outcomes include which of the following? (Select all that apply) a. Measurable b. Time-limited c. Observable d. Diagnostic e. Realistic

A, B, C, E

Spironolactone (Aldactone), an aldosterone antagonist, is prescribed for a patient. Which statement, by the patient, indicates that the teaching about this medication has been effective? A. "I will eat peaches for a snack instead of honeydew." B. "I will use a salt-substitute to decrease my sodium intake" C. "I will increase my intake of avacados, tomatoes and peanut butter." D. "I will include yogurt in my diet"

A. "I will eat peaches for a snack instead of honeydew."

12. A nurse is getting ready to assess a patient in a neighborhood community clinic. He was newly diagnosed with diabetes just a month ago. He has other health problems and a history of not being able to manage his health. Which of the following questions reflects the nurse's cultural competence in making an accurate diagnosis? (Select all that apply.) a. How is your diabetic diet affecting you and your family? b. You seem to not want to follow health guidelines. Can you explain why? c. What worries you the most about having diabetes? d. What do you expect from us when you do not take your insulin as instructed? e. What do you believe will help you control your blood sugar?

A: How is it affecting you and your family? C: What worries you most about having diabetes? E: What do you believe will help you control your blood sugar? Asking "How is your diabetic diet affecting you and your family?" "What worries you the most about having diabetes?" and "What do you believe will help you control your blood sugar?" are open-ended and allow the patient to share his values and health practices. The statements "You seem to not want to follow health guidelines. Can you explain why?" and "What do you expect from us when you do not take your insulin as instructed?" both show the nurse's bias.

13. The use of standard formal nursing diagnostic statements serves several purposes in nursing practice, including which of the following? (Select all that apply.) a. Defines a patients problem, giving members of the health care team a common language for understanding the patients needs. b. Allows physicians and allied health staff to communicate with nurses how they provide care among themselves c. Helps nurses focus on the scope of the nursing practice. d. Creates practice guidelines for collaborative health care activities e. Builds and expands nursing knowledge

A: defines the patients problem, giving members of the health care team a common language for understanding the patients needs. C: helps nurses focus on the scope of the nursing practice E: builds and expands nursing knowledge The use of nursing diagnosis creates a common language for nurses to communicate patient care needs, allows nurses to focus on the realm and scope of nursing practice, and helps to develop nursing knowledge. It is not a language for physicians and allied health staff because they do not rely on providing nursing interventions. Terminology in nursing diagnosis may be familiar to other health care providers but not in a way for directing nursing interventions. Nursing diagnosis has the purpose of creating practice guidelines for nursing.

26. During a routine physical examination of a 70-year-old patient, a blowing sound is auscultated over the carotid artery. Which assessment finding will the nurse report to the health care provider? a.Bruit b.Thrill c.Phlebitis d.Right-sided heart failure

ANS: A A bruit is the sound of turbulence of blood passing through a narrowed blood vessel and is auscultated as a blowing sound. A bruit can reflect cardiovascular disease in the carotid artery of middle-aged to older adults. Intensity or loudness is related to the rate of blood flow through the heart or the amount of blood regurgitated. A thrill is a continuous palpable sensation that resembles the purring of a cat. Jugular venous distention, not bruit, is a possible sign of right-sided heart failure. Some patients with heart disease have distended jugular veins when sitting. Phlebitis is an inflammation of a vein that occurs commonly after trauma to the vessel wall, infection, immobilization, and prolonged insertion of IV catheters. It affects predominantly peripheral veins.

30. A nurse is caring for a group of patients. Which patient will the nurse see first? a.An adult with an S4 heart sound b.A young adult with an S3 heart sound c.An adult with vesicular lung sounds in the lung periphery d.A young adult with bronchovesicular breath sounds between the scapula posteriorly

ANS: A A fourth heart sound (S4) occurs when the atria contract to enhance ventricular filling. An S4 is often heard in healthy older adults, children, and athletes, but it is not normal in adults. Because S4 also indicates an abnormal condition, report it to a health care provider. An S3 is considered abnormal in adults over 31 years of age but can often be heard normally in children and young adults. Vesicular lungs sounds in the periphery and bronchovesicular lung sounds in between the scapula are normal findings.

4. A nurse is developing a care plan for a patient with a pelvic fracture on bed rest. Which goal statement is realistic for the nurse to assign to this patient? a.Patient will increase activity level this shift. b.Patient will turn side to back to side with assistance every 2 hours. c.Patient will use the walker correctly to ambulate to the bathroom as needed. d.Patient will use a sliding board correctly to transfer to the bedside commode as needed.

ANS: A A goal is a broad statement of desired change; the patient will increase activity level is a broad statement. Turning is the expected outcome. When determining goals, the nurse needs to ensure that the goal is individualized and realistic for the patient. Since the patient is on bed rest, using a walker and bedside commode is contraindicated.

22. During a genitourinary examination of a 30-year-old male patient, the nurse identifies a small amount of a white, thick substance on the patient's uncircumcised glans penis. What is the nurse's next step? a.Record this as a normal finding. b.Avoid embarrassing questions about sexual activity. c.Notify the provider about a suspected sexually transmitted infection. d.Tell the patient to avoid doing self-examinations until symptoms clear.

ANS: A A small amount of thick, white smegma sometimes collects under the foreskin in the uncircumcised male and is considered normal. Penile pain or swelling, genital lesions, and urethral discharge are signs and symptoms that may indicate sexually transmitted infections (STI). All men 15 years and older need to perform a male-genital self-examination monthly. The nurse needs to assess a patient's sexual history and use of safe sex habits. Sexual history reveals risks for STI and HIV.

3. The standing orders for a patient include acetaminophen 650 mg every 4 hours prn for headache. After assessing the patient, the nurse identifies the need for headache relief and determines that the patient has not had acetaminophen in the past 4 hours. Which action will the nurse take next? a.Administer the acetaminophen. b.Notify the health care provider to obtain a verbal order. c.Direct the nursing assistive personnel to give the acetaminophen. d.Perform a pain assessment only after administering the acetaminophen.

ANS: A A standing order is a preprinted document containing orders for the conduct of routine therapies, monitoring guidelines, and/or diagnostic procedures for specific patients with identified clinical problems. The nurse will administer the medication. Notifying the health care provider is not necessary if a standing order exists. The nursing assistive personnel are not licensed to administer medications; therefore, medication administration should not be delegated to this person. A pain assessment should be performed before and after pain medication administration to assess the need for and effectiveness of the medication.

7. The nurse is reviewing a patient's database for significant changes and discovers that the patient has not voided in over 8 hours. The patient's kidney function lab results are abnormal, and the patient's oral intake has significantly decreased since previous shifts. Which step of the nursing process should the nurse proceed to after this review? a.Diagnosis b.Planning c.Implementation d.Evaluation

ANS: A After a thorough assessment, the nurse should proceed to analyzing the data and formulating a nursing diagnosis before proceeding with developing the plan of care and determining appropriate interventions; this is the diagnosis phase. The evaluation phase involves determining whether the goals were met and interventions were effective.

5. The following statements are on a patient's nursing care plan. Which statement will the nurse use as an outcome for a goal of care? a.The patient will verbalize a decreased pain level less than 3 on a 0 to 10 scale by the end of this shift. b.The patient will demonstrate increased tolerance to activity over the next month. c.The patient will understand needed dietary changes by discharge. d.The patient will demonstrate increased mobility in 2 days.

ANS: A An expected outcome is a specific and measurable change that is expected as a result of nursing care. Verbalizing decreased pain on a 0 to 10 scale is an outcome. The other three options in this question are goals. Demonstrating increased mobility in 2 days and understanding necessary dietary changes by discharge are short-term goals because they are expected to occur in less than a week. Demonstrating increased tolerance to activity over a month-long period is a long-term goal because it is expected to occur over a longer period of time.

4. Which action indicates a nurse is using critical thinking for implementation of nursing care to patients? a.Determines whether an intervention is correct and appropriate for the given situation b.Reads over the steps and performs a procedure despite lack of clinical competency c.Establishes goals for a particular patient without assessment d.Evaluates the effectiveness of interventions

ANS: A As you implement interventions, use critical thinking to confirm whether the interventions are correct and still appropriate for a patient's clinical situation. You are responsible for having the necessary knowledge and clinical competency to perform interventions for your patients safely and effectively. The nurse needs to recognize the safety hazards of performing an intervention without clinical competency and seek assistance from another nurse. The nurse cannot evaluate interventions until they are implemented. Patients need ongoing assessment before establishing goals because patient conditions can change very rapidly.

14. A nurse is developing a care plan. Which intervention is most appropriate for the nursing diagnostic statement Risk for loneliness related to impaired verbal communication? a.Provide the patient with a writing board each shift. b.Obtain an interpreter for the patient as soon as possible. c.Assist the patient in performing swallowing exercises each shift. d.Ask the family to provide a sitter to remain with the patient at all times.

ANS: A Choose interventions to alter the etiological (related to) factor or causes of the diagnosis. If the etiology is impaired verbal communication, then the nurse should choose an intervention that will address the problem. Providing the patient with a writing board will allow the patient to communicate by writing because the patient is unable to communicate verbally at this time. Obtaining an interpreter might be an appropriate intervention if the patient spoke a foreign language. Assisting with swallowing exercises will help the patient with swallowing, which is a different etiology than impaired verbal communication. Asking the family to provide a sitter at all times is many times unrealistic and does not relate to the impaired verbal communication; the goal would relate to the loneliness.

15. A nurse adds a nursing diagnosis to a patient's care plan. Which information did the nurse document? a.Decreased cardiac output related to altered myocardial contractility. b.Patient needs a low-fat diet related to inadequate heart perfusion. c.Offer a low-fat diet because of heart problems. d.Acute heart pain related to discomfort.

ANS: A Decreased cardiac output related to altered myocardial contractility is a correctly written nursing diagnosis. Patient needs a low-fat diet related to inadequate heart perfusion is a goal phrased statement, not a nursing diagnosis. Offer a low-fat diet is an intervention, not a diagnosis. Acute pain related to discomfort is a circular diagnosis and gives no direction to nursing care.

4. A nurse is conducting Weber's test. Which action will the nurse take? a.Place a vibrating tuning fork in the middle of patient's forehead. b.Place a vibrating tuning fork on the patient's mastoid process. c.Compare the number of seconds heard by bone versus air conduction. d.Compare the patient's degree of joint movement to the normal level.

ANS: A During Weber's test (lateralization of sound), the nurse places the vibrating tuning fork in the middle of the patient's forehead. During a Rinne test (comparison of air and bone conduction), the nurse places a vibrating tuning fork on the patient's mastoid process and compares the length of time air and bone conduction is heard. Comparing the patient's degree of joint movement to the normal level is a test for range of motion.

19. A hospital's wound nurse consultant made a recommendation for nurses on the unit about how to care for the patient's dressing changes. Which action should the nurses take next? a.Include dressing change instructions and frequency in the care plan. b.Assume that the wound nurse will perform all dressing changes. c.Request that the health care provider look at the wound. d.Encourage the patient to perform the dressing changes.

ANS: A Incorporate the consultant's recommendations into the care plan. The wound nurse clearly recommends that nurses on the unit, not the patient, should continue dressing changes. The nurses should not make a wrong assumption that the wound nurse is doing all the dressing changes. The recommendation states for the nurses to do the dressing changes. If the nurses feel strongly about obtaining another opinion, then the health care provider should be contacted. No evidence in the question suggests that the patient needs a second opinion.

20. The nurse is revising the care plan. In which order will the nurse perform the tasks, beginning with the first step? 1. Revise specific interventions. 2. Revise the assessment column. 3. Choose the evaluation method. 4. Delete irrelevant nursing diagnoses. a.2, 4, 1, 3 b.4, 2, 1, 3 c.3, 4, 2, 1 d.4, 2, 3, 1

ANS: A Modification of an existing written care plan includes four steps: 1. Revise data in the assessment column to reflect the patient's current status. Date any new data to inform other members of the health care team of the time that the change occurred. 2. Revise the nursing diagnoses. Delete nursing diagnoses that are no longer relevant and add and date any new diagnoses. Revise related factors and the patient's goals, outcomes, and priorities. Date any revisions. 3. Revise specific interventions that correspond to the new nursing diagnoses and goals. Be sure that revisions reflect the patient's present status. 4. Choose the method of evaluation for determining whether you achieved patient outcomes.

17. A staff development nurse is providing an inservice for other nurses to educate them about the Nursing Interventions Classification (NIC) system. During the inservice, which statement made by one of the nurses in the room requires the staff development nurse to clarify the information provided? a."This system can help medical students determine the cost of the care they provide to patients." b."If the nursing department uses this system, communication among nurses who work throughout the hospital may be enhanced." c."We could use this system to help organize orientation for new nursing employees because we can better explain the nursing interventions we use most frequently on our unit." d."The NIC system provides one way to improve safe and effective documentation in the hospital's electronic health record."

ANS: A NIC does not help determine the cost of services provided by nurses. The staff development nurse would need to correct this misconception. Because this system is specific to nursing practice, it would not help medical students determine the costs of care. The NIC system developed by the University of Iowa differentiates nursing practice from that of other health care disciplines. All the other statements are true. Benefits of using NIC include enhancing communication among nursing staff and documentation, especially within health information systems such as an electronic documentation system. NIC also helps nurses identify the nursing interventions they implement most frequently. Units that identify routine nursing interventions can use this information to develop checklists for orientation.

23. The nurse is preparing for a rectal examination of a nonambulatory male patient. In which position will the nurse place the patient? a.Sims' b.Knee-chest c.Dorsal recumbent d.Forward bending with flexed hips

ANS: A Nonambulatory patients are best examined in a side-lying Sims' position. Forward bending would require the patient to be able to stand upright. Knees to chest would be difficult to maintain in a nonambulatory male and is embarrassing and uncomfortable. Dorsal recumbent does not provide adequate access for a rectal examination and is used for abdominal assessment because it promotes relaxation of abdominal muscles.

13. Which initial intervention is most appropriate for a patient who has a new onset of chest pain? a.Reassess the patient. b.Notify the health care provider. c.Administer a prn medication for pain. d.Call radiology for a portable chest x-ray.

ANS: A Preparation for implementation ensures efficient, safe, and effective nursing care; the first activity is reassessment. The cause of the patient's chest pain is unknown, so the patient needs to be reassessed before pain medication is administered or a chest x-ray is obtained. The nurse then notifies the patient's health care provider of the patient's current condition in anticipation of receiving further orders. The patient's chest pain could be due to muscular injury or a pulmonary issue. The nurse needs to reassess first.

8. A nurse identifies lice during a child's scalp assessment. The nurse teaches the parents about hair care. Which information from the parents indicates the nurse needs to follow up? a.We will use lindane-based shampoos. b.We will use the sink to wash hair. c.We will use a fine-toothed comb. d.We will use a vinegar hair rinse.

ANS: A Products containing lindane, a toxic ingredient, often cause adverse reactions; the nurse will need to follow up to correct the misconception. All the rest are correct. Instruct parents who have children with head lice to shampoo thoroughly with pediculicide (shampoo available at drugstores) in cold water at a basin or sink, comb thoroughly with a fine-toothed comb, and discard the comb. A dilute solution of vinegar and water helps loosen nits.

16. The patient is a 45-year-old African-American male who has come in for a routine annual physical. Which type of preventive screening does the nurse discuss with the patient? a.Digital rectal examination of the prostate b.Complete eye examination every year c.CA 125 blood test once a year d.Colonoscopy every 3 years

ANS: A Recommended preventive screenings include a digital rectal examination of the prostate and prostate-specific antigen test starting at age 50. CA 125 blood tests are indicated for women at high risk for ovarian cancer. Patients over the age of 65 need to have complete eye examinations yearly. Colonoscopy every 10 years is recommended in patients 50 years of age and older.

19. The nurse is intervening for a patient that has a risk for a urinary infection. Which direct care nursing intervention is most appropriate? a.Teaches proper handwashing technique b.Properly cleans the patient's toilet c.Transports urine specimen to the lab d.Informs the oncoming nurse during hand-off

ANS: A Teaching proper handwashing technique is a direct care nursing intervention. All the rest are indirect nursing care: cleaning the toilet, transporting specimens, and performing hand-off reports.

9. A parent calls the school nurse with questions regarding the recent school vision screening. Snellen chart examination revealed 20/60 for both eyes. Which response by the nurse is the best regarding the eye examination results? a.Your child needs to see an ophthalmologist. b.Your child is suffering from strabismus. c.Your child may have presbyopia. d.Your child has cataracts.

ANS: A The child needs an eye examination with an ophthalmologist or optometrist. Normal vision is 20/20. The larger the denominator, the poorer the patient's visual acuity. For example, a value of 20/60 means that the patient, when standing 20 feet away, can read a line that a person with normal vision can read from 60 feet away. Strabismus is a (congenital) condition in which both eyes do not focus on an object simultaneously: The eyes appear crossed. Acuity may not be affected; Snellen test does not test for strabismus. Presbyopia is impaired near vision that occurs in middle-aged and older adults and is caused by loss of elasticity of the lens. Cataracts, a clouding of the lens, develop slowly and progressively after age 35 or suddenly after trauma.

9. A nurse administers an antihypertensive medication to a patient at the scheduled time of 0900. The nursing assistive personnel (NAP) then reports to the nurse that the patient's blood pressure was low when it was taken at 0830. The NAP states that was busy and had not had a chance to tell the nurse yet. The patient begins to complain of feeling dizzy and light-headed. The blood pressure is rechecked and it has dropped even lower. In which phase of the nursing process did the nurse first make an error? a.Assessment b.Diagnosis c.Implementation d.Evaluation

ANS: A The diagnostic process should flow from the assessment. In this case, the nurse should have assessed the patient's blood pressure before giving the medication. The nurse could have prevented the patient's untoward reaction if the low blood pressure was assessed first. Diagnosis follows assessment. Administering the medication occurs in implementation, but this is not the first error. There are no errors in evaluation.

18. A nurse is preparing to make a consult. In which order, beginning with the first step, will the nurse take? 1. Identify the problem. 2. Discuss the findings and recommendation. 3. Provide the consultant with relevant information about the problem. 4. Contact the right professional, with the appropriate knowledge and expertise. 5. Avoid bias by not providing a lot of information based on opinion to the consultant. a.1, 4, 3, 5, 2 b.4, 1, 3, 2, 5 c.1, 4, 5, 3, 2 d.4, 3, 1, 5, 2

ANS: A The first step in making a consultation is to assess the situation and identify the general problem area. Second, direct the consultation to the right professional such as another nurse or social worker. Third, provide a consultant with relevant information about the problem area and seek a solution. Fourth, do not prejudice or influence consultants. Fifth, be available to discuss a consultant's findings and recommendations.

10. A nurse is caring for a patient with a nursing diagnosis of Constipation related to slowed gastrointestinal motility secondary to pain medications. Which outcome is most appropriate for the nurse to include in the plan of care? a.Patient will have one soft, formed bowel movement by end of shift. b.Patient will walk unassisted to bathroom by the end of shift. c.Patient will be offered laxatives or stool softeners this shift. d.Patient will not take any pain medications this shift.

ANS: A The identified problem, or nursing diagnosis, is constipation. Therefore, the outcome should be that the constipation is relieved. To measure constipation relief, the nurse will be observing for the patient to have a bowel movement. During planning, you select goals and expected outcomes for each nursing diagnosis or problem to provide clear direction for the type of interventions needed to care for your patient and to then evaluate the effectiveness of these interventions. Not taking pain medications may or may not relieve the constipation. Although not taking pain medicines might be an intervention, the nurse doesn't want the patient to be in pain to relieve constipation. Other measures, such as administering laxatives or stool softeners, might be appropriate interventions but they are not outcomes. The patient walking unassisted to the bathroom addresses mobility, not constipation. The patient may need to walk to the bathroom to have a bowel movement, but the appropriate outcome for constipation is that the constipation is relieved as evidenced by a bowel movement—something that the nurse can observe.

12. A male student comes to the college health clinic. He hesitantly describes that he found something wrong with his testis when taking a shower. Which assessment finding will alert the nurse to possible testicular cancer? a.Hard, pea-sized testicular lump b.Rubbery texture of testes c.Painful enlarged testis d.Prolonged diuretic use

ANS: A The most common symptoms of testicular cancer are a painless enlargement of one testis and the appearance of a palpable, small, hard lump, about the size of a pea, on the front or side of the testicle. Normally, the testes feel smooth, rubbery, and free of nodules. Use of diuretics, sedatives, or antihypertensives can lead to erection or ejaculation problems.

19. A nurse assesses that a patient has not voided in 6 hours. Which question should the nurse ask to assist in establishing a nursing diagnosis of Urinary retention? a."Do you feel like you need to go to the bathroom?" b."Are you able to walk to the bathroom by yourself?" c."When was the last time you took your medicine?" d."Do you have a safety rail in your bathroom at home?"

ANS: A The nurse must establish that the patient feels the urge and is unable to void. The question "Do you feel like you need to go to the bathroom?" is the most appropriate to ask. This question can be answered without knowledge of the diagnosis of Urinary retention. Discussing the ability to walk to the bathroom and asking about safety rails pertain to mobility and safety issues, not to retention of urine. Taking certain medications may lead to urinary retention, but that information would establish the etiology. The question is asking for the nurse to first establish the correct diagnosis.

6. A febrile preschool-aged child presents to the after-hours clinic. Varicella (chickenpox) is diagnosed on the basis of the illness history and the presence of small, circumscribed skin lesions filled with serous fluid. Which type of skin lesion will the nurse report? a.Vesicles b.Wheals c.Papules d.Pustules

ANS: A Vesicles are circumscribed, elevated skin lesions filled with serous fluid that measure less than 1 cm. Wheals are irregularly shaped, elevated areas of superficial localized edema that vary in size. They are common with mosquito bites and hives. Papules are palpable, circumscribed, solid elevations in the skin that are smaller than 1 cm. Pustules are elevations of skin similar to vesicles, but they are filled with pus and vary in size like acne.

2. A nurse is providing nursing care to a group of patients. Which actions are direct care interventions? (Select all that apply.) a.Ambulating a patient b.Inserting a feeding tube c.Performing resuscitation d.Documenting wound care e.Teaching about medications

ANS: A, B, C, E All of the interventions listed (ambulating, inserting a feeding tube, performing resuscitation, and teaching) are direct care interventions involving patient and nurse interaction, except documenting wound care. Documenting wound care is an example of an indirect intervention.

4. Which interventions are appropriate for a patient with diabetes and poor wound healing? (Select all that apply.) a.Perform dressing changes twice a day as ordered. b.Teach the patient about signs and symptoms of infection. c.Instruct the family about how to perform dressing changes. d.Gently refocus patient from discussing body image changes. e.Administer medications to control the patient's blood sugar as ordered.

ANS: A, B, C, E Nursing priorities include interventions directed at enhancing wound healing. Teaching the patient about signs and symptoms of infection will help the patient identify signs of appropriate wound healing and know when the need for calling the health care provider arises. Performing dressing changes, controlling blood sugars through administration of medications, and instructing the family in dressing changes all contribute to wound healing. As long as a patient is stable and alert, it is appropriate to allow family to assist with care. The patient should be allowed to discuss body image changes.

20. A nurse is planning care for a patient with a nursing diagnosis of Impaired skin integrity. The patient needs many nursing interventions, including a dressing change, several intravenous antibiotics, and a walk. Which factors does the nurse consider when prioritizing interventions? (Select all that apply.) a.Rank all the patient's nursing diagnoses in order of priority. b.Do not change priorities once they've been established. c.Set priorities based solely on physiological factors. d.Consider time as an influencing factor. e.Utilize critical thinking.

ANS: A, D, E By ranking a patient's nursing diagnoses in order of importance and always monitoring changing signs and symptoms (defining characteristics) of patient problems, you attend to each patient's most important needs and better organize ongoing care activities. Prioritizing the problems, or nursing diagnoses, will help the nurse decide which problem to address first. Symptom pattern recognition from your assessment database and certain knowledge triggers help you understand which diagnoses require intervention and the associated time frame to intervene effectively. Planning requires critical thinking applied through deliberate decision making and problem solving. The nurse avoids setting priorities based solely on physiological factors; other factors should be considered as well. The order of priorities changes as a patient's condition and needs change, sometimes within a matter of minutes.

24. A teen patient is tearful and reports locating lumps in her breasts. Other history obtained is that she is currently menstruating. Physical examination reveals soft and movable cysts in both breasts that are painful to palpation. The nurse also notes that the patient's nipples are erect, but the areola is wrinkled. Which action will the nurse take after talking with the health care provider? a.Reassure patient that her symptoms are normal. b.Discuss the possibility of fibrocystic disease as the probable cause. c.Consult a breast surgeon because of the abnormal nipples and areola. d.Tell the patient that the symptoms may get worse when her period ends.

ANS: B A common benign condition of the breast is benign (fibrocystic) breast disease. This patient has symptoms of fibrocystic disease, which include bilateral lumpy, painful breasts sometimes accompanied by nipple discharge. Symptoms are more apparent during the menstrual period. When palpated, the cysts (lumps) are soft, well differentiated, and movable. Deep cysts feel hard. Although a common condition, benign breast disease is not normal; therefore, the nurse does not tell the patient that this is a normal finding. During examination of the nipples and areolae, the nipple sometimes becomes erect with wrinkling of the areola. Therefore, consulting a breast surgeon to treat her nipples and areolae is not appropriate.

21. A teen female patient reports intermittent abdominal pain for 12 hours. No dysuria is present. Which action will the nurse take when performing an abdominal assessment? a.Assess the area that is most tender first. b.Ask the patient about the color of her stools. c.Recommend that the patient take more laxatives. d.Avoid sexual references such as possible pregnancy.

ANS: B Abdominal pain can be related to bowels. If stools are black or tarry (melena), this may indicate gastrointestinal alteration. The nurse should caution patients about the dangers of excessive use of laxatives or enemas. There is not enough information about the abdominal pain to recommend laxatives. Determine if the patient is pregnant, and note her last menstrual period. Pregnancy causes changes in abdominal shape and contour. Assess painful areas last to minimize discomfort and anxiety.

18. The nurse is intervening for a family member with role strain. Which direct care nursing intervention is most appropriate? a.Assisting with activities of daily living b.Counseling about respite care options c.Teaching range-of-motion exercises d.Consulting with a social worker

ANS: B Family caregivers need assistance in adjusting to the physical and emotional demands of caregiving. Sometimes they need respite (i.e., a break from providing care). Counseling is an example of a direct care nursing intervention. The other options do not address the identified problem of role strain (activities of daily living and range-of-motion exercises). Consulting is an indirect care nursing intervention.

25. A nurse is performing a mental status examination and asks an adult patient what the statement "Don't cry over spilled milk" means. Which area is the nurse assessing? a.Long-term memory b.Abstract thinking c.Recent memory d.Knowledge

ANS: B For an individual to explain common phrases such as "A stitch in time saves nine" or "Don't cry over spilled milk" requires a higher level of intellectual function or abstract thinking. Knowledge-based assessment is factual. Assess knowledge by asking how much the patient knows about the illness or the reason for seeking health care. To assess past (long-term) memory, ask the patient to recall the maiden name of the patient's mother, a birthday, or a special date in history. It is best to ask open-ended questions rather than simple yes/no questions. Patients demonstrate immediate recall (recent memory) by repeating a series of numbers in the order in which they are presented or in reverse order.

10. A new nurse is working in a unit that uses interdisciplinary collaboration. Which action will the nurse take? a.Act as a leader of the health care team. b.Develop good communication skills. c.Work solely with nurses. d.Avoid conflict.

ANS: B Good communication between other health care providers builds trust and is related to the acceptance of your role in the health care team. As a beginning nurse, you will not be considered a leader of the health care team, but your input as an interdisciplinary team member is critical. Interdisciplinary involves other health care providers, not just nurses. Organizational culture includes leadership, communication processes, shared beliefs about the quality of clinical guidelines, and conflict resolution.

32. The nurse is assessing an adult patient's patellar reflex. Which finding will the nurse record as normal? a.1+ b.2+ c.3+ d.4+

ANS: B Grade reflexes as follows: 0: No response; 1+: Sluggish or diminished; 2+: Active or expected response; 3+: More brisk than expected, slightly hyperactive; and 4+: Brisk and hyperactive with intermittent or transient clonus.

13. A new nurse writes the following nursing diagnoses on a patient's care plan. Which nursing diagnosis will cause the nurse manager to intervene? a.Wandering b.Hemorrhage c.Urinary retention d.Impaired swallowing

ANS: B Hemorrhage is a collaborative problem, not a nursing diagnosis; the nurse manager will need to correct this misunderstanding with the new nurse. Nurses manage collaborative problems such as hemorrhage, infection, and paralysis using medical, nursing, and allied health (e.g., physical therapy) interventions. Wandering, urinary retention, and impaired swallowing are all examples of nursing diagnoses.

8. A patient visiting with family members in the waiting area tells the nurse "I don't feel good, especially in the stomach." What should the nurse do? a.Request that the family leave, so the patient can rest. b.Ask the patient to return to the room, so the nurse can inspect the abdomen. c.Ask the patient when the last bowel movement was and to lie down on the sofa. d.Tell the patient that the dinner tray will be ready in 15 minutes and that may help the stomach feel better.

ANS: B In this case, the environment needs to be conducive to completing a thorough assessment. A patient's care environment needs to be safe and conducive to implementing therapies. When you need to expose a patient's body parts, do so privately by closing room doors or curtains because the patient will then be more relaxed; the patient needs to return to the room for an abdominal assessment for privacy and comfort. The family can remain in the waiting area while the nurse assists the patient back to the room. Beginning the assessment in the waiting area (lie down on the sofa) in the presence of family and other visitors does not promote privacy and patient comfort. Telling the patient that the dinner tray is almost ready is making an assumption that the abdominal discomfort is due to not eating. The nurse needs to perform an assessment first.

14. The nurse is examining a female with vaginal discharge. Which position will the nurse place the patient for proper examination? a.Sitting b.Lithotomy c.Knee-chest d.Dorsal recumbent

ANS: B Lithotomy is the position for examination of female genitalia. The lithotomy position provides for the maximum exposure of genitalia and allows the insertion of a vaginal speculum. Sitting does not allow adequate access for speculum insertion and is better used to visualize upper body parts. Dorsal recumbent is used to examine the head and neck, anterior thorax and lungs, breasts, axillae, heart, and abdomen. Knee-chest provides maximal exposure of the rectal area but is embarrassing and uncomfortable.

1. A nurse is a preceptor for a nurse who just graduated from nursing school. When caring for a patient, the new graduate nurse begins to explain to the patient the purpose of completing a physical assessment. Which statement made by the new graduate nurse requires the preceptor to intervene? a."I will use the information from my assessment to figure out if your antihypertensive medication is working effectively." b."Nursing assessment data are used only to provide information about the effectiveness of your medical care." c."Nurses use data from their patient's physical assessment to determine a patient's educational needs." d."Information gained from physical assessment helps nurses better understand their patients' emotional needs."

ANS: B Nursing assessment data are used to evaluate the effectiveness of all aspects of a patient's care, not just the patient's medical care. Assessment data help to evaluate the effectiveness of medications and to determine a patient's health care needs, including the need for patient education. Nurses also use assessment data to identify patients' psychosocial and cultural needs.

15. The nurse establishes trust and talks with a school-aged patient before administering an injection. Which type of implementation skill is the nurse using? a.Cognitive b.Interpersonal c.Psychomotor d.Judgmental

ANS: B Nursing practice includes cognitive, interpersonal, and psychomotor skills. Interpersonal skills involve developing trusting relationships with patients, conveying caring and compassion, and communicating clearly. Cognitive skills include critical thinking and decision-making skills. Psychomotor skill requires the integration of cognitive and motor abilities, such as administering the injection. Being judgmental is not appropriate in nursing; nurses are nonjudgmental.

37. An older-adult patient is taking aminoglycoside for a severe infection. Which assessment is the priority? a.Eyes b.Ears c.Skin d.Reflexes

ANS: B Older adults are especially at risk for hearing loss caused by ototoxicity (injury to auditory nerve) resulting from high maintenance doses of antibiotics (e.g., aminoglycosides). While eyes and skin are important, they are not the priority. Reflexes are expected to be diminished in older adults.

17. Which action will the nurse take after the plan of care for a patient is developed? a.Place the original copy in the chart, so it cannot be tampered with or revised. b.Communicate the plan to all health care professionals involved in the patient's care. c.File the plan of care in the administration office for legal examination. d.Send the plan of care to quality assurance for review.

ANS: B Setting realistic goals and outcomes often means you must communicate these goals and outcomes to caregivers in other settings who will assume responsibility for patient care. The plan of care communicates nursing care priorities to nurses and other health care professionals. Know also that a plan of care is dynamic and changes as the patient's needs change. All health care professionals involved in the patient's care need to be informed of the plan of care. The plan of care is not sent to the administrative office or quality assurance office.

13. Which action indicates the nurse is using a PICOT question to improve care for a patient? a.Practices nursing based on the evidence presented in court b.Implements interventions based on scientific research c.Uses standardized care plans for all patients. d.Plans care based on tradition

ANS: B The best answer is implementing interventions based on scientific research. Using results of a literature search to a PICOT question can help a nurse decide which interventions to use. Practicing based on evidence presented in court is incorrect. Practice is based on current research. Using standardized care plans may be one example of evidence-based practice, but it is not used on all patients. The nurse must be careful in using standardized care plans to ensure that each patient's plan of care is still individualized. Planning care based on tradition is incorrect because nursing care should be based on current research.

18. An older-adult patient is being seen for chronic entropion. Which condition will the nurse assess for in this patient? a.Ptosis b.Infection c.Borborygmi d.Exophthalmos

ANS: B The diagnosis of entropion can lead to lashes of the lids irritating the conjunctiva and cornea. Irritation can lead to infection. Exophthalmos is a bulging of the eyes and usually indicates hyperthyroidism. An abnormal drooping of the lid over the pupil is called ptosis. In the older adult, ptosis results from a loss of elasticity that accompanies aging. Hyperactive sounds are loud, "growling" sounds called borborygmi, which indicate increased GI motility.

15. A nurse is completing a care plan. Which intervention is most appropriate for the nursing diagnostic statement Impaired skin integrity related to shearing forces? a.Administer pain medication every 4 hours as needed. b.Turn the patient every 2 hours, even hours. c.Monitor vital signs, especially rhythm. d.Keep the bed side rails up at all times.

ANS: B The most appropriate intervention for the diagnosis of Impaired skin integrity is to turn the patient. Choose interventions to alter the etiological (related to) factor or causes of the diagnosis. The other options do not directly address the shearing forces. The patient may need pain medication, but Acute pain would be another nursing diagnosis. Monitoring vital signs does not have when or how often these should be done. Keeping the side rails up addresses safety, not skin integrity.

18. Which question would be most appropriate for a nurse to ask a patient to assist in establishing a nursing diagnosis of Diarrhea? a."What types of foods do you think caused your upset stomach?" b."How many bowel movements a day have you had?" c."Are you able to get to the bathroom in time?" d."What medications are you currently taking?"

ANS: B The nurse needs to first ensure that the symptoms support the diagnosis. By definition, diarrhea means that a patient is having frequent stools; therefore, asking about the number of bowel movements is most appropriate. Asking about irritating foods and medications may help the nurse determine the cause of the diarrhea, but first the nurse needs to make sure the diagnosis is appropriate. Asking the patient if he can make it to the bathroom will help to establish a diagnosis of incontinence, not diarrhea. The question is asking for the most appropriate statement to establish the diagnosis of Diarrhea.

29. While assessing the skin of an 82-year-old patient, a nurse discovers nonpainful, ruby red papules on the patient's trunk. What is the nurse's next action? a.Explain that the patient has basal cell carcinoma and should watch for spread. b.Document cherry angiomas as a normal older adult skin finding. c.Tell the patient that this is a benign squamous cell carcinoma. d.Record the presence of petechiae.

ANS: B The skin is normally free of lesions, except for common freckles or age-related changes such as skin tags, senile keratosis (thickening of skin), cherry angiomas (ruby red papules), and atrophic warts. Basal cell carcinoma is most common in sun-exposed areas and frequently occurs in a background of sun-damaged skin; it almost never spreads to other parts of the body. Squamous cell carcinoma is more serious than basal cell and develops on the outer layers of sun-exposed skin; these cells may travel to lymph nodes and throughout the body. Report abnormal lesions to the health care provider for further examination. Petechiae are nonblanching, pinpoint-size, red or purple spots on the skin caused by small hemorrhages in the skin layers.

1. After assessing a patient, a nurse develops a standard formal nursing diagnosis. What is the rationale for the nurse's actions? a.To form a language that can be encoded only by nurses b.To distinguish the nurse's role from the physician's role c.To develop clinical judgment based on other's intuition d.To help nurses focus on the scope of medical practice

ANS: B The standard formal nursing diagnosis serves several purposes. Nursing diagnoses distinguish the nurse's role from that of the physician/health care provider and help nurses focus on the scope of nursing practice (not medical) while fostering the development of nursing knowledge. A nursing diagnosis provides the precise definition that gives all members of the health care team a common language for understanding the patient's needs. A diagnosis is a clinical judgment based on information.

11. The patient database reveals that a patient has decreased oral intake, decreased oxygen saturation when ambulating, reports of shortness of breath when getting out of bed, and a productive cough. Which elements will the nurse identify as defining characteristics for the diagnostic label of Activity intolerance? a.Decreased oral intake and decreased oxygen saturation when ambulating b.Decreased oxygen saturation when ambulating and reports of shortness of breath when getting out of bed c.Reports of shortness of breath when getting out of bed and a productive cough d.Productive cough and decreased oral intake

ANS: B There are defining characteristics (observable assessment cues such as patient behavior, physical signs) that support each problem-focused diagnostic judgment. The signs and symptoms, or defining characteristics, for the diagnosis Activity intolerance include decreased oxygen saturation when ambulating and reports of shortness of breath when getting out of bed. The key to supporting the diagnosis of Activity intolerance is that only these two characteristics involve how the patient tolerates activity. Decreased oral intake and productive cough do not define activity intolerance.

3. Which information indicates a nurse has a good understanding of a goal? a.It is a statement describing the patient's accomplishments without a time restriction. b.It is a realistic statement predicting any negative responses to treatments. c.It is a broad statement describing a desired change in a patient's behavior. d.It is a measurable change in a patient's physical state.

ANS: C A goal is a broad statement that describes a desired change in a patient's condition or behavior. A goal is mutually set with the patient. An expected outcome is the measurable changes (patient behavior, physical state, or perception) that must be achieved to reach a goal. Expected outcomes are time limited, measurable ways of determining if a goal is met.

8. A patient with a spinal cord injury is seeking to enhance urinary elimination abilities by learning self-catheterization versus assisted catheterization by home health nurses and family members. The nurse adds Readiness for enhanced urinary elimination in the care plan. Which type of diagnosis did the nurse write? a.Risk b.Problem focused c.Health promotion d.Collaborative problem

ANS: C A health promotion nursing diagnosis is a clinical judgment concerning motivation and desire to increase well-being and actualize human health potential. A problem-focused nursing diagnosis describes a clinical judgment concerning an undesirable human response to a health condition/life process that exists in an individual, family, or community. A risk nursing diagnosis is a clinical judgment concerning the vulnerability of an individual, family, group or community for developing an undesirable human response to health conditions/life processes. A collaborative problem is an actual or potential physiological complication that nurses monitor to detect the onset of changes in a patient's health status.

34. The nurse completed assessments on several patients. Which assessment finding will the nurse record as normal? a.Pulse strength 3 b.1+ pitting edema c.Constricting pupils when directly illuminated d.Hyperactive bowel sounds in all four quadrants

ANS: C A normal finding is pupils constricting when directly illuminated with a penlight. A pulse strength of 3 indicates a full or increased pulse; 2 is normal. 1+ pitting edema is abnormal; there should be no edema for a normal finding. Hyperactive bowel sounds are abnormal and indicate increased GI motility; normal bowel sounds are active.

10. A nurse adds the following diagnosis to a patient's care plan: Constipation related to decreased gastrointestinal motility secondary to pain medication administration as evidenced by the patient reporting no bowel movement in seven days, abdominal distention, and abdominal pain. Which element did the nurse write as the defining characteristic? a.Decreased gastrointestinal motility b.Pain medication c.Abdominal distention d.Constipation

ANS: C Abdominal distention, no reported bowel movement, and abdominal pain are the defining characteristics. Decreased gastrointestinal motility secondary to pain medication is an etiology or related to factor. Constipation (problem or NANDA-1 diagnosis) is the identified problem derived from the defining characteristics.

1. The nurse completes a thorough assessment of a patient and analyzes the data to identify nursing diagnoses. Which step will the nurse take next in the nursing process? a.Assessment b.Diagnosis c.Planning d.Implementation

ANS: C After identifying a patient's nursing diagnoses and collaborative problems, a nurse prioritizes the diagnoses, sets patient-centered goals and expected outcomes, and chooses nursing interventions appropriate for each diagnosis. This is the third step of the nursing process, planning. The assessment phase of the nursing process involves gathering data. The implementation phase involves carrying out appropriate nursing interventions. During the evaluation phase, the nurse assesses the achievement of goals and effectiveness of interventions.

7. The nurse is caring for a patient who requires a complex dressing change. While in the patient's room, the nurse decides to change the dressing. Which action will the nurse take just before changing the dressing? a.Gathers and organizes needed supplies b.Decides on goals and outcomes for the patient c.Assesses the patient's readiness for the procedure d.Calls for assistance from another nursing staff member

ANS: C Always be sure a patient is physically and psychologically ready for any interventions or procedures. After determining the patient's readiness for the dressing change, the nurse gathers needed supplies. The nurse establishes goals and outcomes before intervening. The nurse needs to ask another staff member to help if necessary after determining readiness of the patient.

6. A charge nurse is reviewing outcome statements using the SMART approach. Which patient outcome statement will the charge nurse praise to the new nurse? a.The patient will ambulate in hallways. b.The nurse will monitor the patient's heart rhythm continuously this shift. c.The patient will feed self at all mealtimes today without reports of shortness of breath. d.The nurse will administer pain medication every 4 hours to keep the patient free from discomfort.

ANS: C An expected outcome should be patient centered; should address one patient response; should be specific, measurable, attainable, realistic, and timed (SMART approach). The statement "The patient will feed self at all mealtimes today without reports of shortness of breath" includes all SMART criteria for goal writing. "The patient will ambulate in hallways" is missing a time limit. Administering pain medication and monitoring the patient's heart rhythm are nursing interventions; they do not reflect patient behaviors or actions.

6. A patient presents to the emergency department following a motor vehicle crash and suffers a right femur fracture. The leg is stabilized in a full leg cast. Otherwise, the patient has no other major injuries, is in good health, and reports only moderate discomfort. Which is the most pertinent nursing diagnosis the nurse will include in the plan of care? a.Posttrauma syndrome b.Constipation c.Acute pain d.Anxiety

ANS: C Based on the assessment data provided, the only supportive evidence for one of the diagnosis options is "Reports only moderate discomfort," which would support Acute pain. No supportive evidence is provided for any of the other diagnoses. The patient may indeed develop signs or symptoms of the other problems, but supportive data are presently lacking in the provided information.

9. A newly admitted patient who is morbidly obese asks the nurse for assistance to the bathroom for the first time. Which action should the nurse take initially? a.Ask for at least two other assistive personnel to come to the room. b.Medicate the patient to alleviate discomfort while ambulating. c.Review the patient's activity orders. d.Offer the patient a walker.

ANS: C Before beginning care, review the plan to determine the need for assistance and the type required. Before intervening, the nurse must check the patient's orders. For example, if the patient is on bed rest, the nurse will need to explain the use of a bedpan rather than helping the patient get out of bed to go to the bathroom. Asking for assistive personnel is appropriate after making sure the patient can get out of bed. If the patient is obese, the nurse will likely need assistance in getting the patient to the bathroom. Medicating the patient before checking the orders is not advised in this situation. Before medicating for pain, the nurse needs to perform a pain assessment. Offering the patient a walker is a premature intervention until the orders are verified.

12. Vital signs for a patient reveal a high blood pressure of 187/100. Orders state to notify the health care provider for diastolic blood pressure greater than 90. What is the nurse's first action? a.Follow the clinical protocol for a stroke. b.Review the most recent lab results for the patient's potassium level. c.Assess the patient for other symptoms or problems, and then notify the health care provider. d.Administer an antihypertensive medication from the stock supply, and then notify the health care provider.

ANS: C Communication to other health care professionals must be timely, accurate, and relevant to a patient's clinical situation. The best answer is to reassess the patient for other symptoms or problems, and then notify the health care provider according to the orders. Reviewing the potassium level does not address the problem of high blood pressure. The nurse does not follow the protocol since the order says to notify the health care provider. The orders read to notify the health care provider, not administer medications.

16. A charge nurse is evaluating a new nurse's plan of care. Which finding will cause the charge nurse to follow up? a.Assigning a documented nursing diagnosis of Risk for infection for a patient on intravenous (IV) antibiotics b.Completing an interview and physical examination before adding a nursing diagnosis c.Developing nursing diagnoses before completing the database d.Including cultural and religious preferences in the database

ANS: C Developing nursing diagnoses before completion of the database needs to be corrected by the charge nurse. Always identify a nursing diagnosis from the data, not the reverse. The data should be clustered and reviewed to see if any patterns are present before a nursing diagnosis is assigned. Risk for infection is an appropriate diagnosis for a patient with an intravenous (IV) site in place. The IV site involves a break in skin integrity and is a potential source of infection. The diagnostic process should proceed in steps. Completing the interview and physical examination before adding a nursing diagnosis is appropriate. The patient's cultural background and developmental stage are important to include in a patient database.

5. A nurse is using assessment data gathered about a patient and combining critical thinking to develop a nursing diagnosis. What is the nurse doing? a.Assigning clinical cues b.Defining characteristics c.Diagnostic reasoning d.Diagnostic labeling

ANS: C Diagnostic reasoning is defined as a process of using the assessment data gathered about a patient to logically explain a clinical judgment, in this case a nursing diagnosis. Defining characteristics are assessment findings that support the nursing diagnosis. Defining characteristics are the subjective and objective clinical cues, which a nurse gathers intentionally and unintentionally. The nurse organizes all of the patient's data into meaningful and usable data clusters, which lead to a diagnostic conclusion. Diagnostic labeling is simply the name of the diagnosis.

27. The nurse considers several new female patients to receive additional teaching on the need for more frequent Pap test and gynecological examinations. Which assessment findings reveal the patient at highest risk for cervical cancer and having the greatest need for patient education? a.13 years old, nonsmoker, not sexually active b.15 years old, social smoker, celibate c.22 years old, smokes 1 pack of cigarettes per day, has multiple sexual partners d.50 years old, stopped smoking 30 years ago, has history of multiple pregnancies

ANS: C Females considered to be at higher risk include those who smoke, have multiple sex partners, and have a history of sexually transmitted infections. Of all the assessment findings listed, the 22-year-old smoker with multiple sexual partners has the greatest number of risk factors for cervical cancer. The other patients are at lower risk: not sexually active, celibate, and do not smoke.

11. During a sexually transmitted illness presentation to high-school students, the nurse recommends the human papillomavirus (HPV) vaccine series. Which condition is the nurse trying to prevent? a.Breast cancer b.Ovarian cancer c.Cervical cancer d.Testicular cancer

ANS: C Human papillomavirus (HPV) infection increases the person's risk for cervical cancer. HPV vaccine is recommended for females aged 11 to 12 years but can be given to females ages 12 through 26; males can also receive the vaccine. HPV is not a risk factor for breast, ovarian, and testicular cancer.

14. A patient has a bacterial infection in left lower leg. Which nursing diagnosis will the nurse add to the patient's care plan? a.Infection b.Risk for infection c.Impaired skin integrity d.Staphylococcal leg infection

ANS: C Impaired skin integrity is the only nursing diagnosis listed that will correlate to the patient information. While risk for infection is a nursing diagnosis, the patient is not at risk; the patient has an actual infection. Infection can be a medical diagnosis as well as a collaborative problem. Staphylococcal leg infection is a medical diagnosis.

1. A nurse is providing nursing care to patients after completing a care plan from nursing diagnoses. In which step of the nursing process is the nurse? a.Assessment b.Planning c.Implementation d.Evaluation

ANS: C Implementation, the fourth step of the nursing process, formally begins after a nurse develops a plan of care. With a care plan based on clear and relevant nursing diagnoses, a nurse initiates interventions that are designed to assist the patient in achieving the goals and expected outcomes needed to support or improve the patient's health status. The nurse gathers data during the assessment phase and mutually sets goals and prioritizes care during the planning phase. During the evaluation phase, the nurse determines the achievement of goals and effectiveness of interventions.

16. The nurse inserts an intravenous (IV) catheter using the correct technique and following the recommended steps according to standards of care and hospital policy. Which type of implementation skill is the nurse using? a.Cognitive b.Interpersonal c.Psychomotor d.Judgmental

ANS: C Nursing practice includes cognitive, interpersonal, and psychomotor skills. Psychomotor skill requires the integration of cognitive and motor abilities. The nurse in this example displayed the psychomotor skill of inserting an intravenous catheter while following standards of care and integrating knowledge of anatomy and physiology. Cognitive involves the application of critical thinking and use of good judgment in making sound clinical decisions. Interpersonal skills involve developing trusting relationships with patients, conveying caring and compassion, and communicating clearly.

17. An advanced practice nurse is preparing to assess the external genitalia of a 25-year-old American woman of Chinese descent. Which action will the nurse do first? a.Place the patient in the lithotomy position. b.Drape the patient to enhance patient comfort. c.Assess the patient's feelings about the examination. d.Ask the patient if she would like her mother to be present in the room.

ANS: C Patients who are Chinese American often believe that examination of the external genitalia is offensive. Before proceeding with the examination, the nurse first determines how the patient feels about the procedure and explains the procedure to answer any questions and to help the patient feel comfortable with the assessment. Once the patient is ready to have her external genitalia examined, the nurse places the patient in the lithotomy position and drapes the patient appropriately. Typically, nurses ask adolescents if they want a parent present during the examination. The patient in this question is 25 years old; asking if she would like her mother to be present is inappropriate.

5. A nurse is reviewing a patient's care plan. Which information will the nurse identify as a nursing intervention? a.The patient will ambulate in the hallway twice this shift using crutches correctly. b.Impaired physical mobility related to inability to bear weight on right leg. c.Provide assistance while the patient walks in the hallway twice this shift with crutches. d.The patient is unable to bear weight on right lower extremity.

ANS: C Providing assistance to a patient who is ambulating is a nursing intervention. The statement, "The patient will ambulate in the hallway twice this shift using crutches correctly" is a patient outcome. Impaired physical mobility is a nursing diagnosis. The statement that the patient is unable to bear weight and ambulate can be included with assessment data and is a defining characteristic for the diagnosis of Impaired physical mobility.

3. A nurse is preparing to perform a complete physical examination on a weak, older-adult patient with bilateral basilar pneumonia. Which position will the nurse use? a.Prone b.Sims' c.Supine d.Lateral recumbent

ANS: C Supine is the most normally relaxed position. If the patient becomes short of breath easily, raise the head of the bed. Supine position would be easiest for a weak, older-adult person during the examination. Lateral recumbent and prone positions cause respiratory difficulty for any patient with respiratory difficulties. Sims' position is used for assessment of the rectum and the vagina.

31. A nurse is auscultating different areas on an adult patient. Which technique should the nurse use during an assessment? a.Uses the bell to listen for lung sounds b.Uses the diaphragm to listen for bruits c.Uses the diaphragm to listen for bowel sounds d.Uses the bell to listen for high-pitched murmurs

ANS: C The bell is best for hearing low-pitched sounds such as vascular (bruits) and certain heart sounds (low-pitched murmurs), and the diaphragm is best for listening to high-pitched sounds such as bowel and lung sounds and high-pitched murmurs.

14. A nurse is making initial rounds on patients. Which intervention for a patient with poor wound healing should the nurse perform first? a.Reinforce the wound dressing as needed with 4 × 4 gauze. b.Perform the ordered dressing change twice daily. c.Observe wound appearance and edges. d.Document wound characteristics.

ANS: C The most appropriate initial intervention is to assess the wound (observe wound appearance and edges). The nurse must assess the wound first before the findings can be documented, reinforcement of the dressing, and the actual skill of dressing changes.

17. A patient exhibits the following symptoms: tachycardia, increased thirst, headache, decreased urine output, and increased body temperature. The nurse analyzes the data. Which nursing diagnosis will the nurse assign to the patient? a.Adult failure to thrive b.Hypothermia c.Deficient fluid volume d.Nausea

ANS: C The signs the patient is exhibiting are consistent with deficient fluid volume (dehydration). Even without knowing the clinical manifestations of dehydration, the question can be answered by the process of elimination. Adult failure to thrive, hypothermia, and nausea are not appropriate diagnoses because data are insufficient to support these diagnoses.

38. The patient has had a stroke that has affected the ability to speak. The patient becomes extremely frustrated when trying to speak. The patient responds correctly to questions and instructions but cannot form words coherently. Which type of aphasia is the patient experiencing? a.Sensory b.Receptive c.Expressive d.Combination

ANS: C The two types of aphasias are sensory (or receptive) and motor (or expressive). The patient cannot form words coherently, indicating expressive or motor aphasia is present. The patient responds correctly to questions and instructions, indicating receptive or sensory aphasia is not present. Patients sometimes suffer a combination of receptive and expressive aphasia, but this is not the case here.

15. On admission, a patient weighs 250 pounds. The weight is recorded as 256 pounds on the second inpatient day. Which condition will the nurse assess for in this patient? a.Anorexia b.Weight loss c.Fluid retention d.Increased nutritional intake

ANS: C This patient has gained 6 pounds in a 24-hour period. A weight gain of 5 pounds (2.3 kg) or more in a day indicates fluid retention problems, not nutritional intake. A weight loss is considered significant if the patient has lost more than 5% of body weight in a month or 10% in 6 months. A downward trend may indicate a reduction in nutritional reserves that may be caused by decreased intake such as anorexia.

36. The nurse is assessing skin turgor. Which technique will the nurse use? a.Press lightly on the forearm. b.Press lightly on the fingertips. c.Grasp a fold of skin on the sternal area. d.Grasp a fold of skin on the back of the hand.

ANS: C To assess skin turgor, grasp a fold of skin on the back of the forearm or sternal area with the fingertips and release. Since the skin on the back of the hand is normally loose and thin, turgor is not reliably assessed at that site. Pressing lightly on the forearm can be used to assess for pitting edema or pain or sense of touch. Pressing lightly on the fingertips and observing nail color is assessing capillary refill.

39. The nurse is assessing the tympanic membranes of an infant. Which action by the nurse demonstrates proper technique? a.Pulls the auricle upward and backward. b.Holds handle of the otoscope between the thumb and little finger. c.Uses an inverted otoscope grip while pulling the auricle downward and back. d.Places the handle of the otoscope between the thumb and index finger while pulling the auricle upward.

ANS: C Using the inverted otoscope grip while pulling the auricle downward and back is a common approach with infant/child examinations because it prevents accidental movement of the otoscope deeper into the ear canal, as could occur with an unexpected pediatric reaction to the ear examination. The other techniques could result in injury to the infant's tympanic membrane. Insert the scope while pulling the auricle upward and backward in the adult and older child. Hold the handle of the otoscope in the space between the thumb and index finger, supported on the middle finger.

35. The patient presents to the clinic with dysuria and hematuria. How does the nurse proceed to assess for kidney inflammation? a.Uses deep palpation posteriorly. b.Lightly palpates each abdominal quadrant. c.Percusses posteriorly the costovertebral angle at the scapular line. d.Inspects abdomen for abnormal movement or shadows using indirect lighting.

ANS: C With the patient sitting or standing erect, use direct or indirect percussion to assess for kidney inflammation. With the ulnar surface of the partially closed fist, percuss posteriorly the costovertebral angle at the scapular line. If the kidneys are inflamed, the patient feels tenderness during percussion. Use a systematic palpation approach for each quadrant of the abdomen to assess for muscular resistance, distention, abdominal tenderness, and superficial organs or masses. Light palpation would not detect kidney tenderness because the kidneys sit deep within the abdominal cavity. Posteriorly, the lower ribs and heavy back muscles protect the kidneys, so they cannot be palpated. Kidney inflammation will not cause abdominal movement. However, to inspect the abdomen for abnormal movement or shadows, the nurse should stand on the patient's right side and inspect from above the abdomen using direct light over the abdomen.

21. A nurse is developing nursing diagnoses for a group of patients. Which nursing diagnoses will the nurse use? (Select all that apply.) a.Anxiety related to barium enema b.Impaired gas exchange related to asthma c.Impaired physical mobility related to incisional pain d.Nausea related to adverse effect of cancer medication e.Risk for falls related to nursing assistive personnel leaving bedrail down

ANS: C, D Impaired physical mobility and Nausea are the only correctly written nursing diagnoses. All the rest are incorrectly written. Anxiety lists a diagnostic test as the etiology. Impaired gas exchange lists a medical diagnosis as the etiology. Risk for falls has a legally inadvisable statement for an etiology.

1. A nurse is implementing interventions for a group of patients. Which actions are nursing interventions? (Select all that apply.) a.Order chest x-ray for suspected arm fracture. b.Prescribe antibiotics for a wound infection. c.Reposition a patient who is on bed rest. d.Teach a patient preoperative exercises. e.Transfer a patient to another hospital unit.

ANS: C, D, E A nursing intervention is any treatment based on clinical judgment and knowledge that a nurse performs to enhance patient outcomes. Repositioning, teaching, and transferring a patient are examples of nursing interventions. Ordering a chest x-ray and prescribing antibiotics are examples of medical interventions performed by a health care provider.

21. A nurse is teaching the staff about the benefits of Nursing Outcomes Classification. Which information should the nurse include in the teaching session? (Select all that apply.) a.Includes seven domains for level 1 b.Uses an easy 3-point Likert scale c.Adds objectivity to judging a patient's progress d.Allows choice in which interventions to choose e.Measures nursing care on a national and international level

ANS: C, E Nursing Outcomes Classification (NOC) links outcomes to NANDA International nursing diagnoses. Such a rating system adds objectivity to judging a patient's progress. Using standardized nursing terminologies such as NOC makes it more possible to measure aspects of nursing care on a national and international level. The indicators for each NOC outcome allow measurement of the outcomes at any point on a 5-point Likert scale from most negative to most positive. This resource is an option you can use in selecting goals and outcomes (not interventions) for your patients. The Nursing Interventions Classification model includes three levels: domains, classes, and interventions for ease of use. The seven domains are the highest level (level 1) of the model, using broad terms (e.g., safety and basic physiological) to organize the more specific classes and interventions.

11. Which action should the nurse take first during the initial phase of implementation? a.Determine patient outcomes and goals. b.Prioritize patient's nursing diagnoses. c.Evaluate interventions. d.Reassess the patient.

ANS: D Assessment is a continuous process that occurs each time the nurse interacts with a patient. During the initial phase of implementation, reassess the patient. Determining the patient's goals and prioritizing diagnoses take place in the planning phase before choosing interventions. Evaluation is the last step of the nursing process.

2. Having misplaced a stethoscope, a nurse borrows a colleague's stethoscope. The nurse next enters the patient's room and identifies self, washes hands with soap, and states the purpose of the visit. The nurse performs proper identification of the patient before auscultating the patient's lungs. Which critical health assessment step should the nurse have performed? a.Running warm water over stethoscope b.Draping stethoscope around the neck c.Rubbing stethoscope with betadine d.Cleaning stethoscope with alcohol

ANS: D Bacteria and viruses can be transferred from patient to patient when a stethoscope that is not clean is used. The stethoscope should be cleaned before use on each patient with isopropyl alcohol. Running water over the stethoscope does not kill bacteria. Betadine is an inappropriate cleaning solution and may damage the equipment. Draping the stethoscope around the neck is not advised.

10. During a routine pediatric history and physical, the parents report that their child was a very small, premature infant that had to stay in the neonatal intensive care unit longer than usual. They state that the infant was yellow when born and developed an infection that required "every antibiotic under the sun" to reach a cure. Which exam is a priority for the nurse to conduct on the child? a.Cardiac b.Respiratory c.Ophthalmic d.Hearing acuity

ANS: D Hearing is the priority. Risk factors for hearing problems include low birth weight, nonbacterial intrauterine infection, and excessively high bilirubin levels. Hearing loss due to ototoxicity (injury to auditory nerves) can result from high maintenance doses of antibiotics. Cardiac, respiratory, and eye examinations are important assessments but are not relevant to this child's condition.

13. The nurse is urgently called to the gymnasium regarding an injured student. The student is crying in severe pain with a malformed fractured lower leg. Which proper sequence will the nurse follow to perform the initial assessment? a.Light palpation, deep palpation, and inspection b.Inspection, light palpation, and deep palpation c.Auscultation and light palpation d.Inspection and light palpation

ANS: D Inspection is the use of vision and hearing to distinguish normal from abnormal findings. Light palpation determines areas of tenderness and skin temperature, moisture, and texture. Deep palpation is used to examine the condition of organs, such as those in the abdomen. Caution is the rule with deep palpation. Deep palpation is performed after light palpation; however, deep palpation is not performed on a fractured leg. Auscultation is used to evaluate sound and is not used to assess a fractured leg.

2. A patient's plan of care includes the goal of increasing mobility this shift. As the patient is ambulating to the bathroom at the beginning of the shift, the patient suffers a fall. Which initial action will the nurse take next to revise the plan of care? a.Consult physical therapy. b.Establish a new plan of care. c.Set new priorities for the patient. d.Assess the patient.

ANS: D Nurses revise a plan when a patient's status changes; assessment is the first step. Know also that a plan of care is dynamic and changes as the patient's needs change. Asking physical therapy to assist the patient is premature before assessing the patient and awaiting the health care provider's orders. The nurse may not need to disregard all previous diagnoses. Some diagnoses may still apply, but the patient needs to be assessed first. Setting new priorities is not recommended before assessment and establishing diagnoses.

7. A nursing assessment for a patient with a spinal cord injury leads to several pertinent nursing diagnoses. Which nursing diagnosis is the highest priority for this patient? a.Risk for impaired skin integrity b.Risk for infection c.Spiritual distress d.Reflex urinary incontinence

ANS: D Reflex urinary incontinence is highest priority. If a patient's incontinence is not addressed, then the patient is at higher risk of impaired skin integrity and infection. Remember that the Risk for diagnoses are potential problems. They may be prioritized higher in some cases but not in this situation. Spiritual distress is an actual diagnosis, but the adverse effects that could result from not assisting the patient with urinary elimination take priority in this case.

16. A patient has reduced muscle strength following a left-sided stroke and is at risk for falling. Which intervention is most appropriate for the nursing diagnostic statement Risk for falls? a.Keep all side rails down at all times. b.Encourage patient to remain in bed most of the shift. c.Place patient in room away from the nurses' station if possible. d.Assist patient into and out of bed every 4 hours or as tolerated.

ANS: D Risk for falls is a risk (potential) nursing diagnosis; therefore, the nurse needs to implement actions that will prevent a fall. Assisting the patient into and out of bed is the most appropriate intervention to prevent the patient from falling. Encouraging activity builds muscle strength, and helping the patient with transfers ensures patient safety. Encouraging the patient to stay in bed will not promote muscle strength. Decreased muscle strength is the risk factor placing the patient in jeopardy of falling. The side rails should be up, not down, according to agency policy. This will remind the patient to ask for help to get up and will keep the patient from rolling out of bed. The patient should be placed near the nurses' station, so a staff member can quickly get to the room and assist the patient if necessary.

9. A patient's son decides to stay at the bedside while his father is confused. When developing the plan of care for this patient, what should the nurse do? a.individualize the care plan only according to the patient's needs. b.Request that the son leave at bedtime, so the patient can rest. c.Suggest that a female member of the family stay with the patient. d.Involve the son in the plan of care as much as possible.

ANS: D The family is often a resource to help the patient meet health care goals. Family should be included in the plan of care as much as possible. Meeting some of the family's needs as well as the patient's needs will possibly improve the patient's level of wellness. The son should not be asked to leave if at all possible. In some situations, it may be best that family members not remain in the room, but no evidence in the question stem suggests that this is the case in this situation. The suggestion of asking a female member to stay is not a justified action without a legitimate reason. No reason is given in this question stem for such a suggestion.

12. A registered nurse administers pain medication to a patient suffering from fractured ribs. Which type of nursing intervention is this nurse implementing? a.Collaborative b.Independent c.Interdependent d.Dependent

ANS: D The nurse does not have prescriptive authority to order pain medications, unless the nurse is an advanced practice nurse. The intervention is therefore dependent. Administering a medication, implementing an invasive procedure (e.g., inserting a Foley catheter, starting an intravenous [IV] infusion), and preparing a patient for diagnostic tests are examples of health care provider-initiated interventions. A collaborative, or an interdependent, intervention involves therapies that require combined knowledge, skill, and expertise from multiple health care professionals. Nurse-initiated interventions are the independent nursing interventions, or actions that a nurse initiates without supervision or direction from others.

33. A patient in the emergency department is reporting left lower abdominal pain. Which proper order will the nurse follow to perform the comprehensive abdominal examination? a.Percussion, palpation, auscultation b.Percussion, auscultation, palpation c.Inspection, palpation, auscultation d.Inspection, auscultation, palpation

ANS: D The order of an abdominal examination differs slightly from that of other assessments. Begin with inspection and follow with auscultation. By using auscultation before palpation, the chance of altering the frequency and character of bowel sounds is lessened.

6. A patient recovering from a leg fracture after a fall reports having dull pain in the affected leg and rates it as a 7 on a 0 to 10 scale. The patient is not able to walk around in the room with crutches because of leg discomfort. Which nursing intervention is priority? a.Assist the patient to walk in the room with crutches. b.Obtain a walker for the patient. c.Consult physical therapy. d.Administer pain medication.

ANS: D The patient's pain is a 7, indicating the priority is pain relief (administer pain medication). Acute pain is the priority because the nurse can address the problem of immobility after the patient receives adequate pain relief. Assisting the patient to walk or obtaining a walker will not address the pain the patient is experiencing.

20. Upon assessment, the patient is breathing normally and has normal vesicular lung sounds. Which expected inspiratory-to-expiratory breath sounds will the nurse hear? a.The expiration phase is longer than the inspiration phase. b.The inspiratory phase lasts exactly as long as the expiratory phase. c.The expiration phase is 2 times longer than the inspiration phase. d.The inspiratory phase is 3 times longer than the expiratory phase.

ANS: D Vesicular breath sounds are normal breath sounds; the inspiratory phase is 3 times longer than the expiratory phase. Bronchovesicular breath sounds have an inspiratory phase equal to the expiratory phase. Bronchial breath sounds have an expiration phase longer than the inspiration phase at a 3:2 ratio.

8. The new nurse is caring for six patients in this shift. After completing their assessments, the nurse asks where to begin in developing care plans for these patients. Which statement is an appropriate suggestion by another nurse? a."Choose all the interventions and perform them in order of time needed for each one." b."Make sure you identify the scientific rationale for each intervention first." c."Decide on goals and outcomes you have chosen for the patients." d."Begin with the highest priority diagnoses, then select appropriate interventions."

ANS: D Work from your plan of care and use patients' priorities to organize the order for delivering interventions and organizing documentation of care. When developing a plan of care, the nurse needs to rank the nursing diagnoses in order of priority, then select appropriate interventions. Choosing all the interventions should take place after ranking of the diagnoses, and interventions should be prioritized by patient needs, not just by time. The chosen interventions should be evidence based with scientific rationales, but the diagnoses need to be prioritized first to prioritize interventions. Goals for a patient should be mutually set, not just chosen by the nurse.

What are the three types of Nursing Diagnosis?

Actual ND Risk ND Health Promotion ND

Collaborative Problem

Actual/ Potential complications that nurse monitors. When developed, nurse collaborates with other health care professional

Mr. Bagley is placed on Isolation Precautions. Isolation Precautions as a treatment intervention are an example of which type of care? A. Direct B. Indirect C. Prevention D. Safety

Answer: B Rationale: Indirect care interventions are treatments performed away from the patient but on behalf of the patient or group of patients. Implementing Isolation Precautions is an example of providing indirect care by managing the patient's environment.

2. You are writing a care plan for a newly admitted patient. Which one of these outcome statements is written correctly? A. The patient will eat 80% of all meals. B. The nursing assistant will set the patient up for a bath every day. C. The patient will have improved airway clearance by June 5. D. The patient will identify the need to increase dietary intake of fiber by June 5.

Answer: D

John knows that a ______________ diagnosis is applied to vulnerable populations.

Answer: risk nursing Rationale: A risk nursing diagnosis describes human responses to health conditions or life processes that may develop in a vulnerable individual, family, or community.

Mr. Bagley's plan of care calls for oxygen therapy to improve his respiratory status. A preprinted document that contains orders for the conduct of routine therapies, such as oxygen therapy, is referred to as a __________ _____________.

Answer: standing order Rationale: A standing order is a preprinted document that contains orders for the conduct of routine therapies, monitoring guidelines, and/or diagnostic procedures for specific patients with identified clinical problems.

Because of John's interest in cardiac nursing, he is familiar with the clinical criteria for heart disease. Which of the following is an example of a clinical criterion? (Select all that apply.) Hypertension Fatigue Food preference High cholesterol

Answers: Hypertension, fatigue, and high cholesterol Rationale: Clinical criteria consist of objective or subjective signs and symptoms or risk factors that lead to a diagnostic conclusion. Hypertension, fatigue, and high cholesterol are all clinical criteria for heart disease, whereas food preference is not.

1. A patient is suffering from shortness of breath. The correct goal statement would be written as: A. the patient will be comfortable by the morning. B. the patient will breath unlabored at 14 to 18 breaths per minute by the end of the shift. C. the patient will not complain of breathing problems within the next 8 hours. D. the patient will have a respiratory rate of 14 to 18 breaths per minute.

B

DJ is placed on isolation precautions. Isolation precautions as a treatment intervention are an example of which type of care? a. Direct b. Indirect c. Prevention d. Safety

B

The nurse determines the patient outcomes for DJ based on his reaction to the medication regimen. Which of the following is an end result that translates into observable patient behaviors that are measurable and desirable? a. Unexpected Outcomes b. Expected Outcomes c. Sensitive Outcomes d. Accomplished Outcomes

B

Your patient has met the goals set for improvement of ambulatory status. You would now a. Modify the care plan b. Discontinue the care plan c. Create a new nursing diagnosis that states goals have been met d. reassesses the patient's response to care and evaluate the implantation step of the nursing process

B

The nursing diagnosis: Impaired Parenting related to mother's developmental delay is an example of a(n): A) Risk Nursing Diagnosis B) Problem-Focused Nursing Diagnosis C) Health Promotion Nursing Diagnosis D) Wellness Nursing diagnosis

B) Problem-Focused Nursing Diagnosis This is an example of a problem-focused nursing diagnosis with a related factor, based on NANDA-I diagnostic terminology. Most health promotion diagnoses do not have established related factors based on NANDA-I; their use is optional. Wellness diagnoses are not one of the types of NANDA-I diagnoses.

A 56-year-old normally healthy patient at the clinic is diagnosed with bacterial community-acquired pneumonia. Before treatment is prescribed, the nurse asks the patient about an allergy to A) amoxicillin B) erythromycin C) sulfonamides D) cephalosporins

B) erythromycin Rationale: Outpatient drug therapy options for a healthy person with community-acquired pneumonia will be consist of macrolides (erythromycin) or doxycycline. If the patient is allergic to macrolides, doxycycline would be prescribed.

A patient is suffering from shortness of breath. The correct goal statement would be written as: A. the patient will be comfortable by the morning. B. the patient will breath unlabored at 14 to 18 breaths per minute by the end of shift. C. the patient will not complain of breathing problems within the next 8 hours. D. the patient will have a respiratory rate of 14 to 18 breaths per minute.

B. the patient will breath unlabored at 14 to 18 breaths per minute by the end of the shift.

Clinical Manifestations of COPD

Barrel chest Tripod position Dyspnea Mucus hypersecretion (gray, white or yellow) Hyperinflation of lungs

What does the nurse teach patients, as a high priority, when discussing diabetes management practices related to the increased risk of the most common cause of death?

Blood pressure control is priority to reduce the risk of an MI Diabetes and blood glucose MUST be controlled!

Consultation occurs most often during which phase of the nursing process? a. Assessment b. Diagnosis c. Planning d. Evaluation

C

A nursing student reports to a lead charge nurse that his assigned patient seems to be less alert and his blood pressure is lower, dropping from 140/80 to 110/60. The nursing student states, "I believe this is a nursing diagnosis of Deficient Fluid Volume." The lead charge nurse immediately goes to the patient's room with the student to assess the patient's orientation, heart rate, skin turgor, and urine output for last 8 hours. The lead charge nurse suspects that the student has made which type of diagnostic error? A) Insufficient cluster of cues B) Disorganization C) Insufficient number of cues D) Evidence that another diagnosis is more likely

C) Insufficient NUMBER of cues!! It is likely the charge nurse suspects that the student has not collected enough cues to support the diagnosis. A change in blood pressure and mental status changes are significant findings that can be attributed to fluid volume excess and other diagnoses. The recommendation of the symptom cluster by the registered nurse would allow the student to have sufficient data to confirm a deficient fluid volume.

A nurse in a mother-baby clinic learns that a 16-year-old has given birth to her first child and has not been to a well-baby class yet. The nurse's assessment reveals that the infant cries when breastfeeding and has difficulty latching on to the nipple. The infant has not gained weight over the last 2 weeks. The nurse identifies the patient's nursing diagnosis as Ineffective Breastfeeding. Which of the following is the best "related to" factor? A) Infant crying at breast B) Infant unable to latch onto the breast correctly C) Mother's deficient knowledge D) lack of infant weight gain

C) Mother's deficit knowledge!!!! In this scenario the related factor is the mother's deficient knowledge. A related factor is a condition, historical factor, or etiology that gives a context for the defining characteristics, in this case the infant crying, inability to latch on to breast, and absent weight gain.

3. Consultation occurs most often during which phase of the nursing process? A. Assessment B. Diagnosis C. Planning D. Evaluation

C: Planning

What are the signs and symptoms of Thyroid Storm?

Cardio-tachyarrhythmia, chest pain, CHF, pulmonary edema, shortness of breath Neuro-agitated, confusion, psychotic; seizures GI-increased vomiting/diarrhea; hypotension and shock Untreated hyperthyroidism or stressor/trigger Life-threatening Rapid increase in metabolic rate requiring Rapid treatment

A nurse administers an antihypertensive medication according to the standardized plan of care for a client admitted with uncontrolled hypertension. Which assessment information indicates the expected client outcome has been met within the first 24 hours? Client reports no headache. Client is drowsy after lunch. Client is normotensive. Client lipids are within range.

Client is normotensive. Explanation: A specific, expected client outcome is written for each day in a collaborative plan of care. An expected client outcome after 24 hours of treatment for hypertension is to have the blood pressure return to the expected range of between 90/60 and 120/80 mm Hg. The other options do not directly indicate successful control of hypertension.

Which outcome is sufficiently measurable? -Client will tolerate a full fluid diet with no reports of nausea by 12/15/2020. -Client will progress from clear fluid diet to full fluid diet without experiencing nausea. -Increase client's diet from clear fluids to full fluids by 12/15/2020. -Client will maintain adequate intake with no reports of nausea by 12/15/2020.

Client will tolerate a full fluid diet with no reports of nausea by 12/15/2020. Explanation: A fully measurable outcome should include a subject, verb, conditions, performance criteria, and target time (though not every outcome requires each parameter). Only the outcome "Client will tolerate a full fluid diet with no reports of nausea by 12/15/2020" includes all appropriate components. The outcome "Client will progress from clear fluid diet to full fluid diet without experiencing nausea" lacks a target time. The outcome "Increase client's diet from clear fluids to full fluids by 12/15/2016" expresses the outcome as a nursing intervention. The outcome "Client will maintain adequate intake with no reports of nausea by 12/15/2016" does not define the performance criteria sufficiently, because "adequate intake" is an imprecise term.

Validation

Comparison of data with another source to determine data accuracy

2. When caring for a patient who has multiple health problems and related medical diagnoses, nurses can best perform nursing diagnoses and nursing interventions by developing a: A. critical pathway. B. nursing care plan. C. concept map. D. diagnostic label.

Concept map?

Related Factors

Condition that gives content for the defining characteristics and shows relation with the nursing diagnosis

Which of the following nursing diagnoses is stated correctly? (Select all that apply.) a. Fluid Volume excess related to heart failure b. Sleep deprivation related to sustained noisy environment c. Impaired bed mobility related to post cardiac cauterization d. Ineffective Protection related to inadequate nutrition e. Diarrhea related to frequent, small, watery stools.

Correctly stated: B) Sleep deprivation related to sustained noisy environment D) Diarrhea related to frequent, small, watery stools. The correct diagnoses of Sleep Deprivation and Ineffective Protection are worded with related factors that will respond to nursing interventions. Nursing interventions do not change a medical diagnosis or diagnostic test. Instead nurses direct nursing interventions at behaviors or conditions that they are able to treat or manage. The first two incorrect diagnoses use a medical diagnosis and diagnostic procedure respectively as related or etiological factors. These are not conditions that nursing interventions can treat. The last diagnosis is incorrect because it is related to an assessment finding of a symptom or a defining characteristic.

A nurse assesses a young woman who works part time but also cares for her mother at home. The nurse reviews clusters of data that include the patient's report of frequent awakenings at night, reduced ability to think clearly at work, and a sense of not feeling well rested. Which of the following diagnoses is in the correct PES format? A) Disturbed Sleep Pattern evidenced by frequent awakening B) Disturbed Sleep Pattern related to family caregiving responsibilities C) Disturbed Sleep Pattern related to need to improve sleep habits D) Disturbed Sleep Pattern related to caregiving responsibilities as evidenced by frequent awakening and not feeling rested

D) Disturbed Sleep Pattern related to caregiving responsibilities as evidenced by frequent awakening and not feeling rested PES format includes: diagnostic label, related factor, and the defining characteristic by which the diagnosis is evidenced. B- is correct in the two-part format for a written diagnosis. A- no related factor C) an error, using a goal as a related factor

A nurse is reviewing a patient's list of nursing diagnoses in the medical record. The most recent nursing diagnosis is Diarrhea related to intestinal colitis. For which of the following reasons is this an incorrectly stated diagnostic statement? A) Identifying the clinical sign instead of an etiology B) Identifying a diagnosis on the basis of prejudicial judgment C) Identifying the diagnostic study rather than a problem caused by the diagnostic study I D) Identifying the medical diagnosis instead of the patient's response to the diagnosis.

D) Identifying the medical diagnosis instead of the patient's response to the diagnosis.

The nurse is caring for a patient with pneumonia. If a pleural effusion is developing, the nurse would expect which finding? A) Barrel-shaped chest B) Paradoxical respirations C) Hyperresonance on percussion D) Localized decreased breath sounds

D) Localized decreased breath sounds Rationale: Clinical manifestations of pleural effusion include diminished breath sounds over the affected area, decreased movement of the chest on the affected side, dullness to percussion, dyspnea, cough, and occasional sharp and nonradiating chest pain that is worse on inhalation.

Which guideline should the nurse follow when including interventions in a plan of care? Make sure the nursing interventions are unrelated to the original outcomes. Date the nursing interventions when written and when the plan of care is reviewed. Make sure the attending physician approves of and signs the nursing interventions. Make sure each nursing intervention does not describe the action the nurse should perform.

Date the nursing interventions when written and when the plan of care is reviewed. Explanation: Nursing interventions describe, and thus communicate to the entire nursing staff and health care team, the specific nursing care to be implemented for the client. Interventions should contain the date, a verb (action to be performed), the subject (who is to do it), and a descriptive phrase (how, when, where, how often, how long, or how much). The interventions should be dated both when written and when the care plan is reviewed. The interventions should directly relate to the outcomes. The physician does not approve and sign the interventions, because they are nursing interventions.

Six factors to consider for Intervention selection

Desired patient outcomes, characteristics of the nursing diagnosis, research based knowledge for the intervention, feasibility of the interventions, acceptability to the patient, nurse's competency

What priorities are important to educate patient to pay attention to help decrease their risk for the most common cause of death?

Determine and review clients prior blood glucose records to determine their usual blood glucose control Blood glucose control has been shown to directly impact mortality outcomes after an MI and is the immediate priority!!!!! Nutrition Management Balance nutrients, expenditure of energy, dose and timing of meds Maintain near normal BGLs Exercise Management

Evaluation

Determine whether the application of the nursing process, patient condition improves

A nurse designs a care plan to improve walking mobility in an older adult client. When the nurse encourages the client to implement the new strategies for ambulation, the client refuses to try and tells the nurse, "I find it easier to use a wheelchair." What action by the nurse may have led to failure to meet the outcome? -Choosing actions that do not solve the problem -Failing to update the written plan of care -Beginning the plan without family to help -Developing the plan without client input

Developing the plan without client input Explanation: Common problems with planning nursing care include failure to involve the client in the planning process, insufficient data collection, use of broadly stated outcomes, stating nursing orders that do not resolve the problem, and failure to update the plan of care. There is no indication that the nurse included strategies in the plan of care that did not solve the client's problem. There is no evidence that the care plan needed to be updated or that the nurse failed to do so. Although family support can be important to achieving client outcomes, not every client outcome requires family support.

What are the 3 parts of a nursing diagnosis?

Diagnostic Label- Approved by NANDA "Related to" statement Related Factor- "as evidence by" reason why patient is displaying ND

When preparing for physician-initiated or collaborative interventions, what is the importance of the nurses role?

Do not automatically implement the therapy, but to determine whether it is appropriate for the patient. The ability to recognize incorrect therapies is particularly important when administering medications or implementing procedures.

Concept mapping is one way to? a. connect concepts to a central subject. b. Relate ideas to patient health problems. c. Challenge a nurse's thinking about patient needs and problems. d. Graphically display ideas by organizing data. e. All of the above.

E

Planning care for patients in community-based settings involves:

Educating the patient/family about care. Guiding them to assume more of the care over time.

Goiter-physiological reason for a goiter

Enlargement of the thyroid gland Thyroid cells are stimulated to grow May result in an overactive thyroid or underactive thyroid

____________ ____________ is part of priority setting.

Ethical care

Basic critical thinking is...

Following a procedure step by step without adjusting to pt needs

Behavior Modifications for Obesity

Food diary Eliminate cues that precipitate eating Recognize factors that affect eating behaviors social support group, group programs

What is Bariatric surgery and what is the BMI requirement ?

For those with BMI >40 Restrictive/malabsorptive process ------Bypass ------Causes rapid weight loss

When assessing move from ____ to _____.

General to specific

Assessment steps: (6)

Health history Physical exam Observe behavior (verbal & nonverbal) Diagnostics & Lab Data Interpret/ validate assessment data Documentation

What is the most common cause of death for patients who have DM?

Heart Attack (Myocardial infarction...MI)

What is the purpose of a student plan care?

Helps you apply knowledge gained from the nursing and medical literature and the classroom to practice situation. Is more elaborate than a care plan used in a hospital or community agency because its purpose is to teach the process of planning care.

Classification of Priorities:

High- Emergent Intermediate Low- Affect patient's future well-being

Ordering of nursing diagnoses or patient problems uses determinations or urgency and/or importance to establish a preferential order for nursing interventions. Name the classification of priorities.

High-Emergent Intermediate-non-life threatening Low-Affect patient's future well-being

Risk Nursing Diagnosis

Human response to health conditions that may develop in vulnerable individual

What is a glycosylated hemoglobin level (HgA1c)? How often are these drawn? What are normal values?

Indicates the amount of glucose linked to hemoglobin ---Assess long-term glycemic control during previous 3 months ---Nurse Responsibility—inform patient that fasting is not necessary and that blood sample will be drawn Normal range: 4%-6% A1C of 6.5% or higher indicates poorly managed diabetes mellitus

A nurse is using a standardized plan of care for a client. Which action would be most important for the nurse to do? Individualize the plan to the client. Expect to modify the plan significantly. Identify the appropriate nursing diagnoses. Include the rationale for the interventions.

Individualize the plan to the client. Explanation: Standardized plans of care are written by a group of nurses who are experts in a given area of practice (e.g., obstetrics, rehabilitation, orthopedics). The plans are written for a client population with a specific medical diagnosis (e.g., total hip replacement, pressure injury, vaginal delivery, coronary artery bypass surgery). These experts identify the most common nursing diagnoses for this client population and write the goals and interventions usually necessary to resolve the problem. Each time a standardized plan of care is used, it must be individualized for a specific client. The danger of a standardized plan of care lies in the fact that it may not fit a specific client. Nurses must make judgments as to the degree to which standardized plans should be modified or whether they should not be used in individual cases. With a standardized plan of care, the most common nursing diagnoses have already been identified. Rationales are typically not included on clinical plans of care.

D.S. is noted to have an apple-type body shape. What do you know about body shape in relation to obesity problems and what other problems is she likely to have related to her weight?

Individuals with fat located primarily in the abdominal area (apple-shaped body) are at a greater risk for obesity-related complications than those whose fat is primarily located in the upper legs (pear-shaped body). Pain in joints, high cholesterol, sleep apnea

What needs to be assessed before removal of an NG tube?

Instill air into tube to assess placement--This clears the tube of any contents such as feeding or gastric drainage and decreases the changes of dragging any drainage through the esophagus and nasopharynx

Collaborative

Interdependent—Require combined knowledge, skill, and expertise of multiple health care professionals

What is interdisciplinary care?

Interdisciplinary care is contributions from all disciplines involved in patient care.

Indirect Care

Interventions/ treatments preformed away from patient ex. Infection control

Nursing- Sensitive Patient Outcome

Is a measurable patient state, behavior, or perception largely influenced by nursing interventions

For planning nursing care, what do nurses need to do?

Know the scientific rationale for the intervention. Possess the necessary psychomotor and interpersonal skills. Be able to function within a setting to use health care resources effectively.

What post-op manifestations might be present after a thyroidectomy?

Laryngeal stridor: (harsh, vibratory sound) may occur doing inspiration and expiration because of edema of the laryngeal nerve.

Expected Outcome

Measurable change that must be achieved to reach a goal Many times, several must be met to meet a single goal

B

Miyoko determines the patient outcomes for Mr. Mashoud based on his reaction to the medication regimen. Which of the following is an end result that translates into observable patient behaviors that are measurable and desirable? A. Unexpected outcome B. Expected outcome C. Sensitive outcome D. Accomplished outcome

all but D

Miyoko follows which steps to objectively evaluate the degree of success in achieving outcomes of care for Mr. Mashoud? (Select all that apply.) A. Identify the exact desired patient behavior. B. Evaluate the patient's actual behavior. C. Compare the outcome criteria with the actual behavior. D. Assess the desired behavior and anticipated outcome. E. Judge the degree of agreement between the outcome criteria and the actual behavior.

ongoing

Miyoko knows that evaluation is an ____________ process that occurs whenever contact with a patient occurs.

What age do babies commonly die from SIDS?

Most deaths occur between 1 and 4 months

in 1982

NANDA was founded

Which statement correctly describes a nurse-initiated intervention? -Nurse-initiated interventions are derived from the nursing diagnosis. -Nurse-initiated interventions require a physician's order. -Nurse-initiated interventions are actions deemed to have a low risk of harm to the client. -Nurse-initiated interventions are actions performed to diagnose a medical problem.

Nurse-initiated interventions are derived from the nursing diagnosis. Explanation: Nurse-initiated interventions, like client goals, are derived from the nursing diagnosis and do not require a physician's order. But whereas the problem statement of the diagnosis suggests the client goals, it is the cause of the problem (etiology) that suggests the nursing interventions. Nurse-initiated interventions do not necessarily pose a low risk of harm to the client. They are not performed to diagnose any problem, medical or otherwise, but to help prevent or resolve a problem identified in a nursing diagnosis and thereby to achieve the related expected client outcome.

Name the different types of interventions.

Nurse-initiated: Independent-actions that a nurse initiates. Health care provider initiated: Dependent-require an order from a physician or other health care professional. Collaborative: Interdependent-require combined knowledge, skill, and expertise of multiple health care professional

What is true of nursing responsibilities with regard to a physician-initiated intervention (physician's order)? Nurses do not carry out physician-initiated interventions. Nurses do carry out interventions in response to a physician's order. Nurses are responsible for reminding physicians to implement orders. Nurses are not legally responsible for these interventions.

Nurses do carry out interventions in response to a physician's order. Explanation: A physician-initiated intervention is initiated in response to a medical diagnosis, but carried out by a nurse in response to a doctor's order. Both the physician and the nurse are legally responsible for these interventions. Although nurses are not responsible for reminding physicians to implement orders, nurses may request a physician to implement an order or question an existing order by the physician if the nurse believes it is in the client's best interests.

Critical thinking in Planning Nursing Care

Nurses need to: Know the scientific rationale for the intervention Possess the necessary psychomotor and interpersonal skills Be able to function within a setting to use health care resources effectively

A client's diagnosis of breast cancer necessitates a bilateral mastectomy and breast reconstruction with tissue expanders. The nurse recognizes that the client's surgery will have a significant impact on the client's activities of daily living (ADLs) during the period of recovery. When should the nurse begin discharge planning to address this client's ADLs? -On the client's admission to the hospital -Once the client has received a discharge order -As soon as possible after the client's surgery -Once the client is admitted to the nursing unit from postanesthetic recovery

On the client's admission to the hospital Explanation: Discharge planning should begin when a client is admitted for treatment. All the other times listed are too late and are not consistent with a client who is able to understand the process of the hospitalization.

Priority- Setting

Ordering of nursing diagnosis using determinations of urgency to prioritize order for nursing action

A nurse is caring for a client with congestive heart failure. The nurse manager informs the nurse that the client was enrolled in a clinical trial to assess whether a 10-minute walk, three times per day, leads to expedited discharge. What type of evaluation best describes what the researchers are examining? Process Structure Outcome Cost-effectiveness

Outcome Explanation: An outcome evaluation determines the extent to which a client's behavioral response to a nursing intervention reflects the expected client outcome. A broad view of evaluation in health care includes three approaches, directed toward structure, process, and outcome, depending on the focus of evaluation and the criteria or standards being used. Process/implementation evaluation determines whether program activities have been implemented as intended. Cost-effectiveness evaluation compares the relative costs to the outcomes (effects) of two or more courses of action. Structure evaluation assesses the effectiveness of various health care structures.

Nursing Health History

Patient history you gather from assessment structured the way you feel is important

Where can sources of data come from?

Patient- best source Family- Can be primary source if patient is a child, incapacitated. Family can validate information Health care team- helps gather information Medical Records- source for history, test results, and treatment plan

What does patient-centered care require?

Patient-centered care requires you to know a patient's preferences, values, and expressed needs.

Subjective data

Patients verbal description of their health problems. Document using quotation marks

Patient Adherence

Patients/ family take the time to carry out required treatments

Database

Place for patient needs, health problems, and responses to problems

Functional health patterns

Practice standards provide categories of information for you to assess

Standing Order

Preprinted document containing orders for routine therapies for specific patients with identified critical problems

Critical Thinking

Process characterized by open- mindness, inquiry, perseverance, and willingness to look at each situation uniquely to determine which assumptions are true and relevant

Inference

Process of drawing conclusions from related pieces of evidence and previous experience

Reflection

Process of purposefully thinking back/recalling a situation to discover its purpose or meaning

Diagnostic Reasoning

Process to determine a patients health problems

Consultation

Process were you seek the expertise of specialist to identify problems/planning

Preventive Nursing Actions

Promote health and prevent illness to avoid need for acute healthcare

Open-ended questions

Prompts patient to discuss situation in more than 2 words

What is Acromegaly? What changes might occur in the patient?

Rare condition characterized by an overproduction of growth hormone. changes resulting from excess GH in adults can occur slowly, over a number of years, and may go unnoticed by the person, family and friends Thickening and enlargement of the boy and soft tissues on the face, feet and head

Implementation Process

Reassess patient Review/ revise plan of care Organize resources and care delivery Anticipate and prevent complications

Patient- Centered Goal

Reflects patient's highest possible level of wellness and independence

After his study session with Beth, John has a better handle on nursing diagnoses. He knows that a _______ diagnosis is applied to a vulnerable populations.

Risk diagnosis!

Guidelines for writing goals: (7)

Singular goal outcome Observable Measurable Time limited Mutual factors Realistic

What is the SMART acronym?

Specific Measurable Attainable Realistic Timed

SMART Acronym when writing goals and expected outcomes

Specific, measurable, attainable, realistic, timed

Assessment of palpation of the thyroid gland

Stand behind the client, place the fingers on either side of the trachea below the thyroid cartilage, have the client tilt the head to the right and swallow As the client swallows, displace the left lobe while palpating the right lobe Repeat the process, but displace the right lobe while palpating the left lobe The normal sized thyroid gland is not usually palpable

The nurse admitting a client with a new diagnosis of diverticulitis plans to teach the client about managing the disorder after discharge. What nursing intervention most completely meets the client's needs? Start from client's knowledge, teach about diet modifications, and check for learning. Present the client with videos and books about diet changes that reduce inflammation. Ask the client's learning style, then teach diet information using that style. Answer the client's questions about diet alterations, and then evaluate understanding.

Start from client's knowledge, teach about diet modifications, and check for learning. Explanation: The nursing interventions written to assist a client to meet an outcome must be comprehensive. Comprehensive nursing interventions specify what assessments need to be made and what nursing interventions, including teaching, counseling, and advocacy, need to be done. They should also include evaluation of the outcome of the intervention. "Start from client's knowledge, teach about diet modifications, and check for learning" provides the most comprehensive intervention for this client, as it includes assessment of the client's current level of knowledge, teaching, and evaluation of the teaching. None of the other answer options includes all three of these elements.

Following knee surgery a client is unable to bend the leg to put on pants, socks, and shoes. The nurse and client set a long-term goal of independence in bathing and dressing. What intervention by the nurse would be most effective in helping the client attain this goal? -Suggest the client use elastic shoe laces and pull clothes over leg with a grip extender. -Assist the client to put on the clothing that goes over the operated leg. -Tell the client's family to bring in clothes a size larger to make dressing easier. -Arrange for the social worker to schedule home health care with discharge planning.

Suggest the client use elastic shoe laces and pull clothes over leg with a grip extender. Explanation: Nursing interventions designed to promote client independence will implement methods for the client to perform a skill without help. Assistive devices for eating, bathing, dressing, and ambulation are common tools to develop client independence. The other options do not directly promote independent activity.

A nurse is working with a client who is having a difficult time accepting a new diagnosis of type 2 diabetes. The nurse pulls up a chair next to the client's bed and holds the client's hand while listening to the client's story. What type of nursing intervention is the nurse engaging in? Supportive Psychosocial Coordinating Supervisory

Supportive Explanation: Supportive interventions emphasize use of communication skills, relief of spiritual distress, and caring behaviors. Psychosocial interventions focus on resolving emotional, psychological, or social problems. Coordinating interventions involve many different activities, such acting as a client advocate and making referrals for follow-up care. Supervisory interventions refer to overseeing the client's overall health care.

What activity or sport is good for children with asthma ?

Swimming

guidelines and protocols

Systematically developed set of statements that helps nurses, physicians, and other health care providers make decisions about appropriate health care for specific clinical situations

Which is an example of a nurse-initiated intervention? Administer morphine sulfate 2 mg intravenous push every 3 hours as needed for pain. Administer oxygen at 4 L/min per nasal cannula. Administer a 1000-mL soap suds enema. Teach the client how to splint an abdominal incision when coughing and deep breathing.

Teach the client how to splint an abdominal incision when coughing and deep breathing. Explanation: A nurse-initiated intervention is an autonomous action based on scientific rationale that a nurse executes to benefit the client in a predictable way (related to the nursing diagnosis and expected outcomes). Nurse-initiated interventions, such as teaching, do not require a physician's order. A physician's order is required for the nurse to administer drugs (morphine sulfate and oxygen) and enemas.

If a patient has Hashimoto's thyroiditis AND a goiter, what are important nursing actions and why?

Teaching patient no to discontinue medications abruptly Tell patient to report to the HCP any change in symptoms, such as difficult breathing or swallowing, swelling to the and extremities or rapid weight gain or loss Reach those receiving thyroid hormone about the side effects of these drugs Patient is at risk for other autoimmune diseases such as Addison's disease, pernicious anemia, or Grave's disease

The nurse has identified the following outcome for the client: The client will have a soft, formed stool. Which error has the nurse made in writing the outcome? The nurse has not made any error in writing the outcome. The nurse has omitted the time frame. The nurse has omitted the defining characteristics. The outcome should indicate what the nurse will do.

The nurse has omitted the time frame. Explanation: Outcomes are client-centered, use action verbs, identify measurable performance criteria, and include a time frame as to when the outcome should be achieved. The time frame has been omitted. Defining characteristics are a component of the nursing diagnosis, not a client outcome. Because outcomes are client-centered, they describe what the client will do, not what the nurse will do.

Collaborative Interventions/ Interdependent

Therapies that require knowledge and skills from multiple health providers

What is hand-off reporting? And what is during this?

This is a critical time when the nurses collaborate and share important information that ensures the continuity of care for a patient and prevents errors or delays in providing nursing interventions. Transferring essential information from one nurse to the next nurse to the next during transitions in care. Ask questions, clarify, and confirm important details about a patient's progress and continuing care needs.

What is Maslow's Hierarchy of Needs

Triangle levels: Self Actualization Esteem Love and Belongings Safety Physiological needs basic to human survival

You work with the nutritionist to develop a meal plan for DJ. T or F: Collaborative interventions are therapies that involve multiple health care professionals.

True

Back-channeling

Using adaptive listening prompts such as: "go on" and "all-right"

D.S. sadly states, "I have tried and failed at every diet there is. It just doesn't matter anyway." What should you tell her about the importance of weight loss and the treatment options and resources available to her?

Weight loss can help reduce the complications associated with obesity, including cardiovascular disease and hypertension, type 2 diabetes, osteoarthritis, sleep apnea, gastroesophageal reflux disease (GERD), gallstones, nonalcoholic steatohepatitis, and cancer. Drug therapy for weight reduction, such as Alli, or bariatric surgery Support groups or individual therapy sessions Talk with her about HER reasons for wanting to lose weight. (motivation

Which is an independent nursing action that should be included in the plan of care for a client after an episode of ketoacidosis? a) Monitoring for signs of hypoglycemia resulting from treatment b.) Withholding glucose in any form until the situation is corrected c.) Giving fruit juices, broth, and milk as soon as the client is able to take fluids orally d.)Regulating insulin dosage according to the amount of ketones found in the client's urine

a) Monitoring for signs of hypoglycemia resulting from treatment During treatment for acidosis, hypoglycemia may develop; careful observation for this complication should be made by the nurse. Withholding all glucose may cause insulin coma. Whole milk and fruit juices are high in carbohydrates, which are contraindicated immediately following ketoacidosis. The regulation of insulin depends on the prescription for coverage; the prescription usually depends on the client's blood glucose level rather than ketones in the urine.

12. A client complains of pain. Which question asked by the nurse are most appropriate to assess the nature of the pain? a. "Can you describe your pain to me?" b. "Is your pain associated with movements?" c. "Can you rate your pain on a scale of 0 to 10?" d. "Do you notice your pain worsening with any activity?"

a. "Can you describe your pain to me?" The nurse may ask the client to describe the pain or to point the area that hurts. It may help to assess the nature of the pain. Asking about effect on pain with movement may help to assess precipitating factors. The severity of a pain could be identified by asking the client to rate it on a scale from 0 to 10. The precipitating factors can be identified by asking the client about worsening of the pain with a particular activity.

A male client with the diagnosis of bipolar disorder, depressed episode, is found lying on the floor in his room in the psychiatric unit. He states, "I don't deserve a comfortable bed; give it to someone else." What is the best response by the nurse? a. "Everyone has a bed. This one is yours." b. "You are not allowed to sleep on the floor." c. "I don't understand why you're on the floor." d. "You're a valuable person. You don't need to lie on the floor."

a. "Everyone has a bed. This one is yours." A matter-of-fact approach helps prevent a cycle in which the nurse expresses concern to a client who feels unworthy, which increases feelings of unworthiness

The nurse is teaching a group of students about assessing for respiratory system manifestations of alkalosis as a nursing priority. Which statement made by the student nurse indicates the need for further teaching? Select all that apply. a. "I should assess for low blood pressure." b. "I should assess for increased digitalis toxicity." c. "I should assess for a decreased rate of ventilation in respiratory alkalosis." d. "I should assess for an increased depth of ventilation in respiratory alkalosis." e. "I should assess for a decreased respiratory effort associated with skeletal muscle weakness in metabolic alkalosis."

a. "I should assess for low blood pressure." b. "I should assess for increased digitalis toxicity." c. "I should assess for a decreased rate of ventilation in respiratory alkalosis." The nurse should assess for low blood pressure and increased digitalis toxicity as cardiovascular manifestations of alkalosis, not respiratory manifestation. The nurse should assess for increased rate of ventilation in respiratory alkalosis. The nurse should assess for increased depth of ventilation in respiratory alkalosis. It is imperative that the nurse check for decreased respiratory effort associated with skeletal muscle weakness in metabolic alkalosis.

The nurse is involved in a therapeutic relationship with a depressed client. Which question and/or statement by the nurse is appropriate for stage 1 of this relationship? Select all that apply. a. "I'm here to talk with you about how you've been feeling." b. "How do you feel about keeping a journal regarding how you are feeling?" c. "Are you experiencing any suicidal or homicidal thought?" d. "Are you open to the prospect of being prescribed antidepressant medication?" e. "What we talk about will be shared only with your treatment team."

a. "I'm here to talk with you about how you've been feeling." c. "Are you experiencing any suicidal or homicidal thought?" e. "What we talk about will be shared only with your treatment team."

A client who is diagnosed with a duodenal ulcer asks, "Now that I have an ulcer, what comes next?" What is the nurse's best response? a. "Most peptic ulcers heal with medical treatment." b. "Clients with peptic ulcers have pain while eating." c. "Early surgery is advisable, especially after the first attack." d. "If ulcers are untreated, cancer of the stomach can develop."

a. "Most peptic ulcers heal with medical treatment." Treatment with medications, rest, diet, and stress reduction relieves symptoms, heals the ulcer, and prevents complications and recurrence. Clients with duodenal ulcers have pain after eating and especially at night; gastric ulcers cause pain during or close to eating. Surgery may be done after multiple recurrences and for treating complications. Perforation, pyloric obstruction, and hemorrhage, not cancer, are major complications.

Which instructions would the nurse include in a teaching plan for a patient with mild gastroesophageal reflux disease (GERD)? a. "The best time to take an as-needed antacid is 1 to 3 hours after meals." b. "A glass of warm milk at bedtime will decrease your discomfort at night." c. "Do not chew gum; the excess saliva will cause you to secrete more acid." d. "Limit your intake of foods high in protein because they take longer to digest

a. "The best time to take an as-needed antacid is 1 to 3 hours after meals."

The nurse is teaching a client newly diagnosed with diabetes about the importance of glucose monitoring. Which blood glucose levels should the nurse identify as hypoglycemia? a. 68 mg/dL (3.8 mmol/L) b. 78 mg/dL (4.3 mmol/L) c. 88 mg/dL (4.9 mmol/L) d. 98 mg/dL (5.4 mmol/L)

a. 68 mg/dL (3.8 mmol/L) Normal blood glucose level for an adult is 72-108 mg/dL (4-6 mmol/L). Clients who have blood glucose levels below 72 mg/dL (4 mmol/L) may experience hypoglycemia; 78 mg/dL (4.3 mmol/L), 88 mg/dL (4.9 mmol/L), and 98 mg/dL (5.4 mmol/L) are normal blood glucose levels.

A client is in a state of uncompensated acidosis. What approximate arterial blood pH does the nurse expect the client to have? a. 7.20 b. 7.35 c. 7.45 d. 7.48

a. 7.20 The pH of blood is maintained within the narrow range of 7.35 to 7.45. When there is an increase in hydrogen ions, the respiratory, buffer, and renal systems attempt to compensate to maintain the pH. If compensation is not successful, acidosis results and is reflected in a lower pH.

Which child is at the highest risk for blunt trauma associated with the indirect entry (hematogenous stage) of microorganisms? a. 8-year-old boy b. 10-year-old girl c. 13-year-old girl d. 14-year-old boy

a. 8-year-old boy e. The indirect entry of microorganisms, which is the hematogenous stage of osteomyelitis, most frequently affects the growing bones of boys younger than 12 years of age. Therefore an 8-year-old boy would be at the highest risk for blunt trauma.

Which individuals would be at high risk for low back pain (select all that apply)? a. A 63-year-old man who is a long-distance truck driver b. A 36-year-old construction worker who is 6 ft 2 in and weighs 260 lb c. A 44-year-old female chef with prior compression fracture of the spine d. A 30-year-old nurse who works on an orthopedic unit and smokes e. A 28-year-old female yoga instructor who is 5 ft 6 in and weighs 130 lb

a. A 63-year-old man who is a long-distance truck driver b. A 36-year-old construction worker who is 6 ft 2 in and weighs 260 lb c. A 44-year-old female chef with prior compression fracture of the spine d. A 30-year-old nurse who works on an orthopedic unit and smokes

A client's arterial blood gas report indicates that pH is 7.25, Pco 2 is 60 mm Hg, and HCO 3 is 26 mEq/L (26 mmol/L). Which client should the nurse consider is most likely to exhibit these blood gas results? a. A 65-year-old with pulmonary fibrosis b. A 24-year-old with uncontrolled type 1 diabetes c. A 45-year-old who has been vomiting for 3 days d. A 54-year-old who takes sodium bicarbonate for indigestion

a. A 65-year-old with pulmonary fibrosis The low pH and elevated Pco 2 are consistent with respiratory acidosis, which can be caused by pulmonary fibrosis, which impedes the exchange of oxygen and carbon dioxide in the lung. A 24-year-old with uncontrolled type 1 diabetes most likely will experience metabolic acidosis from excess ketone bodies in the blood. A 45-year-old who has been vomiting for 3 days most likely will experience metabolic alkalosis from the loss of hydrochloric acid from vomiting. A 54-year-old who takes sodium bicarbonate for indigestion most likely will experience metabolic alkalosis from an excess of base bicarbonate.

Analyze the following diagnostic findings for your patient with type 2 diabetes. Which result will need further assessment? a. A1C 9% b. BP 126/80 mm Hg c. FBG 130 mg/dL (7.2 mmol/L) d. LDL cholesterol 100 mg/dL (2.6 mmol/L)

a. A1C 9%

A patient is concerned that he may have asthma. Of the symptoms that he relates to the nurse, which ones suggest asthma or risk factors for asthma (select all that apply)? a. Allergic rhinitis b. Prolonged inhalation c. History of skin allergies d. Cough, especially at night e. Gastric reflux or heartburn

a. Allergic rhinitis c. History of skin allergies d. Cough, especially at night e. Gastric reflux or heartburn

A nurse is caring for a client with cirrhosis of the liver. Which laboratory test should the nurse monitor that, when abnormal, might identify a client who may benefit from neomycin enemas? a. Ammonia level b. Culture and sensitivity c. White blood cell count d. Alanine aminotransferase (ALT) level

a. Ammonia level Increased ammonia levels indicate that the liver is unable to detoxify protein by-products. Neomycin reduces the amount of ammonia-forming bacteria in the intestines. Culture and sensitivity testing is unnecessary; cirrhosis is an inflammatory, not infectious, process. Increased white blood cell count may indicate infection; however, this will have no relationship to the need for neomycin enemas. ALT, also called serum glutamic-pyruvic transaminase (SGPT), assesses for liver disease but has no relationship to the need for neomycin enemas.

An infant has developmental dysplasia of the hip. What clinical finding should the nurse expect to note during an assessment? a. Apparent shortening of one leg b. Limited ability to adduct the affected leg c. Narrowing of the perineum with an anal stricture d. Inability to palpate movement of the femoral head

a. Apparent shortening of one leg The affected leg appears to be shorter because the femoral head is displaced upward. The child's ability to abduct, not adduct, the affected leg is limited. Narrowing of the perineum with an anal stricture does not occur with hip dysplasia. When the femoral head slips out of the acetabulum, it is palpable.

A nurse is instructing a client with peptic ulcer disease (PUD) about the diet that should be followed during the acute phase. Which type of diet should the nurse stress? a. Bland foods b. Regular diet c. Gluten-free foods d. Low-carbohydrate foods

a. Bland foods A bland, nonirritating diet is recommended during the acute symptomatic phase. During the acute phase, a regular diet can cause discomfort. Clients should be instructed to avoid substances that increase gastric acid secretion, such as coffee, tea, and cola. Bedtime snacks should be avoided because they may stimulate gastric acid secretion as well. Gluten-free foods do not decrease gastric acid secretion. Low-carbohydrate foods do not decrease gastric acid secretion.

An 18-year-old high school student arrives at the local blood drive center to donate blood for the first time. As the site is being prepared for needle insertion, the student becomes agitated, starts to hyperventilate, and complains of dizziness and tingling of the hands. What should the nurse instruct the student to do? a. Breathe into cupped hands. b. Pant using rapid, shallow breaths. c. Use a rapid deep-breathing pattern. d. Hold the breath for as long as possible

a. Breathe into cupped hands. Breathing into cupped hands allows carbon dioxide to reenter the lungs, which will increase the serum bicarbonate level, relieving the respiratory alkalosis that is occurring as a result of hyperventilation. A rapid breathing pattern will exacerbate the respiratory alkalosis because excess carbon dioxide will continue to be expelled with rapid breathing, lowering the serum bicarbonate level. A fast deep-breathing pattern will exacerbate the respiratory alkalosis because excess carbon dioxide will continue to be expelled with rapid breathing, lowering the serum bicarbonate level. A person who is experiencing a panic attack will not be able to hold his or her breath.

After abdominal surgery a client returns to the unit with a nasogastric (NG) tube to low intermittent wall suction. The primary healthcare provider prescribes an antiemetic every six hours as needed for nausea. When the client complains of nausea, what should the nurse do first? a. Check for correct placement of the NG tube. b. Administer the prescribed antiemetic. c. Irrigate the NG tube with normal saline. d. Notify the primary healthcare provider immediately.

a. Check for correct placement of the NG tube. With a nasogastric (NG) tube for decompression in place, nausea may indicate tube displacement or obstruction. Checking placement can determine whether it is in the stomach; once placement is verified, then fluid can be instilled to ensure patency. The antiemetic may relieve the discomfort, but it will not determine the cause. If the tube is displaced it may be in the trachea or bronchi, and instillation of fluid will cause respiratory impairment before placement is confirmed. The nurse should always assess a situation carefully before notifying the healthcare provider.

A 10-year-old girl with diabetes joins the school's soccer team. Her mother is unsure whether to tell the coach of her child's condition. The mother asks the school nurse for guidance. On what information should the nurse base the response? a. Children with diabetes who participate in active sports can have episodes of hypoglycemia. b. Children may have to leave athletic teams if school authorities learn that they have diabetes. c. The school nurse will treat the child if clinical findings of hypoglycemia are recognized early. d. The coach might violate confidentiality by discussing the child's condition with other faculty members.

a. Children with diabetes who participate in active sports can have episodes of hypoglycemia. The people associated with the school who are interacting with the child should be told about the child's condition. Knowledgeable people can be alert for early signs of hypoglycemia and have snacks available for the child to help prevent a hypoglycemic episode. Forcing the child to leave the team is a form of discrimination; children with diabetes are allowed to engage in activities as long as their diabetes remains under control. The adult who is with the child when the signs of hypoglycemia first appear should be prepared to treat the child; this person may or may not be the nurse. Information about the child's health status is on a "need to know" basis; professionals are expected to honor confidentiality.

A client who underwent spinal surgery reports pain at bony prominences. On assessing, the nurse finds skin breakdown and tears. Which interventions performed by the nurse are appropriate? Select all that apply. a. Cleansing the ulcer with saline b. Removing the loose bits of tissue c. Measuring wound size every alternate week d. Repositioning the client at least every 5 hours e. Removing the old dressings daily if the ulcer is covered

a. Cleansing the ulcer with saline b. Removing the loose bits of tissue e. Removing the old dressings daily if the ulcer is covered The client who underwent spinal surgery may be bedridden for long periods of time. The client assessed with skin breakdown and tears is at a risk of pressure ulcers. The nursing interventions in the client include cleaning the ulcer with saline or a prescribed solution. The loose bits of tissue should be removed or trimmed. The nurse should remove old dressings or coverings daily if the ulcer is covered. Measuring wound size should be performed weekly or more often. The client with pressure ulcers should be repositioned at least every 1 or 2 hours.

A nurse is assessing a client with diabetic ketoacidosis. Which clinical manifestations should the nurse expect? Select all that apply. a. Dry skin b. Abdominal pain c. Kussmaul respirations d. Absence of ketones in the urine e. Blood glucose level of less than 72 mg/dL (3.3 mmol/L)

a. Dry skin b. Abdominal pain c. Kussmaul respirations Dry skin is a sign of dehydration in response to polyuria associated with the osmotic effect of an elevated serum glucose level. Abdominal pain is associated with diabetic ketoacidosis. In the absence of insulin, glucose cannot enter the cell or be converted to glycogen, so it remains in the blood. Breakdown of fats as an energy source causes an accumulation of ketones, which results in acidosis. The lungs, in an attempt to compensate for lowered pH, will blow off CO 2 (Kussmaul respirations). An absence of ketones in the urine indicates adequate production of glucose for energy. Insulin deficiency stimulates production of ketones as a by-product of fat oxidation for energy. Blood glucose level of less than 72 mg/dL (4 mmol/L) indicates hypoglycemia, not ketoacidosis.

When a client is expressing severe anxiety by sobbing in the fetal position on the bed, what is the nurse's priority? a. Ensuring a safe therapeutic milieu b. Monitoring and documenting vital signs c. Eliminating the cause of the client's anxiety d. Ensuring that the client's physical needs are met

a. Ensuring a safe therapeutic milieu Client safety is the nurse's first priority, and because the client has not experienced any physical injuries and is not at risk, attention should be directed toward psychiatric risk, in this case crisis control. The severely stressed individual is likely to experience increased vital signs and will continue to have physiologic needs such as food and water; however, these issues do not take priority over a psychiatric crisis. The client will not be able to concentrate on therapy related to identifying the source of the anxiety until the crisis has been managed.

A client is diagnosed with hyperthyroidism and is treated with 131I. Before discharge the nurse teaches the client to observe for signs and symptoms of therapy-induced hypothyroidism. Which signs and symptoms should be included in the teaching? Select all that apply. a. Fatigue b. Dry skin c. Insomnia d. Intolerance to heat e. Progressive weight gain

a. Fatigue b. Dry skin e. Progressive weight gain

You are caring for a patient with newly diagnosed type 1 diabetes. What information is essential to include in your patient teaching before discharge from the hospital (select all that apply)? a. Insulin administration b. Elimination of sugar from diet c. Need to reduce physical activity d. Use of a portable blood glucose monitor e. Hypoglycemia prevention, symptoms, and treatment

a. Insulin administration d. Use of a portable blood glucose monitor e. Hypoglycemia prevention, symptoms, and treatment

A client is found to have an adjustment disorder with mixed anxiety and depression. What should the nurse anticipate as the client's primary problem? a. Low self-esteem b. Deficient memory c. Intolerance of activity d. Disturbed personal identity

a. Low self-esteem When a client has an adjustment disorder, anxiety may be related to a disturbance in self-esteem and depression may be related to impaired social interaction. Problems with memory are not specifically related to an adjustment disorder. Activity intolerance, which is related to oxygenation problems, is not associated with adjustment disorders. A client with an adjustment disorder does not experience a disturbance in personal identity.

A nurse is caring for a client who is experiencing an underproduction of thyroxine (T 4). Which client response is associated with an underproduction of thyroxine? a. Myxedema b. Acromegaly c. Graves disease d. Cushing disease

a. Myxedema Myxedema is the severest form of hypothyroidism. Decreased thyroid gland activity means reduced production of thyroid hormones. Acromegaly results from excess growth hormone in adults once the epiphyses are closed. Graves disease results from an excess, not a deficiency, of thyroid hormones. Cushing disease results from excess glucocorticoids.

A client with colitis has had a hemicolectomy. Three days after surgery the nurse identifies that the client has abdominal distention and absent bowel sounds and has vomited 300 mL of dark green viscous fluid. The nurse contacts the primary healthcare provider and recommends which intervention? a. Nasogastric tube for decompression b. Antiemetic for nausea/vomiting c. Intravenous (IV) lactated Ringer for fluid replacement d. Stat electrolytes to assess for probable electrolyte imbalance e. Decompression removes collected secretions behind the nonfunctioning bowel segment

a. Nasogastric tube for decompression Decompression removes collected secretions behind the nonfunctioning bowel segment (paralytic ileus), thus reducing pressure on the suture line and allowing healing. Vomiting will subside as the bowel is decompressed. Although IV lactated Ringer for fluid replacement is important, the primary concern is decompression of the bowel; the amount of fluid removed will direct fluid and electrolyte replacement therapy.

A nurse is performing the history and physical examination of a client with Parkinson disease. Which assessments identified by the nurse support this diagnosis? Select all that apply. a. Nonintention tremors b. Frequent bouts of diarrhea c. Masklike facial expression d. Hyperextension of the neck e. Rigidity to passive movement

a. Nonintention tremors c. Masklike facial expression e. Rigidity to passive movement Nonintention tremors associated with Parkinson disease result from degeneration of the dopaminergic pathways and excess cholinergic activity in the feedback circuit. A masklike facial expression results from nigral and basal ganglial depletion of dopamine, an inhibitory neurotransmitter. Cogwheel rigidity is increased resistance to passive motion and is a classic sign of Parkinson. Constipation, not diarrhea, is a common problem because of a weakness of muscles used in defecation. The tendency is for the head and neck to be drawn forward, not hyperextended, because of loss of basal ganglial control.

Six hours after major abdominal surgery, a client reports severe abdominal pain and feeling faint. The nurse identifies a thready, rapid pulse. The nurse checks the medication administration record (MAR) (Physiological Aspects of Care record) and determines that the client can receive another injection of pain medication in an hour. Which is the most appropriate action by the nurse? a. Notify the healthcare provider about the client's symptoms b. Explain to the client that it is too early to have an injection for pain c. Reposition the client for greater comfort and turn on the television as a distraction d. Prepare the injection to administer it to the client early because of the severe pain

a. Notify the healthcare provider about the client's symptoms The client's signs and symptoms suggest the possibility of shock; the primary healthcare provider must be alerted to this possible life-threatening condition. Explaining to the client that it is too early is missing the big picture; the client may be hemorrhaging. The client has unmet needs that must be addressed first. Distraction is effective with mild, not severe, pain. Preparing and administering the pain medication early are outside the scope of nursing practice. Healthcare provider prescriptions must be followed as prescribed, or the healthcare provider should be notified.

An older client with shortness of breath is admitted to the hospital. The medical history reveals hypertension in the last year and a diagnosis of pneumonia three days ago. Which vital sign assessment would be seen as a sign that the client meeds immediate medical attention? a. Oxygen Saturation: 89% b. Body temperature: 101°F c. Blood Pressure: 130/80 mmHg d. Respiratory rate: 26 beats/minute

a. Oxygen Saturation: 89% An oxygen saturation less than 90% observed in a client with pneumonia indicates that the client is at risk of respiratory depression. Oxygen saturation would take priority in initiating the care. The client's body temperature indicates fever due to pneumonia, which should be considered secondary to the oxygen saturation problem. The blood pressure reading is normal. The increased respiratory rate may be due to fever, which would be considered secondary to the oxygen saturation problem.

For which patients with pneumonia would the nurse suspect aspiration as the likely cause of pneumonia (select all that apply)? a. Patient with seizures b. Patient with head injury c. Patient who had thoracic surgery d. Patient who had a myocardial infarction e. Patient who is receiving nasogastric tube feeding

a. Patient with seizures b. Patient with head injury e. Patient who is receiving nasogastric tube feeding

A nurse collaborates with a depressed client to increase self-esteem. What behavior should the nurse recall as typical of this type of client? a. Sets unrealistic goals b. Engages in criminal activity c. Attempts to manipulate others d. Overestimates current strengths

a. Sets unrealistic goals A depressed client may formulate goals that are unrealistic and therefore unattainable because of a lack of physical or emotional energy

When assessing a client's fluid and electrolyte status, the nurse recalls that the regulator of extracellular osmolarity is what? a. Sodium b. Potassium c. Chloride d. Calcium

a. Sodium Sodium is the most abundant extracellular fluid cation and regulates serum (extracellular) osmolarity, as well as nerve impulse transmission and acid-base balance. Potassium is the major intracellular osmolarity regulator, and it also regulates metabolic activities, transmission and conduction of nerve impulses, cardiac conduction, and smooth and skeletal muscle contraction. Chloride is a major extracellular fluid anion and follows sodium. Calcium is an extracellular cation necessary for bone and teeth formation, blood clotting, hormone secretion, cardiac conduction, transmission of nerve impulses, and muscle contraction.

A client is experiencing persistent vomiting, and serum electrolytes have been prescribed. The nurse should monitor which laboratory results? a. Sodium and chloride levels b. Bicarbonate and sulfate levels c. Magnesium and protein levels d. Calcium and phosphate levels

a. Sodium and chloride levels Sodium, which helps regulate the extracellular fluid volume, is lost with vomiting. Chloride, which balances cations in the extracellular compartment, also is lost with vomiting. Because sodium and chloride are parallel electrolytes, hyponatremia will accompany hypochloremia. Bicarbonate and sulfate levels, magnesium and protein levels, and calcium and phosphate levels do not provide significant information in relation to the effects of vomiting.

A client newly diagnosed with type 2 diabetes is receiving glyburide and asks the nurse how this drug works. What mechanism of action does the nurse provide? a. Stimulates the pancreas to produce insulin b. Accelerates the liver's release of stored glycogen c. Increases glucose transport across the cell membrane d. Lowers blood glucose in the absence of pancreatic function

a. Stimulates the pancreas to produce insulin Glyburide, an antidiabetic sulfonylurea, stimulates insulin production by the beta cells of the pancreas. Accelerating the liver's release of stored glycogen occurs when serum glucose drops below normal levels. Increasing glucose transport across the cell membrane occurs in the presence of insulin and potassium. Antidiabetic medications of the chemical class of biguanide improve sensitivity of peripheral tissue to insulin, which ultimately increases glucose transport into cells. Beta cells must have some function to enable this drug to be effective.

What should the nurse do when talking with a client with a history of panic disorder who is displaying many of the emotional and physiologic symptoms of a panic attack? a. Use short sentences and an authoritative voice. b. Describe the possible reasons for the client's anxiety. c. Keep asking questions, because the client is probably not going to volunteer much information. d. Suggest that the client refrain from crying, because most of the time crying makes matters worse.

a. Use short sentences and an authoritative voice. During a panic attack the attention span is shortened, making it difficult to follow long sentences. An authoritative voice lets the client know that the nurse is in control of the situation; the client is unable to set controls because of the anxiety level. Describing to the client the possible reasons for the anxiety may increase the client's anxiety level further. Asking questions may increase the client's anxiety level further. Crying is an outlet and should not be discouraged; telling someone not to cry usually worsens the crying and the anxiety.

Pain management for the burn patient is most effective when (select all that apply) a. a pain rating tool is used to monitor the patient's level of pain. b. painful dressing changes are delayed until the patient's pain is completely relieved. c. the patient is informed about and has some control over the management of the pain. d. a multimodal approach is used (e.g., sustained-release and short-acting opioids, NSAIDs, adjuvant analgesics). e. nonpharmacologic therapies (e.g., music therapy, distraction) replace opioids in the rehabilitation phase of a burn injury.

a. a pain rating tool is used to monitor the patient's level of pain. d. a multimodal approach is used (e.g., sustained-release and short-acting opioids, NSAIDs, adjuvant analgesics). e. nonpharmacologic therapies (e.g., music therapy, distraction) replace opioids in the rehabilitation phase of a burn injury.

To maintain a positive nitrogen balance in a major burn, the patient must a. eat a high-protein, high-carbohydrate diet. b. increase normal caloric intake by about three times. c. eat at least 1500 calories/day in small, frequent meals. d. eat a gluten-free diet for the chemical effect on nitrogen balance.

a. eat a high-protein, high-carbohydrate diet.

The nurse is alerted to possible anaphylactic shock immediately after a patient has received IM penicillin by the development of a. edema and itching at the injection site. b. sneezing and itching of the nose and eyes. c. a wheal-and-flare reaction at the injection site. d. chest tightness and production of thick sputum.

a. edema and itching at the injection site.

A plan of care for the patient with COPD could include (select all that apply) a. exercise such as walking. b. high flow rate of O2 administration. c. low-dose chronic oral corticosteroid therapy. d. use of peak flow meter to monitor the progression of COPD. e. breathing exercises such as pursed-lip breathing that focus on exhalation.

a. exercise such as walking. e. breathing exercises such as pursed-lip breathing that focus on exhalation.

An 80-year-old female patient is receiving palliative care for heart failure. The primary purpose(s) of her receiving palliative care is (are) to (select all that apply) a. improve her quality of life. b. assess her coping ability with disease. c. have time to teach patient and family about disease. d. focus on reducing the severity of disease symptoms. e. provide care that the family is unwilling or unable to give

a. improve her quality of life. d. focus on reducing the severity of disease symptoms.

Which blood gas result should the nurse expect an adolescent with diabetic ketoacidosis to exhibit? a. pH 7.30, CO 2 40 mm Hg, HCO 3 - 20 mEq/L (20 mmol/L) b. pH 7.35, CO 2 47 mm Hg, HCO 3 - 24 mEq/L (24 mmol/L) c. pH 7.46, CO 2 30 mm Hg, HCO 3 - 24 mEq/L (24 mmol/L) d. pH 7.50, CO 2 50 mm Hg, HCO 3 - 22 mEq/L (22 mmol/L)

a. pH 7.30, CO 2 40 mm Hg, HCO 3 - 20 mEq/L (20 mmol/L) A client in diabetic ketoacidosis will have blood gas readings that indicate metabolic acidosis. The pH will be acidic (7.30), and the HCO 3 - will be low (20 mEq/L [20 mmol/L]). The normal pH is 7.35 to 7.45; CO 2 ranges from 35 to 45 mm Hg, and HCO 3 - ranges from 22 to 26 (22 to 26 mmol/L). A pH of 7.35 and a CO 2 of 47 mm Hg indicate respiratory acidosis. pH values of 7.46 and 7.50 represent alkalosis, not acidosis.

The nurse is caring for a client with a diagnosis of diabetic ketoacidosis. Which arterial blood gas results are associated with this diagnosis? a. pH: 7.28; PCO 2: 28; HCO 3: 18 b. pH: 7.30; PCO 2: 54; HCO 3: 28 c. pH: 7.50; PCO 2: 49; HCO 3: 32 d. pH: 7.52; PCO 2: 26; HCO 3: 20 .

a. pH: 7.28; PCO 2: 28; HCO 3: 18 A low pH and bicarbonate reflect metabolic acidosis; a low PCO 2 indicates compensatory hyperventilation. A low pH and elevated PCO 2 reflect hypoventilation and respiratory acidosis. An elevated pH and bicarbonate reflect metabolic alkalosis; an elevated PCO 2 indicates compensatory hypoventilation. An elevated pH and low PCO 2 reflect hyperventilation and respiratory alkalosis

The injury that is least likely to result in a full-thickness burn is a. sunburn. b. scald injury. c. chemical burn. d. electrical injury

a. sunburn.

What is Ascites?

accumulation of serious fluid in the peritoneal or abdominal cavity *Assessment—shifting dullness in abdomen, Board-like, hard*

activities of daily living (ADLs)

activities usually performed in the course of a normal day, including ambulation, eating, dressing, bathing, and grooming.

Criterion- based Standards for Evaluation:

are the physiological, emotional, and behavioral responses that are a patient's goal and expected outcomes.

Concept maps...

are visual representation of all of a patient's nursing diagnoses that allows you to diagram interventions for each. group and categorize nursing concepts to give you a holistic view of your patient's health care needs and help you make better clinical decisions in planning care. help you learn the interrelationships among nursing diagnoses to create a unique meaning and organization of information.

The health care provider prescribes levothyroxine for a patient with hypothyroidism. After teaching regarding this drug, the nurse determines that further instruction is needed when the patient says a. "I can expect the medication dose may need to be adjusted." b. "I only need to take this drug until my symptoms are improved." c. "I can expect to return to normal function with the use of this drug." d. "I will report any chest pain or difficulty breathing to the doctor right away."

b. "I only need to take this drug until my symptoms are improved."

The nurse is teaching a group of students about neuromuscular manifestations of alkalosis with hypocalcemia. Which statements provided by a student nurse indicate the need for further learning? Select all that apply. a. "The client would show signs of twitching." b. "The client would show signs of hyporeflexia." c. "The client would show signs of paresthesias." d. "The client would show signs of muscle cramping." e. "The client would show signs of skeletal muscle weakness."

b. "The client would show signs of hyporeflexia." c. "The client would show signs of paresthesias." The neuromuscular manifestation of alkalosis with hypocalcemia is hyperreflexia, not hyporeflexia. Paresthesias is a symptom of alkalosis, which is manifested in the central nervous system not the neuromuscular system. The manifestation of alkalosis is neuromascular and can be observed through twitching, muscle cramping, and skeletal muscle weakness.

A patient in the unit has a 103.7° F temperature. Which intervention would be most effective in restoring normal body temperature? a. Use a cooling blanket while the patient is febrile. b. Administer antipyretics on an around-the-clock schedule. c. Provide increased fluids and have the UAP give sponge baths. d. Give prescribed antibiotics and provide warm blankets for comfort.

b. Administer antipyretics on an around-the-clock schedule.

A nurse is caring for an infant whose vomiting is intractable. Which complication is most likely to occur? a. Acidosis b. Alkalosis c. Hyperkalemia d. Hypernatremia

b. Alkalosis Excessive vomiting causes an increased loss of hydrogen ions (hydrochloric acid), leading to metabolic alkalosis, an excess of base bicarbonate. Acidosis is caused by retention of hydrogen ions and a loss of base bicarbonate, which is more likely to occur with diarrhea. Hypokalemia, not hyperkalemia, will occur. With the loss of chloride ions, hyponatremia is more likely to occur.

When an older client with heart failure is transferred from the emergency department to the medical service, what should the nurse on the unit do first? a. Interview the client for a health history. b. Assess the client's heart and lung sounds. c. Monitor the client's pulse and temperature. d. Obtain the client's blood specimen for electrolytes.

b. Assess the client's heart and lung sounds. With heart failure, the left ventricle is not functioning effectively, which is evidenced by an increased heart rate and crackles associated with pulmonary edema. The health history interview is done after vital signs and breath sounds are obtained and the client is stabilized. Although an infection would complicate heart failure, there are no signs that indicate this client has an infection. Obtaining the client's blood specimen for electrolytes is inappropriate for immediate monitoring; it should be done after vital signs and clinical assessments have been completed.

While caring for his dying wife, the husband states that his wife is a devout Roman Catholic but he is a Baptist. Who is considered the most reliable source for spiritual preferences concerning EOL care for the dying wife? a. A priest b. Dying wife c. Hospice staff d. Husband of dying wife

b. Dying wife

A 65-year-old stroke patient with limited mobility has a purple area of suspected deep tissue injury on the left greater trochanter. Which nursing diagnoses are most appropriate (select all that apply)? a. Acute pain related to tissue damage and inflammation b. Impaired skin integrity related to immobility and decreased sensation c. Impaired tissue integrity related to inadequate circulation secondary to pressure d. Risk for infection related to loss of tissue integrity and undernutrition secondary to stroke e. Ineffective peripheral tissue perfusion related to arteriosclerosis and loss of blood supply to affected area

b. Impaired skin integrity related to immobility and decreased sensation c. Impaired tissue integrity related to inadequate circulation secondary to pressure

An older adult fell at home and fractured the left hip. Which response should the emergency department nurse identify as a typical clinical indicator associated with a fractured hip? a. Affected hip is ecchymotic. b. Left leg is noticeably shorter than the right. c. Left extremity is internally rotated. d. Affected hip is tender when touched.

b. Left leg is noticeably shorter than the right. There is overriding of bones in the fractured hip, and the leg on the affected side appears noticeably shorter than the unaffected leg. Ecchymosis is evidence of soft tissue and blood vessel damage; this may or may not be associated with a fractured hip. The affected leg is externally, not internally, rotated with a fractured hip. Pain associated with a fractured hip is not mild; it causes extreme pain.

What client factor does the nurse consider to have the greatest impact on the effectiveness of bariatric surgery? a. Freedom from concurrent high-risk conditions b. Motivation to cooperate with required lifestyle changes c. Willingness to have a panniculectomy a year after weight is stabilized d. Ability to tolerate the large abdominal incision necessary for this surgery

b. Motivation to cooperate with required lifestyle changes Bariatric surgery requires the client to engage in significant postoperative lifestyle changes (e.g., radically modifying eating habits, taking nutritional supplements, meeting numerous emotional challenges, engaging in exercise); clients who cannot cooperate with the postoperative program are not considered candidates for bariatric surgery

A nurse is caring for depressed older adults. What precipitating factors for depression are most common in the older adult without cognitive problems? Select all that apply. a. Dementia b. Multiple losses c. Declines in health d. A milestone birthday e. An injury requiring hospitalization

b. Multiple losses c. Declines in health Depression in the older adult is most often associated with the loss of family members and friends (e.g., death, relocation) and declines in mobility, health, and income. A decline in health, particularly when associated with a chronic illness, frequently precipitates depression in older adults. Dementia is a cognitive problem. Research does not correlate the onset of depression with a milestone birthday in older adults. A traumatic injury does not precipitate the onset of depression in the older adult as often as does a chronic illness.

Which is the priority intervention for the dependent client with peptic ulcer disease (PUD) who is vomiting bright red blood? a. Apply oxygen b. Place the client in a side-lying position c. Prepare to administer packed red blood cells d. Assess the client's pulse and blood pressure

b. Place the client in a side-lying position Recall the airway, breathing, and circulation (ABCs) of priority care. The client who needs assistance to manage self-care (dependent) should be placed in the side-lying position when vomiting to prevent aspiration.

A 6-year-old child who has colicky abdominal pain and guarding, as well as nausea, anorexia, and a low-grade fever, is admitted to the pediatric unit. During the admission assessment, a nurse palpates the abdomen and elicits pain in the right lower quadrant. What care does the nurse expect to implement in light of this assessment? a. Symptomatic, until the viral infection resolves b. Preoperative, for removal of an inflamed appendix c. Preparatory, to test for the cause of an irritable bowel d. Anticipatory, for administration of a drug for the parasitic infestation

b. Preoperative, for removal of an inflamed appendix Preoperative care, for removal of an inflamed appendix; the signs and symptoms described are the classic signs and symptoms related to acute appendicitis. They are caused by inflammation and altered gastrointestinal function. Symptomatic, until the viral infection resolves does not apply since appendicitis is not related to a viral infection. The child will have diarrhea if there is an irritable bowel. Anticipatory, for administration of a drug for the parasitic infestation does not apply since appendicitis is not related to a parasitic infestation.

Which activity is most appropriate for a nurse to introduce to a depressed client during the early part of hospitalization? a. Board game b. Project involving drawing c. Small aerobic exercise group d. Card game with three other clients

b. Project involving drawing An art project that may be worked on successfully at one's own pace is appropriate for a depressed client. A board game or card game with three other clients require too much concentration and may increase the client's feelings of despair. This client is probably experiencing psychomotor retardation, and at this time an aerobic exercise group would not be appropriate.

A school-aged child is receiving 45 units of intermediate-acting insulin at 7:00 AM and 7:00 PM. What will the nurse tell the parents regarding a bedtime snack? a. Offer a snack at bedtime if there are signs of hyperglycemia. b. Provide a bedtime snack to prevent hypoglycemia during the night. c. Withhold the snack after dinner to prevent hyperglycemia during sleep. d. Leave a snack at the bedside in case the child becomes hungry during the night.

b. Provide a bedtime snack to prevent hypoglycemia during the night. Intermediate-acting insulin peaks in 4 to 12 hours; a bedtime snack will prevent hypoglycemia during the night. Offering a snack at bedtime if there are signs of hyperglycemia is unsafe because it will intensify the hyperglycemia; if hyperglycemia is present, the child needs insulin. Bedtime snacks are recommended for people taking intermediate-acting insulin. When hypoglycemia develops, the child will be asleep; the snack should be eaten before bed.

The nurse is providing care for parents who have experienced a stillbirth. What is the most appropriate intervention at this time? a. Giving a detailed explanation of what may have caused the stillbirth b. Providing the parents the opportunity to say goodbye to their newborn c. Explaining that autopsy is not recommended in the setting of a stillbirth d. Waiting to provide any information about follow-up care until the parents have had an opportunity to adjust to the grief

b. Providing the parents the opportunity to say goodbye to their newborn

A client falls at home and is brought to the emergency department by family members. The client reports intercostal pain and is confused and disoriented. Which is the best way for the nurse to determine whether this behavior is new for the client? a. Interview the client to identify when the confusion started. b. Question the family members about the client's usual behavior. c. Ask the primary healthcare provider when the confusion was noted first. d. Observe the client for a few hours before determining the onset of confusion.

b. Question the family members about the client's usual behavior. Family members usually know the client's behavior and serve as important sources of information when a client is confused. The primary healthcare provider is an additional source, but family members are the nurse's best source of information when the client is confused. In the presence of confusion, the client is an unreliable source. Observing the client will not alter the information.

Three days after a cast is applied to a client's fractured tibia, the client reports that there is a burning pain over the ankle. The cast over the ankle feels warm to the touch, and the pain is not relieved when the client changes position. What is the nurse's priority action? a. Obtain a prescription for an antibiotic. b. Report the client's concern to the primary healthcare provider. c. Administer the prescribed medication for pain. d. Explain that this is typical after a cast is applied.

b. Report the client's concern to the primary healthcare provider. The client's concern indicates tissue hypoxia or breakdown and should be reported to the primary healthcare provider. Other data, such as elevated temperature or increased white blood cells, are not present to support the presence of an infection. Although administering the prescribed medication for pain will be done to provide relief of pain, the priority is to notify the primary healthcare provider. This is not a typical response to a cast and may indicate a complication.

A client receiving morphine is being monitored by the nurse for signs and symptoms of overdose. Which clinical findings support a conclusion of overdose? Select all that apply. a. Polyuria b. Sedation c. Bradycardia d. Dilated pupils e. Slow respirations

b. Sedation c. Bradycardia e. Slow respirations The central nervous system (CNS) depressant effect of morphine causes sedation. The CNS depressant effect of morphine causes bradycardia. The CNS depressant effect of morphine causes bradypnea. Morphine does not increase urine output. Morphine causes constriction of pupils.

A client with multiple sclerosis is in remission. Which diversional activity should the nurse encourage that best meets the client's needs while in remission? a. Hiking b. Swimming c. Sewing classes d. Watching television

b. Swimming Swimming helps keep the muscles supple, without requiring fine-motor activity. Hiking might prove too rigorous for the client. Sewing requires fine-motor activity and will be difficult for the client. Sedentary activities are not helpful in maintaining muscle tone.

A nurse plans to use family therapy as a means of helping a family cope with their child's terminal illness. The nurse bases this choice on what principle? a. It is more efficient to interact with the whole family together. b. The entire family is involved because what happens to one member affects them all. c. It will prevent the parents from deceiving each other about the true nature of their child's condition. d. The nurse can control manipulation and alliances better by using this mode of intervention.

b. The entire family is involved because what happens to one member affects them all. Family therapy views the whole (gestalt) within the context in which the emotional problems are occurring. Efficiency is not an adequate rationale for choosing this therapeutic approach. The nurse may or may not control manipulation and alliances better; an astute nurse can control manipulation and alliance within any group. Promotion of truthfulness is a secondary gain achieved with this mode of therapy.

A patient 1 day postoperative after abdominal surgery has incisional pain, 99.5° F temperature, slight erythema at the incision margins, and 30 mL serosanguineous drainage in the Jackson-Pratt drain. Based on this assessment, what conclusion would the nurse make? a. The abdominal incision shows signs of an infection. b. The patient is having a normal inflammatory response. c. The abdominal incision shows signs of impending dehiscence. d. The patient's physician must be notified about her condition.

b. The patient is having a normal inflammatory response.

A 6-year-old child who was rescued from a burning building is admitted to the burn unit with a diagnosis of smoke inhalation. For which priority complication should the nurse assess the child? a. Systemic infection b. Tracheobronchial edema c. Posttraumatic stress disorder d. Generalized adaptation to stress

b. Tracheobronchial edema Heat and inhaled smoke-related irritants may cause fluid to shift from the intravascular compartment into the interstitial compartment, resulting in edema, which obstructs the airway. Although monitoring for infection is important, a patent airway is the priority. Although monitoring for posttraumatic stress disorder is important because the condition could occur later, maintaining a patent airway is the priority. Although monitoring for physical and emotional responses to stress is important, maintaining a patent airway is the priority.

A client who has been pregnant for 5 months experiences a spontaneous abortion after an accident. The client tells the nurse that she feels depressed over the loss of her son. She describes how he would have looked and how bright he would have been. What is the client demonstrating? a. Panic level of anxiety b. Typical grief syndrome c. Pathological grief reaction d. Diminished ability to test reality

b. Typical grief syndrome The client is grieving the loss of a fantasized child; talking about it is part of the typical grief reaction.

The nurse assessing a patient with a chronic leg wound finds local signs of erythema and the patient complains of pain at the wound site. What would the nurse anticipate being ordered to assess the patient's systemic response? a. Serum protein analysis b. WBC count and differential c. Punch biopsy of center of wound d. Culture and sensitivity of the wound

b. WBC count and differential

A client with arthritis states that the prescribed aspirin causes stomach irritation even when taken with food. How does the nurse instruct the client to take the aspirin? a. An hour before a meal b. With a full glass of water c. With sodium bicarbonate d. At the same time as the other drugs

b. With a full glass of water A full glass of water helps decrease gastric irritation by diluting the acidic substances in the stomach. If aspirin is taken on an empty stomach, gastric irritation is increased. Although taking the medicine with sodium bicarbonate will limit gastric irritation, it will also decrease the effect of aspirin by increasing its renal excretion. Aspirin has a gastric-irritating and ulcerogenic effect, which may be potentiated by other drugs.

Appropriate nonopioid analgesics for mild pain include (select all that apply) a. oxycodone. b. ibuprofen (Advil). c. lorazepam (Ativan). d. acetaminophen (Tylenol). e. codeine with acetaminophen (Tylenol #3).

b. ibuprofen (Advil). d. acetaminophen (Tylenol).

A patient has 25% TBSA burn from a car fire. His wounds have been debrided and covered with a silver-impregnated dressing. The nurse's priority intervention for wound care would be to a. reapply a new dressing without disturbing the wound bed. b. observe the wound for signs of infection during dressing changes. c. apply cool compresses for pain relief in between dressing changes. d. wash the wound aggressively with soap and water three times a day.

b. observe the wound for signs of infection during dressing changes.

The patient with advanced cirrhosis asks why his abdomen is so swollen. The nurse's response is based on the knowledge that a. a lack of clotting factors promotes the collection of blood in the abdominal cavity. b. portal hypertension and hypoalbuminemia cause a fluid shift into the peritoneal space. c. decreased peristalsis in the GI tract contributes to gas formation and distention of the bowel. d. bile salts in the blood irritate the peritoneal membranes, causing edema and pocketing of fluid.

b. portal hypertension and hypoalbuminemia cause a fluid shift into the peritoneal space.

The son of a terminally ill woman is concerned about his mother's condition. He asks the nurse, "Will she get better?" What is the most appropriate response by the nurse? a. "Her vital signs are stable. Right now she's holding her own." b. "Of course she will. You can't give up. You have to hope for the best." c. "Her condition is very serious. It might help you if we discuss your concerns." d. "I don't know; you'll have to ask her oncologist. I'll leave a note that you're here."

c. "Her condition is very serious. It might help you if we discuss your concerns." Offering to discuss the situation provides the son with an opportunity to express his feelings. Telling the son that the woman is holding her own does not address the family member's concern. Telling the son that the woman will pull through is false reassurance and cuts off communication. Telling the son to speak with the health care provider shuts off communication and abdicates nursing responsibility to the client.

A nurse moves into the working phase of a therapeutic relationship with a depressed client who has a history of suicide attempts. What question should the nurse ask the client when exploring alternative coping strategies? a. "How have you managed your problems in the past?" b. "What do you feel that you've learned from this suicide attempt?" c. "How will you manage the next time your problems start piling up?" d. "Were there other things going on in your life that made you want to die?"

c. "How will you manage the next time your problems start piling up?" How will you manage the next time your problems start piling up?" focuses the interaction toward the future and invites the client to explore alternative coping strategies.

A client is newly diagnosed with multiple sclerosis. The client is obviously upset with the diagnosis and asks, "Am I going to die?" Which is the nurse's best response? a. "Most individuals with your disease live a normal life span." b. "Is your family here? I would like to explain your disease to all of you." c. "The prognosis is variable; most individuals experience remissions and exacerbations." d. "Why don't you speak with your healthcare provider? You probably can get more details about your disease."

c. "The prognosis is variable; most individuals experience remissions and exacerbations." "The prognosis is variable; most individuals experience remissions and exacerbations" is a truthful answer that provides some realistic hope. The response "Most individuals with your disease live a normal life span" provides false reassurance; repeated exacerbations may reduce the life span. The response "Is your family here? I would like to explain your disease to all of you" avoids the client's question; the family did not ask the question. The response "Why don't you speak with your healthcare provider? You probably can get more details about your disease" avoids the client's question and transfers responsibility to the practitioner.

A client is receiving total parenteral nutrition. The nurse assesses for which client response that indicates hyperglycemia? a. Paralytic ileus b. Respiratory rate below 16 c. A fruity odor to the breath d. Serum glucose of 105 mg/100 mL

c. A fruity odor to the breath Hyperglycemia is indicated by a fruity odor to the breath. Paralytic ileus is not associated with hyperglycemia. With hyperglycemia there is hyperventilation. Serum glucose of 105 mg/100 mL is within the expected range.

A nurse is caring for a client admitted to the hospital for diabetic ketoacidosis. Which clinical findings related to this event should the nurse document in the client's clinical record? Select all that apply. a. Sweating b. Retinopathy c. Acetone breath d. Increased arterial bicarbonate level e. Decreased arterial carbon dioxide level

c. Acetone breath e. Decreased arterial carbon dioxide level A fruity odor to the breath (acetone breath) occurs when the ketone level is elevated in ketoacidosis. Metabolic acidosis initiates respiratory compensation in the form of Kussmaul respirations to counteract the effects of ketone buildup, resulting in a decreased arterial carbon dioxide level. As the glucose level decreases in hypoglycemia, the sympathetic nervous system is activated, and epinephrine and norepinephrine are secreted, causing diaphoresis. Retinopathy is a long-term complication of diabetes caused by microvascular changes in the retina; it is not a sign of ketoacidosis. With ketoacidosis, the serum bicarbonate level is decreased, not increased, in an effort to neutralize ketones when seeking acid-base balance.

A client admitted in the emergency department has airway obstruction, chest wall trauma, external hemorrhage, and hypoglycemia. Which condition of the client will be given the highest priority? a. Hypoglycemia b. Chest wall trauma c. Airway obstruction d. External hemorrhage

c. Airway obstruction The highest priority intervention is to establish a patent airway because inadequate oxygen supply to the brain may cause brain death. Assessing the metabolic conditions is done after the airway is cleared. Once the airway is cleared, then the chest wall of the client is assessed. Hemorrhage is assessed after the airway of the client is cleared.

A depressed client says, "I'm no good. I'm better off dead." What is the priority nursing intervention? a. Responding, "I'll stay with you until you're less depressed." b. Replying, "I think you're good; you should think about living." c. Alerting the staff to schedule 24-hour observation of the client d. Unobtrusively removing those articles that may be used in a suicide attempt

c. Alerting the staff to schedule 24-hour observation of the client Alerting the staff to schedule 24-hour observation of the client is the most therapeutic approach to preventing suicide. A staff member also provides special attention to help the client meet dependency needs and reduce a self-defeating attitude

Windows in the recreation room of the adolescent psychiatric unit have been broken on numerous occasions. After a group discussion one of the adolescents confides that another adolescent client broke them. What should the nurse do when using an assertive intervention instead of aggressive confrontation? a. Confront the adolescent openly in the group, using a controlled voice and maintaining direct eye contact. b. Knock on the door of the adolescent's room and ask whether the adolescent would come out to talk about the situation. c. Approach the adolescent when the client is alone and, after making direct eye contact, inquire about the involvement in these incidents. d. Use a trusting approach toward the adolescent and imply that the staff doubts the adolescent's involvement but requests a denial for the record.

c. Approach the adolescent when the client is alone and, after making direct eye contact, inquire about the involvement in these incidents. A private confrontation with presentation of reported facts allows verification; a calm, direct manner is most assertive

A nurse becomes aware of an older client's feeling of loneliness when the client states, "I only have a few friends. My daughter lives in another state and couldn't care less whether I live or die. She doesn't even know I'm in the hospital." How should the nurse interpret the client's communication? a. As a call for help to prevent the client from acting on suicidal thoughts b. As a manipulative attempt to persuade the nurse to call the daughter c. As a reflection of depression that is causing feelings of hopelessness d. As a request for information about social support groups in the community

c. As a reflection of depression that is causing feelings of hopelessness This statement provides clues that the client feels no one cares, so there is no reason the client should care. These feelings are common in depression. The clues presented should not lead the nurse to conclude that the client is looking for help to prevent suicidal activities, is attempting to manipulate the nurse, or is looking for information about community social support groups.

A depressed client has feelings of failure and a low self-esteem. In what activity should the client initially be encouraged to become involved? a. Joining other clients in playing a board game b. Singing in a karaoke contest to be held at the end of the week c. Assisting a staff member in working on the monthly bulletin board d. Selecting the movie to be played during the evening recreation period

c. Assisting a staff member in working on the monthly bulletin board Working on the bulletin board with staff members involves minimal energy and decision-making and is the least threatening activity

How would the nurse describe the exudate characteristic of a serosanguineous wound? a. Greenish-blue pus b. Creamy yellow pus c. Blood-tinged amber fluid d. Beige pus with a fishy odor

c. Blood-tinged amber fluid Blood-tinged amber fluid is characteristic of serosanguineous wound exudate. Greenish-blue pus, creamy yellow pus, and beige pus with a fishy odor are characteristics of purulent wound exudate.

Which words are most likely to be used to describe neuropathic pain (select all that apply)? a. Dull b. Mild c. Burning d. Shooting e. Shock-like

c. Burning d. Shooting e. Shock-like

What is the priority action for the nurse to take if the patient with type 2 diabetes complains of blurred vision and irritability? a. Call the physician. b. Administer insulin as ordered. c. Check the patient's blood glucose level. d. Assess for other neurologic symptoms

c. Check the patient's blood glucose level.

Which one of the orders should a nurse question in the plan of care for an elderly immobile stroke patient with a stage III pressure ulcer? a. Pack the ulcer with foam dressing. b. Turn and position the patient every hour. c. Clean the ulcer every shift with Dakin's solution. d. Assess for pain and medicate before dressing change

c. Clean the ulcer every shift with Dakin's solution.

A nurse teaches an elderly client safety tip to prevent falls. Which physiologic change may have occurred in the client? a. Slowed movement b. Cartilage degeneration c. Decreased bone density d. Decreased range of motion (ROM)

c. Decreased bone density Teaching safety tips to prevent falls would best help a client with decreased bone density. If a client experiences slow movements, the nurse should not rush the client because the client may become frustrated if hurried. Providing a client with cartilage degeneration with a moist heat source such as a shower or a warm compress is beneficial because this action may increase blood flow to the area. A nurse should assess a client's ability to perform activities of daily living and mobility to help improve the self-care skills of clients with a decreased range of motion.

A client feeling increasingly tired seeks medical care. Type 1 diabetes is diagnosed. What causes increased fatigue with type 1 diabetes? a. Increased metabolism at the cellular level b. Increased glucose absorption from the intestine c. Decreased production of insulin by the pancreas d. Decreased glucose secretion into the renal tubules

c. Decreased production of insulin by the pancreas Insulin facilitates transport of glucose across the cell membrane to meet metabolic needs and prevent fatigue. With diabetes there is decreased cellular metabolism because of the decrease in glucose entering the cells. Glucose is not absorbed from the intestinal tract by the cells; fatigue is caused by decreased, not increased, cellular levels of glucose. Filtration and excretion of glucose by the kidneys do not regulate energy levels; if insulin production is adequate, glucose does not spill into the urine.

A client presents with gastric pain, vomiting, dehydration, weakness, lethargy, and shallow respirations. Laboratory results indicate metabolic alkalosis. The diagnosis of gastric ulcer has been made. What is the primary nursing concern? a. Chronic pain b. Risk for injury c. Electrolyte imbalance d. Inadequate gas exchange

c. Electrolyte imbalance The stomach produces about 3 L of secretions per day. Fluid lost through vomiting can produce inadequate fluid volume and electrolyte imbalance, which can lead to dysrhythmias and death. Although pain is associated with gastric ulcers and requires intervention, it is not life threatening as is an electrolyte imbalance. Although the risk for injury is a concern, it is not the priority. Although respirations may be shallow when the client is experiencing pain, this is not the priority.

A client's wound is healing. Which event occurs in the proliferative phase of wound healing? a. Thinning of scar tissue b. Strengthening of collagen c. Formation of "granulation" tissue d. Increase in capillary permeability

c. Formation of "granulation" tissue "Granulation" tissue is formed in the proliferative tissue. Thinning of scar tissue and strengthening of collagen fibers is seen in the maturation phase of wound healing. The increase in capillary permeability occurs in the inflammatory phase of wound healing.

The nurse is assessing a client's arterial blood gases and determines that the client is in compensated respiratory acidosis. The pH value is 7.34; which other result helped the nurse reach this conclusion? a. PO 2 value is 80 mm Hg. b. PCO 2 value is 60 mm Hg. c. HCO 3 value is 50 mEq/L (50 mmol/L). d. Serum potassium value is 4 mEq/L (4 mmol/L).

c. HCO 3 value is 50 mEq/L (50 mmol/L). The HCO 3 value is elevated. The urinary system compensates by retaining H + ions, which become part of the bicarbonate ions; the bicarbonate level becomes elevated and increases the pH level to near the expected range. The expected HCO 3 value is 21 to 28 mEq/L (21 to 28 mmol/L), and the expected pH value is 7.35 to 7.45. The body's usual PO 2 value is 80 to 100 mm Hg; 80 mm Hg is within the expected range. The body's PCO 2 value is 35 to 45 mm Hg; although in compensated respiratory acidosis [1] [2] the PCO 2 level may be increased, it is the increased HCO 3 level that indicates compensation. A K + level of 4 mEq/L (4 mmol/L) is within the expected range of 3.5 to 5 mEq/L (3.5 to 5 mmol/L); the serum potassium level is not significant in identifying compensated respiratory acidosis.

After abdominal surgery, a goal is to have the client achieve alveolar expansion. The nurse determines that this goal is most effectively achieved by what method? a. Postural drainage b. Pursed-lip breathing c. Incentive spirometry d. Sustained exhalation

c. Incentive spirometry Incentive spirometry expands collapsed alveoli and enhances surfactant activity, thereby preventing atelectasis. Postural drainage helps clear accumulated secretions from the pulmonary tree; it does not directly promote alveolar expansion. Pursed-lip breathing promotes sustained exhalation, not inhalation. Sustained exhalation promotes the collapse, not expansion, of alveoli.

A nurse is assessing a client experiencing a diabetic coma. What unique response associated with diabetic coma that is not exhibited with hyperglycemic hyperosmolar nonketotic syndrome (HHNS) should the nurse identify when assessing this client? a. Fluid loss b. Glycosuria c. Kussmaul respirations d. Increased blood glucose level

c. Kussmaul respirations Kussmaul respirations occur in diabetic coma as the body attempts to correct a low pH caused by accumulation of ketones (ketoacidosis). HHNS affects people with type 2 diabetes who still have some insulin production; the insulin prevents the breakdown of fats into ketones. Fluid loss is common to both because an increased blood glucose level ultimately leads to polyuria. Glycosuria is common to both conditions. Hyperglycemia is common to both conditions.

The nurse prepares an intravenous solution of lactated Ringer solution to replace the T-tube output of a client who had a cholecystectomy and common bile duct exploration. Which condition will improve if the administration of lactated Ringer solution is effective? a. Urinary stasis b. Paralytic ileus c. Metabolic acidosis d. Increased potassium level

c. Metabolic acidosis Lactated Ringer solution is an alkaline solution that replaces bicarbonate ions lost from T-tube bile drainage, thus preventing or treating acidosis. Urinary stasis is unrelated to the effectiveness of the administration of intravenous lactated Ringer solution. Paralytic ileus is unrelated to the effectiveness of the administration of intravenous lactated Ringer solution. An increased potassium level is unrelated to the effectiveness of the administration of intravenous lactated Ringer solution.

The laboratory data for a client with prolonged vomiting reveal arterial blood gases of pH 7.51, Pco 2 of 50 mm Hg, HCO 3 of 58 mEq/L (59 mmol/L), and a serum potassium level of 3.8 mEq/L (3.8 mmol/L). The nurse concludes that the findings support what diagnosis? a. Hypocapnia b. Hyperkalemia c. Metabolic alkalosis d. Respiratory acidosis

c. Metabolic alkalosis Elevated plasma pH and elevated bicarbonate levels support metabolic alkalosis. The arterial carbon dioxide level of 50 mm Hg is elevated more than the expected value of 35 to 45 mm Hg; hypercapnia, not hypocapnia, is present. The client's serum potassium level is within the expected level of 3.5 to 5 mEq/L (3.5 to 5 mmol/L). With respiratory acidosis the pH will be less than 7.35.

A client arrives in the emergency department with epigastric pain and prolonged vomiting. Assessment findings include rapid and shallow respirations, dry and flushed skin, weakness, and lethargy. Which is the primary nursing concern? a. Acute pain b. Risk for injury c. Metabolic alkalosis d. Ineffective breathing

c. Metabolic alkalosis Prolonged vomiting results in fluid loss and acid (hydrochloric) loss; the client's adaptations reflect dehydration and metabolic alkalosis. Although it is important to address the client's pain, the fluid and electrolyte/acid/base imbalance must be addressed first because this imbalance can be life threatening. Although risk for injury is a potential problem, the priority is the fluid and electrolyte/acid/base problem. The ineffective breathing pattern most likely is caused by the metabolic alkalosis; the fluid and electrolyte/acid/base imbalance is a higher priority and must be addressed first.

A client sustained minor skin injuries following an accident. Which event occurs close to the time of injury? a. Thinning of the scar tissue b. Formation of granulation tissue c. Migration of leukocytes to the site of injury d. Arrival of fibroblasts to the site of infection

c. Migration of leukocytes to the site of injury Beginning at the time of injury and lasting 3 to 5 days is the inflammatory phase in which migration of leucocytes takes place. Scar tissue is formed in the maturation phase. Formation of "granulation" tissue and migration of fibroblasts occurs in the proliferative phase.

Which statement best describes the etiology of obesity? a. Obesity primarily results from a genetic predisposition. b. Psychosocial factors can override the effects of genetics in the etiology of obesity. c. Obesity is the result of complex interactions between genetic and environmental factors. d. Genetic factors are more important than environmental factors in the etiology of obesity.

c. Obesity is the result of complex interactions between genetic and environmental factors.

A teenager is admitted with an acute onset of right lower quadrant pain at McBurney point. Appendicitis is suspected. For which clinical indicator should the nurse assess the client to determine if the pain is secondary to appendicitis? a. Urinary retention b. Gastric hyperacidity c. Rebound tenderness d. Increased lower bowel motility

c. Rebound tenderness Rebound tenderness is a classic subjective sign of appendicitis. Urinary retention does not cause acute lower right quadrant pain. Hyperacidity causes epigastric, not lower right quadrant pain. There generally is decreased bowel motility distal to an inflamed appendix.

A client with chronic obstructive pulmonary disease (COPD) has a blood pH of 7.25 and a PCO 2 of 60 mm Hg. What complication does the nurse conclude the client is experiencing? a. Metabolic acidosis b. Metabolic alkalosis c. Respiratory acidosis d. Respiratory alkalosis

c. Respiratory acidosis The pH indicates acidosis [1] [2]; the PCO 2 level is the parameter for respiratory function. The expected PCO 2 is 40 mm Hg. These results do not indicate a metabolic disorder or indicate respiratory alkalosis.

A client complaining of fatigue is admitted to the hospital with a diagnosis of chronic obstructive pulmonary disease (COPD). What should the nurse do to prevent fatigue? a. Provide small, frequent meals b. Encourage pursed-lip breathing c. Schedule nursing activities to allow for rest d. Encourage bed rest until energy level improves

c. Schedule nursing activities to allow for rest Rest limits muscle contractions, which diminishes oxygen needs and decreases fatigue. Although small, frequent meals may decrease pressure on the diaphragm and facilitate breathing, this precaution does not address the client's fatigue. Although pursed-lip breathing facilitates gas exchange, it does not reduce the metabolic demand for oxygen. Bed rest promotes pooling of pulmonary secretions, which may aggravate the client's respiratory status.

An 82-year-old man is being cared for at home by his family. A pressure ulcer on his right buttock measures 1 × 2 × 0.8 cm in depth, and pink subcutaneous tissue is completely visible on the wound bed. Which stage would the nurse document on the wound assessment form? a. Stage I b. Stage II c. Stage III d. Stage IV

c. Stage III

Which assessment finding is considered the earliest sign of decreased tissue oxygenation? a. Cyanosis b. Cool, clammy skin c. Unexplained restlessness d. Retraction of interspaces on inspiration

c. Unexplained restlessness Unexplained restlessness is considered the earliest sign of decreased oxygenation. The other assessment findings, such as cyanosis, cool, clammy skin, and retraction of interspaces on inspiration, are considered late signs of decreased oxygenation.

Which intervention is most likely to prevent or limit barotrauma in the patient with ARDS who is mechanically ventilated? a. Decreasing PEEP b. Increasing the tidal volume c. Use of permissive hypercapnia d. Use of positive pressure ventilation

c. Use of permissive hypercapnia

5. moderate pain with short periods of severe pain during dressing changes is a. probably exaggerating his pain. b. best treated by referral for surgical treatment of his pain. c. best treated by receiving a long-acting and a short-acting opioid. d. best treated by regularly scheduled short-acting opioids plus acetaminophen.

c. best treated by receiving a long-acting and a short-acting opioid.

Unrelieved pain is a. expected after major surgery. b. expected in a person with cancer. c. dangerous and can lead to many physical and psychologic complications. d. an annoying sensation, but it is not as important as other physical care needs.

c. dangerous and can lead to many physical and psychologic complications.

Maintenance of fluid balance in the patient with ARDS involves a. hydration using colloids. b. administration of surfactant. c. fluid restriction and diuretics as necessary. d. keeping the hemoglobin at levels above 9 g/dL (90 g/L).

c. fluid restriction and diuretics as necessary.

Providing opioids to a dying patient who is experiencing moderate to severe pain a. may cause addiction. b. will probably be ineffective. c. is an appropriate nursing action. d. will likely hasten the person's death.

c. is an appropriate nursing action.

The typical fluid replacement for the patient with a fluid volume deficit is a. dextran. b. 0.45% saline. c. lactated Ringer's. d. 5% dextrose in 0.45% saline.

c. lactated Ringer's.

In planning care for the patient with Crohn's disease, the nurse recognizes that a major difference between ulcerative colitis and Crohn's disease is that Crohn's disease a. frequently results in toxic megacolon. b. causes fewer nutritional deficiencies than ulcerative colitis. c. often recurs after surgery, whereas ulcerative colitis is curable with a colectomy. d. is manifested by rectal bleeding and anemia more often than is ulcerative colitis.

c. often recurs after surgery, whereas ulcerative colitis is curable with a colectomy.

A 65-yr-old woman was just diagnosed with Parkinson's disease. The priority nursing intervention is a. searching the Internet for educational videos. b. evaluating the home for environmental safety. c. promoting physical exercise and a well-balanced diet. d. designing an exercise program to strengthen and stretch specific muscles.

c. promoting physical exercise and a well-balanced diet.

A patient with diabetes has a serum glucose level of 824 mg/dL (45.7 mmol/L) and is unresponsive. After assessing the patient, the nurse suspects diabetic ketoacidosis rather than hyperosmolar hyperglycemic syndrome based on the finding of a. polyuria. b. severe dehydration. c. rapid, deep respirations. d. decreased serum potassium.

c. rapid, deep respirations.

A patient with a C7 spinal cord injury undergoing rehabilitation tells the nurse he must have the flu because he has a bad headache and nausea. The nurse's first priority is to a. call the HCP. b. check the patient's temperature. c. take the patient's blood pressure. d. elevate the head of the bed to 90 degrees.

c. take the patient's blood pressure.

Encephalopathy

cerebral edema and accumulation of neurotoxins in the blood • High ammonia levels are a sign Assessment: *Asterixis—flapping tremor *

Why are children at risk for Metabolic acidosis?

children are vulnerable to acid-base imbalance because their metabolic rates are faster and their ratios of water to total body weight are lower o Severe diarrhea and intestinal malabsorption

nursing diagnosis

clinical judgement about the patient in response to an actual or potential health problem

health promotion nursing diagnosis

clinical judgement of motivation, desire, and readiness to enhance well-being and actualize human health potential

Stage IV (Severe) COPD has a REV1 rate of __________of predicted value a) >80 % b) 50-80% c) 30-50% d) less than 50 %

d) less than 50 %

On the first postoperative day after a total hip replacement a client asks for assistance onto the bedpan. What should the nurse instruct the client to do? a. "Use your elbows and hands to lift your pelvis off the bed." b. "Extend both legs and pull on the trapeze to lift your pelvis." c. "Turn gently toward the operative side while lifting your pelvis off the bed." d. "Flex the knee on the unoperated leg and pull on the trapeze to lift your pelvis."

d. "Flex the knee on the unoperated leg and pull on the trapeze to lift your pelvis."

A registered nurse teaches a client and the caregiver about pressure ulcer care. Which statement made by the caregiver indicates the need for further teaching? a. "I should inspect the client's skin daily." b. "I should manage the client's incontinence as quickly as possible." c. "I should properly dispose of the client's contaminated dressings." d. "I should not worry about what the client eats."

d. "I should not worry about what the client eats." The nurse should teach the caregiver about the role that good nutrition plays in enhancing a client's healing to correct this misconception. All the other statements are correct and require no further teaching. The nurse should teach the caregiver to conduct daily skin inspections. The nurse should instruct the caregiver about how to manage a client's incontinence and how to properly dispose of contaminated dressings.

A client with multiple sclerosis is admitted to the hospital. The client's exacerbations have become more frequent and more severe. One day, the client's partner confides to the nurse, "Life is getting very hard and depressing, and I am upset with myself for thinking about a nursing home." After listening to the partner's concerns, which is the best response by the nurse? a. "You may be able to lessen your feelings of guilt by seeking counseling." b. "It would be helpful if you become involved in volunteer work at this time." c. "I recognize it's hard to deal with this, but try to remember that this too shall pass." d. "Joining a support group of people who are coping with this problem may be helpful."

d. "Joining a support group of people who are coping with this problem may be helpful."

An x-ray film indicates that an older client has a fractured femur. The client asks the nurse, "Will I be able to walk again?" What is the best response by the nurse? a. "I have no idea because only time will tell." b. "You only broke a bone. It could have been worse." c. "You'll walk again. This is a common issue in older people." d. "Tell me more about your concerns about being able to walk."

d. "Tell me more about your concerns about being able to walk." The phrase "Tell me more" shows interest in the client's concerns, is nonjudgmental, and encourages expression and exploration of feelings. First the client's feelings must be explored before providing a direct answer that may cut off communication. The responses "I have no idea" and "You only broke a bone. It could have been worse" places the client on the defensive; it is demeaning to the client and discourages further communication. The general response "You'll walk again. This is a common issue in older people" dismisses the client's concerns; the client is not recognized as an individual whose injury is a traumatic and personal event.

A registered nurse is examining the medical reports of different clients. Which client may need immediate assessment? a. A client who is scheduled for a bronchoscopy b. A client who is scheduled for a thoracentesis c. A client with pleural effusion and decreased breath sounds d. A client with acute asthma and 85% oxygen saturation

d. A client with acute asthma and 85% oxygen saturation A client with acute asthma may have low peripheral arterial oxygen saturation. Pulse oximetry results less than 86% requires immediate assessment and treatment. Scheduled bronchoscopies and thoracenteses do not require immediate action. Pleural effusions with decreased breath sounds are an issue, but this condition does not require immediate assessment.

A client with a diagnosis of major depression refuses to participate in unit activities, claiming to be "just too tired." What is the best nursing approach? a. Planning one rest period during each activity b. Explaining why the staff believes that the activities are therapeutic c. Encouraging the client to express negative feelings about the activities d. Accepting the client's feelings about activities calmly while setting firm limits

d. Accepting the client's feelings about activities calmly while setting firm limits Fatigue and apathy are symptoms of depression and should be accepted; however, limits should be set to facilitate participation in unit activities. Planning one rest period during each activity allows the client to manipulate the environment. Explaining why the staff believes that the activities are therapeutic will not change the client's mind about them, and this response does not show an understanding of the client's needs. Encouraging the client to express negative feelings about the activities will reinforce negative feelings about participating in them.

A patient is admitted to the burn center with burns to his head, neck, and anterior and posterior chest after an explosion in his garage. On assessment, the nurse auscultates wheezes throughout the lung fields. On reassessment, the wheezes are gone, and the breath sounds are greatly diminished. Which action is the most appropriate for the nurse to take next? a. Encourage the patient to cough and auscultate the lungs again. b. Obtain vital signs, oxygen saturation, and a STAT arterial blood gas. c. Document the findings and continue to monitor the patient's breathing. d. Anticipate the need for endotracheal intubation and notify the physician.

d. Anticipate the need for endotracheal intubation and notify the physician.

A nurse in a mental health unit of the emergency department of a hospital frequently cares for adolescents who attempt suicide. What is important for the nurse to remember about adolescent suicide behavior? a. Boys account for more attempts than do girls. b. Girls use more dramatic methods than do boys. c. Girls talk more about suicide before attempting it. d. Boys are more likely to use lethal methods than are girls.

d. Boys are more likely to use lethal methods than are girls. The finding that boys are more likely to use lethal methods than are girls is supported by research; girls account for 90% of suicide attempts, but boys are three times more successful because of the methods they use

A client is admitted with suspected atelectasis. Which clinical manifestation does the nurse expect to identify when assessing this client? a. Slow, deep respirations b. Normal oral temperature c. Dry, unproductive cough d. Diminished breath sounds

d. Diminished breath sounds Because atelectasis [1] [2] involves collapsing of alveoli distal to the bronchioles, breath sounds are diminished in the lower lobes. The client will have rapid, shallow respirations to compensate for poor gas exchange. Atelectasis results in an elevated temperature. Atelectasis results in a loose, productive cough.

The clinical findings of a client with diabetes mellitus show decreased glucose tolerance. Which complication is anticipated in the client? a. Cystitis b. Thin and dry skin c. Decreased bone density d. Frequent yeast infections

d. Frequent yeast infections Decreased glucose tolerance may cause frequent yeast infections, but it is not associated with the risk of cystitis, thin and dry skin, and decreased bone density. The risk of cystitis, thin and dry skin, and decreased bone density are due to decreased ovarian production of estrogen.

A client with type 1 diabetes receives 30 units of NPH insulin at 7 am. At 3:30 pm the client becomes diaphoretic, weak, and pale. What does the nurse determine that these physiologic responses are associated with? a. Diabetic coma b. Somogyi effect c. Diabetic ketoacidosis d. Hypoglycemic reaction

d. Hypoglycemic reaction

Which drug does the nurse recognize as an effective mood-stabilizing drug used in clients with bipolar disorder and in the acute treatment of mania and prevention of recurrent mania and depressive episodes? a. Doxepin b. Clozapine c. Amitriptyline d. Lithium carbonate

d. Lithium carbonate Lithium carbonate is often the first choice of treatment, once primary acute mania has been diagnosed, to calm acute manic symptoms and relieve recurrent mania.

A client with parkinsonism is taking an anticholinergic medication for morning stiffness and tremors in the right arm. During a visit to the clinic, the client complains of some numbness in the left hand. What is the nurse's priority intervention? a. Refer the client to the primary healthcare provider only if other neurologic deficits are present. b. Ask the primary healthcare provider to increase the client's dosage of the anticholinergic medication. c. Stress the importance of having the client call the primary healthcare provider as soon as possible. d. Make arrangements immediately for further medical evaluation by the client's primary healthcare provider.

d. Make arrangements immediately for further medical evaluation by the client's primary healthcare provider. Numbness, a sensory deficit, is inconsistent with parkinsonism; further medical evaluation is necessary. Numbness, even in the absence of other problems, may be indicative of an impending brain attack (cerebrovascular accident, CVA). This symptom is not caused by parkinsonism; increasing the dosage of the anticholinergic medication will not be helpful. Stressing the importance of having the client call the primary healthcare provider as soon as possible can cause a delay in the client's receiving immediate medical attention.

A nurse must establish and maintain an airway in a client who has experienced a near-drowning in the ocean. For which potential danger should the nurse assess the client? a. Alkalosis b. Renal failure c. Hypervolemia d. Pulmonary edema

d. Pulmonary edema Additional fluid from surrounding tissues will be drawn into the lung because of the high osmotic pressure exerted by the salt content of the aspirated ocean water; this results in pulmonary edema. Hypoxia and acidosis may occur after a near-drowning, not alkalosis. Renal failure is not a sequela of near-drowning. Hypovolemia occurs because fluid is drawn into the lungs by the hypertonic saltwater.

Which guideline would be a part of teaching patients how to use a metered-dose inhaler (MDI)? a. After activating the MDI, breathe in as quickly as you can. b. Estimate the amount of remaining medicine in the MDI by floating the canister in water. c. Disassemble the plastic canister from the inhaler and rinse both pieces under running water every week. d. To determine how long the canister will last, divide the total number of puffs in the canister by the puffs needed per day.

d. To determine how long the canister will last, divide the total number of puffs in the canister by the puffs needed per day.

The primary purpose of hospice is to a. allow patients to die at home. b. provide better quality of care than the family can. c. coordinate care for dying patients and their families. d. provide comfort and support for dying patients and their families.

d. provide comfort and support for dying patients and their families.

An appropriate nursing intervention for a patient with pneumonia with the nursing diagnosis of ineffective airway clearance related to thick secretions and fatigue would be to a. perform postural drainage every hour. b. provide analgesics as ordered to promote patient comfort. c. administer O2 as prescribed to maintain optimal O2 levels. d. teach the patient how to cough effectively to bring secretions to the mouth.

d. teach the patient how to cough effectively to bring secretions to the mouth.

actual nursing diagnosis

describes human responses to health conditions or life processes

risk nursing diagnosis

describes human responses to health conditions/life processes that may develop

interpersonal skills

developing a trusting relationship, expressing a level of caring, and communicating clearly with a patient and his or her family

The nurse is considering the needs of the postoperative client in the home setting. The nurse is performing: discharge planning. initial planning. ongoing planning. comprehensive planning.

discharge planning. Explanation: Discharge planning begins at the time of admission with the nurse teaching the client and family specific skills necessary for self-care behaviors in the home. Comprehensive planning occurs from time of admission to time of discharge and includes initial, ongoing, and discharge planning. Initial planning is done at time of admission based on the nurse's admission assessment. Ongoing planning is conducted by any nurse caring for the client throughout the nurse-client relationship.

medical diagnosis

identification of a disease condition based on specific evaluation of signs and symptoms

in 1980 and 1995

the American Nurses Association (ANA) included diagnosis as a separate activity in its publication Nursing: a Social Policy Statement.

indirect care

treatments performed away from the patient but on behalf of the patient or group of patients examples: -managing the patient's environment (safety and infection control) -documentation -interdisceplinary collaboration -behind the scenes

direct care

treatments performed through interactions with patients examples: -medication administration -insertion of an intravenous (IV) infusion -counseling during a time of grief


Kaugnay na mga set ng pag-aaral

English Unit 2: Becoming a Nation

View Set

CPP - Certified Payroll Professional Exam for 2019 limits

View Set

Project management short answer test 1

View Set